Old Exam Questions Flashcards
Old man with multiple comorbidities, acute chole with cholecystoduodenal fistula, see air fluid level in GB, stone impacted in cystic duct, improved on admission with antibiotics, management:
a. Repair fistula
b. Continue abx
c. Cholecystostomy tube
d. Ercp, sphincterotomy
B
Post chole, 11mm CBD, no stones
a. ercp
b. HJ
c. Sphincter of oddi manometry
d. Ercp with sphincterotomy
e. choledochojejenostomy
D
Type I SOD = Biliary pain, AbN Liver enzymes, Dilated CBD, delayed drainage
Type II SOD = Biliary pain plus one or tow of above
Type III SOD= Biliary pain plus none of the above
Work up includes: U/S, MRCP (secretin enhanced), EUS, CT abdo, Biliary scintigraphy with CCK, and ERCP
Type I and II improve with ERCP and sphincterotomy. Type III is functional problem
Lady prego 16 weeks, gallstone pancreatitis, feels better now and biochemical resolution. What to do?
a. lap chole and gram
b. change diet and do chole after birth
c. mrcp
d. ercp
A
Youngish lady with jaundice and inflamed GB with imaging consistent with Mirizzi I (stone in Hartmanns impacting CBD)
a. ercp and stent
b. lap chole and gram
c. lap chole
d. open chole
A in answers but maybe B?
Type I is extrinsic compression of the CBD by an impacted gallstone: total or subtotal
Type II involves one-third the circumference of CBD :
Type III involves two-thirds the circumference of the bile duct.
Type IV involves the whole circumference of the bile duct.
Type V involves types I to IV with the addition of a bilioenteric fistula
●Type I – Partial or total cholecystectomy, either laparoscopic or open. Common bile duct exploration is typically not required (see ‘Laparoscopic surgery’ below).
●Type II – Cholecystectomy plus closure of the fistula, either by suture repair with absorbable material, T tube placement, or choledochoplasty with the remnant gallbladder.
●Type III – Choledochoplasty or bilioenteric anastomosis (choledochoduodenostomy, cholecystoduodenostomy, or choledochojejunostomy), is required, depending on the size of the fistula. Suture of the fistula is not indicated.
●Type IV – Bilioenteric anastomosis, typically choledochojejunostomy, is preferred because the entire wall of the common bile duct has been destroyed.
Endoscopic treatment can be effective as a temporizing measure before surgery and can be definitive treatment for unsuitable surgical candidates. Endoscopic removal of common bile duct stones also avoids the need for common bile duct exploration at the time of surgery.
Lady with symptomatic gallstones but incidental finding of 4.5cm liver lesion with central scar and mild compression of IVC
a. lap chole
b. chole and right hepatectomy
c. embolize
d. RFA
A
Sounds like liver lesion is an FNH
50F presents with ascites RUQ pain and delirium. Imaging shows suprahepatic vein and retrohepatic IVC obstruction. Bili 88 INR 1.3
a. Transplant
b. Heparin
c. TIPS
d. portocaval shunt
A
Acute Budd-Chiari syndrome. Heparin, plasty/stent, TIPS/shunting, transplant. If liver failure refer to transplant centre. If not, medical therapy. In this case, pt has liver failure.
Lady found to have lesion in pancreas for w/u for gastritis of some kind (?) that lights 11mm, in isthmus
a. central pancreatectomy with distal roux-en-Y pancreatecojejunostomy
b. subtotal pancreatectomy
c. central panc with no anastomosis
d. observe
A or D
<1cm NF-PNET: observe
>2cm: resect
1-2cm: controversial so shouldn’t be any questions on this, but favour resection (enucleation vs formal resection. NCCN observes <1cm.
Enucleation possible for NF-PNET <2-3cm; if >3cm, definitely formal resection. Even small PNETs can be malignant.
Alcoholic liver failure, ascites, portal vein thrombus, resistant to diuretic and Na restriction, what to do
a. TIPS
b. transplant
c. portovenous shunt
d. paracentesis
D
PV thrombus is a contraindication to TIPS
Young female, few days post op, post op bile leak, HIDA shows drainage from GB fossa, mrcp normal
a. perc drain
b. ercp
c. laparotomy
A
Lipschitz says ERCP sometimes does work
Hiatus hernia, comes in with gastric volvulus, incarceration, reduced in OR, stomach viable, what to do next in the OR?
a. gastrostomy tube
b. repair crura, reinforce with mesh, do fundo
c. proximal gastrectomy
d. crural repair
D in answer key.
Although A is not an unreasonable option.
Perineal hernia for 75 yr old lady after apr, asymptomatic
a. observe
b. repair through laparotomy
c. repair through perineum
A
Lap incisional hernia repair on 34yo man. You make a 2mm SB enterotomy with no spillage. What to do big boy?
a. fix bowel laparoscopically and abandon then fix hernia in one week
b. continue with repair as planned
c. lap repair with biologic mesh
d. open repair with biologic mesh
A
Safe recommendation is to delay the repair or do tissue-based.
If a true enterotomy is not made and only a serosal injury has occurred, then the bowel can be oversewn laparoscopically, depending on the skill set of the surgeon. In this case, the lumen is not entered and there is no contamination; it is usually acceptable to proceed with mesh placement. If a full-thickness injury occurs, subsequent management depends on several factors, including the defect size, amount of spillage, and surgeon experience. In this situation, there should be a low threshold for converting to an open operation if such an approach is needed to ensure adequate repair of the injury. If an open approach is used, then the hernia may be repaired either primarily or by implanting a biological mesh. However, several laparoscopic options have also been described. It may be possible to repair the injury laparoscopically, complete the adhesiolysis, and delay the hernia repair. For this protocol, the patient is admitted to the hospital, kept on antibiotics, and returned to the operating room in 2 to 6 days for laparoscopic mesh placement.9,10 In select cases with minimal or no spillage, several reports documented successful outcomes after proceeding with synthetic mesh placement after laparoscopic repair of the enterotomy during the same operation.
Incisional hernia repair with mesh, didn’t say lap or open, now with 2mm opening in skin draining some fluid and 20cm seroma on CT, what to do
a. open skin and place dressing
b. open skin and do vac
c. perc drain
d. abx and sterile dressing
C
better control, complete evacuation of fluid before it becomes infected.
Guy returns with pain at base of penis after open hernia repair. Why?
a. Injury to ilioinguinal nerve
b. Injury to genitofemoral nerve
c. Iliohypogatric injury
A
Ilioinguinal - Supplies sensory innervation to the proximal and medial thigh. In females it innervates the mons pubis and labium majus; in males it innervates the root of the penis and upper scrotum.
Genitofemoral – scrotum, cremaster, labium majus, mons pubis
Iliohypogastric – same as ilioinguinal
Lateral cutaneous – anterolateral thigh
Old guy returns with painful swollen testicle after open hernia repair with reduction of large amount of omentum from indirect sac at time of OR. What to do? (He’s five or so days out)
a. Warm compress and elevation of scrotum
b. Take back to OR
c. Antibiotics
d. Orchidectomy
A
Ischemic orchitis usually occurs from thrombosis of the small veins of the pampiniform plexus within the spermatic cord. This results in venous congestion of the testis, which becomes swollen and tender 2 to 5 days after surgery. The process may continue for an additional 6 to 12 weeks and usually results in testicular atrophy. Ischemic orchitis also can be caused by ligation of thetesticular artery. It is treated with anti-inflammatory agents and analgesics. Orchiectomy is rarely necessary.
The incidence of ischemic orchitis can be minimized by avoiding unnecessary dissection within the spermatic cord. The incidence increases with dissection of the distal portion of a large hernia sac and in patients who have anterior operations for hernia recurrence or for spermatic cord pathology. In these situations, the use of a posterior approach is preferred. Testicular atrophy is a consequence of ischemic orchitis. It is more common after repair of recurrent hernias, particularly when an anterior approach is used. The incidence of ischemic orchitis increases by a factor of three or four with each subsequent hernia recurrence.
Post-op bariatric surgery, 2 yrs ago now. Prior lap chole. Lost weight. Returns with RUQ pain and imaging showing bowel in RUQ. U/S non-contributory. Likely diagnosis?
a. Stricture at jej-jej
b. Afferent syndrome
c. Internal hernia
C
Patient with minimal leaking from umbilical hernia, history of cirrhosis, mx?
a. paracentesis & diuretics
b. TIPS ascites and diuretics
c. paracentesis & repair
A
Patients with advanced liver disease, ascites, and umbilical hernia require special consideration. Enlargement of the umbilical ring usually occurs in this clinical situation as a result of increased intra-abdominal pressure from uncontrolled ascites. First line of therapy is aggressive medical correction of the ascites and paracentesis for tense ascites with respiratory compromise. These hernias are usually filled with ascitic fluid, but omentum or bowel may enter the defect after large-volume paracentesis. Uncontrolled ascites may lead to skin breakdown on the protuberant hernia and eventual ascitic leak, which can predispose the patient to bacterial peritonitis. Patients with refractory ascites may be candidates for transjugular intrahepatic portocaval shunt (TIPS) or eventual liver transplantation. Umbilical hernia repair should be deferred until after the ascites is controlled.
Urgent operation only for incarceration
Chronic giant inguinal hernia. Most likely complication?
a. respiratory distress
b. hydrocele
c. wound infection
A
45 y M alcoholic with portal HTN and ascites. 3cm umbilical hernia with ulceration. ascites is refractory to diuretics. best mgmt?
a. TIPS
b. levine shunt
c. surgical repair (and leave drain, medical management, TIPS postop)
d. large volume paracentesis
Answer key says C or D
34 F post C-section with infraumbilical 4cm abdo wall mass. Bx shows desmoid
a. Sulindac and tamoxifen (OR)
b. Radiation
c. Resect with primary closure
d. Resect with mesh closure
A
Shift towards observation of abdominal wall DT; pregnancy and hormonal factors drive progression, but only 50% of peri-pregnancy DT required surgery. All comers, 1/3 progress, 1/3 regress, 1/3 remain stable. Observation is now 1st line for asymptomatic. Size >7cm predictors failure of observation and should be resected as delay may ultimately require greater extent of surgery. RT if not surgical candidate or high morbidity with surgery. If surgery is required, ~95% require mesh reconstruction.
Inguinal hernia repaired with mesh. did not specify open or lap. 15 y ago. has a recurrence that has incarcerated but spontaneously resolved.
a. lap hernia repair
b. open hernia repair
c. tissue repair
d. biologic mesh repair
A
Obese with incisional hernia. how to best prevent infection. infraumbilical after C-section
a. laparoscopic hernia repair
b. component seperation
c. open repair with mesh
A
Anterior boundary of Spigalian Hernia
a. Coopers
b. External Oblique
c. Transversalis
B
The hernial orifice of a Spigelian hernia is located in the Spigelian fascia, that is, between the lateral border of the rectus abdominis muscle and the semilunar line, through the transversus abdominis aponeurosis, close to the level of the arcuate line.
What is the posterior border of inguinal canal?
a. Transversalis and peritoneum
b. Coopers
c. Transversalis fascia and transversus muscle
d. Internal and transversus muscles
C
Boundaries
- Anterior: External oblique + internal oblique
- Posterior: Transversalis fascia and transversus abdominsus
- Superior: Internal oblique and transversus abdominis
- Inferior: Inguinal ligament
Femoral Hernia with discoloured bowel - you are accessing via a proximal thigh transverse incision. Can’t deliver up bowel. What is the BEST way to release the bowel?
a. midline laparotomy and deliver
b. Divide cooper
c. Divide conjoint
d. Divide transversalis and inguinal
D
Old guy who gets a plug n patch inguinal hernia repair under local anaesthetic. Tries to get up and walk, but collapses. What is the next best management?
a. MRI
b. Reposition mesh
c. Remove mesh
d. Observe
D?
2 year old kid with 2cm umbilical hernia. Previously presented with incarceration that was red and tender but spontaneously reduced and now reduces spontaneously/easily.
a. Observe
b. Repair primarily
c. Repair with mesh
d. Repair at 4 years old
B
In general, umbilical hernia has a tendency to close on its own in approximately 80% of cases, and therefore elective repair should be deferred until approximately 5 years of age. Also, umbilical hernia is rarely associated with significant complications but there are unique exceptions to this general rule, for which an earlier elective repair should be considered. Although rare, a history of incarceration clearly warrants prompt surgical repair, irrespective of age. Enlarging umbilical hernia over time, in particular with a large skin proboscis more than 3 cm, or a significantly large umbilical fascial defect (>2 cm) is unlikely to resolve spontaneously; therefore, surgical repair should be considered at an early age.
Child delivered at 28 weeks gestational age. Now at 38 weeks. Before discharge home, mother finds lump in groin that spontaneously reduces. On exam, you can’t find it, but she points to the location where an inguinal hernia would be. Management?
a. Bilateral exploration
b. Fix when incarcerates
c. Unilateral repair before d/c home
d. Fix when clinically confirmed.
A
Selective contralateral exploration is offered after consultation with the parents of patients who fall into the following categories: girls younger than 5 years with a unilateral hernia, and premature infants. In boys with a question of bilateral disease, the authors recommend laparoscopy through the umbilicus to evaluate the contralateral side.
Sabiston:
Although early hernia repair may be associated with a higher risk for injury to the cord structures, recurrence rate, and postoperative apneic episodes, most pediatric surgeons advocate operative repair prior to discharge from the hospital for premature infants because of their significant risks for incarceration. However, for those infants diagnosed after hospital discharge, elective hernia repair may be deferred until the infant is beyond 52 weeks postconceptional age, when postoperative apnea risk decreases. In patients presenting with incarcerated inguinal hernia, unless there is clinical evidence of peritonitis, attempts are made to reduce the hernia. Manual reduction is successful in up to 70% of cases. Once reduced, the patient is admitted for observation and hernia repair is performed at 24 to 48 hours, when local tissue edema resolves. A nonreducible incarcerated hernia should be promptly explored in the operating room. Routine contralateral inguinal exploration at the time of symptomatic hernia repair for infants is standard practice based on the high incidence of contralateral patent process vaginalis (4% to 65%). However, the issue regarding the routine exploration of the asymptomatic contralateral side in toddlers remains unresolved. Most pediatric surgeons routinely explore the asymptomatic contralateral side in children 2 years of age or younger; some surgeons extend the contralateral exploration criteria to those up to 5 years of age.
47 F had previous laparotomy for MVC and had splenectomy. PMHx of DM, HTN, and OA, BMI 44. Presented with an incarcerated incisional hernia that spontaneously reduced. Management?
a. Laparoscopic repair
b. Open repair
c. Repair when incarcerates again
d. Laparoscopic sleeve gastrectomy
Answer Key says D
Multiple comorbidities that would improve with bariatric procedure; unacceptably high rate of recurrence with BMI 44.
Laparoscopic ventral hernia repair with mesh, got sick taken back to OR. find a 2 cm colotomy in transverse colon with feculent peritonitis
a. Deliver the injury as a loop colostomy
b. Primary Repair colon
c. Resect mesh, loop colostomy, primary repair of hernia
d. Resect mesh, ?repair colon, biologic mesh of hernia
C
One month post inguinal hernia. Numbness to lateral scrotum. Nerve injured?
a. Ilioinguinal
b. Iliohypogastric
c. Genitofemoral
C
Old man with enlarging scrotum. Transilluminates. No palpable hernias in groin. Mgmt.
a. Scrotal exploration
b. Inguinal exploration and repair indirect inguinal hernia
c. Transscrotal drainage
Answer Key says A
Sounds like a hydrocele? Ultrasound?
Young guy after inguinal hernia repair. Now with ischemic orchitis. Cause?
a. vascular compression at external ring
b. vascular compression at internal ring
c. ligation of testicular artery
d. thrombosis of testicular vein
D
What’s the most important step for component separation?
a. divide internal oblique
b. divide laterally on external oblique aponeurosis
c. mobilize rectus bilaterally
B
What is the transversalis fascia attached to?
a. inguinal ligament
b. coopers ligament
c. conjoint tendon
d. pubic tubercle
A
Older guy with history of incarcerated inguinal hernia that was reduced. Suppose to have elective hernia repair but then had unstable angina and went for PCI. Had drug eluding stents placed and was discharged home with ASA and plavix. When to do his elective hernia repair?
a. 4-6weeks
b. 6 months
c. 1 year
d. when he incarcerates
C
2012 ACC/AHA: dual antiplatelet for 12 months due to high rate of delayed stent thrombosis
UTD: 12 months DAPT for DES, in patients at high risk for bleeding, can go to single after 6 months
Patient post right hemi dehisced. Didin’t say if he was skinny or obese.
a. VAC
b. Close with mesh (if Vicryl)
c. Close with retention sutures
d. Reclose fascia primarily with running suture
C
Management of the incision is a function of the condition of the fascia. When technical mistakes are made and the fascia is strong and intact,primary closure is warranted. If the fascia is infected or necrotic, débridement is performed. The incision can then be closed with retention sutures; however, to avoid tension, use of a prosthetic material may be preferred. Closure with an absorbable mesh (polyglactin or polyglycolic acid) may be preferable because the mesh is well tolerated in septic wounds and allows bridging the gap between the edges of the fascia without tension, prevents evisceration, and allows the underlying cause of the patient’s dehiscence to resolve.
Femoral hernia elective repair in female. What does transversalis fascia attach to
a. Conjoint tendon
b. Inguinal ligament
c. Pubic tubercle
d. Cooper’s ligament
Answer key says D. Maybe B?
superiorly it is continuous with the inferior diaphragmatic fascia
posteriorly it fuses with thoracolumbar fascia and is lost in the fat covering the posterior surface of the kidney
inferiorly it is attached to the iliac crest and posterior margin of the inguinal ligament
inferior to the inguinal ligament it is attached to the inguinal ligament by a fibrous tissue that forms the iliopubic tract
medial to the femoral vessels it attaches to the pubis and pectineal line. It descends and forms the anterior part of the femoral sheath
Healthy 75 yo male with an acute onset of a bulging mass LLQ, presents to ER with 4h history, no peritonitis. US revealed the abnormality to be in the abdomnal wall only. What would you do next:
a. CT scan
b. I & D tonight in ER
c. sigmoidoscopy in AM
d. follow up in AM
e. barium enema
A
Dx is ? spigelian hernia or rectus sheath hematoma
Which is the commonest type of hernia in a 75 yo female:
a. indirect
b. direct
c. femoral
d. epigastric
e. umbilical
A
Which anatomic structure makes up the posterior wall of the inguinal canal:
a. transversalis fascia
b. external oblique aponeurosis
c. lacunar ligament
d. Cooper ligament
e. transversalis muscle
A
Boundaries of the inguinal canal (Fig. 9-17) are as follows:
- Anterior: The anterior boundary is the aponeurosis of the external oblique muscle and, more laterally, the internal oblique muscle. Remember, there are no external oblique muscle fibers in the inguinal area, only aponeurotic fibers.
- Posterior: In about ¾ of subjects, the posterior wall (floor) is formed laterally by the aponeurosis of the transversus abdominis muscle and the transversalis fascia; in the remainder, the posterior wall is transversalis fascia only. Medially the posterior wall is reinforced by the internal oblique aponeurosis.
- Superior: The roof of the canal is formed by the arched fibers of the lower edge (roof) of the internal oblique muscle and by the transversus abdominis muscle and aponeurosis.
- Inferior: The wall of the canal is formed by the inguinal ligament (Poupart’s) and the lacunar ligament (Gimbernat’s).
Groin hernia that does not reduce completaly:
a. combination of femoral and indirect
b. combination direct and indirect
c. sliding
C
A sliding inguinal hernia is a protrusion of a retroperitoneal organ through an abdominal wall defect.
If all contents of the sac cannot be reduced, adhesions of viscera to the sac or a sliding component may be present.
Femoral hernia, all except:
a. less common in men
b. >50% incarcerated if patient >50
c. most common hernia in females
d. cause of meralgia paresthetica
e. frequently missed in indirect hernia repair
f. more likely to strangulate
C and D
Most common hernia in females is indirect inguinal
Metalgia paresthetica is syndrome of a dull ache, sharp pain, burning sensation, tingling or numbness in the distribution of the lateral cutaneous nerve of the thigh. Usually assoc with hernai repair
The most common cause of incisional hernia:
a. wound infection
b. diabetes mellitus
c. malnutrition
d. chronic bronchitis
e. obesity
A
An obturator hernia can display all except:
a. pain
b. SBO
c. mass
d. referred pain along the medial aspect of the thigh
C
D is also known as Howship Romberg sign
Meralgia paresthetica is caused by which nerve:
a. lateral cutaneous nerve of the thigh
b. genitofemoral
c. ilioinguinal
d. iliohypogastric
A
What hernia is most likely to recur: a. indirect b. Littres’s hernia c. incisional hernia d. femoral hernia Richter’s hernia
C
80 yo male straining develops pain in the LLQ. Examination revealed tenderness and no disappearance with straight leg raise. US reveals mass confined to anterior abdo wall. He is otherwise well. Your next management:
a. CT scan tonight
b. I & D
c. sigmoidoscopy
d. observe overnight and reassess in AM
A
? Spigelian hernia vs rectus sheath hematoma
A spigelian hernia;
a. located between the linea semilunaris and edge of rectus sheath
b. describes a hernia between rectus abdo muscles
c. easily detected
d. detected with US
A
But technically it is along linea semilunaris, lateral to edge of rectus sheath, but not between the two
With respect to obturator hernias
a. usually presents with mass
b. inguinal incision is best approach
c. may be palp with rectal exam
d. pain radiates to vulva
e. obesity increases risk
C
A non-reducible hernia is likely:
a. sliding
b. femoral and direct
c. direct and indirect
d. richter’s
e. litre’s
A
Hernia recurrence most common in repair of:
a. incisional hernia
b. femoral hernia
c. umbilical hernia
d. inguinal hernia
A
A 3 month old has a reducible umbilical hernia with a 6 mm defect. What is the appropriate management:
a. Urgent OR in 72 hrs
b. Elective OR in 6 weeks
c. Reduce and tape hernia
d. Reassurance
D
Femoral hernia cause all EXCEPT:
a. more common in women
b. can present as a lump in the groin
c. can be mistaken for an abscess
d. can cause meralgia paresthetica
e. can be missed with an indirect hernia repair
D
10 months boy with Rt inguinal hernia. Family history of bilateral inguinal hernias (father, brother).
a. unilateral hernia repair, explore other side when symptomatic
b. image other side
c. unilateral hernia repair, explore other side concurrently
d. unilateral hernia repair, use laparoscope through processus vaginalis
D
Explore contralateral side in preemies and girls. In this case, family history is indication.
Old man, scrotal swelling, transilluminates, non reducible, no hernia detected
a. inguinal exploration, fix indirect inguinal hernia
b. transscrotal drainage, fixation
c. transscrotal excision of mass
Answer key says C (aka hydrocelectomy)
Best overall answer is hydrocoelectomy. Consider imaging to rule out varicocele from obstructed gonadal vein (retroperitoneal mass).
Open inguinal hernia. Back with swelling and pain from incision. SC emphysema of perineum.
a. Surgical intervention
b. ABx
c. Perc drain
A
Post open inguinal hernia repair with mesh, returns with redness at mid incision with fluctuance. Not septic
a. open and drain, abx
b. observe
c. remove mesh
d. perc drain
A
Sabiston: most groin mesh infections can be salvaged with antibiotics; trial long-term abx before mesh removal.
Nerve injured in lap hernia repair giving pain to lateral scrotum
a. femoral branch of genitofemoral
b. genital branch of genitofemoral
c. genital branch of iliohypogastric
d. ilioinguinal
B
Alcoholic male with incarcerated inguinal hernia. At OR find dusky small bowel requiring resection, and a moderate amount of ascites. Best management?
a. Something with mesh
b. Lichtenstein repair
c. Bassini repair
d. Repair with biologic mesh and drain ascites
C
Incisional hernia repair, least risk of fistula formation
a. goretex
b. vicryl
c. proline
d. polyester
B
- most common complication of hernia repair for inguinoscrotal hernia
a. recurrence
b. urinary retention
c. Infection
B
Hematoma/seroma: 4-6%
Urinary retention: 2.1%
Infection: 0.3-0.6%, recurrence 1-2% if Lichtenstein
75 year old man with small, minimally symptomatic, reducible right inguinal hernia. On exam can palpate the hernia at internal ring. Management?
a. Observe
b. Laparoscopic repair
c. Plug and patch repair
d. Lichtenstein repai
A
Debate about watchful waiting in men outside the 40-65 age range. The JAMA study (Watchful Waiting vs Repair of Inguinal Hernia in Minimally Symptomatic MenA Randomized Clinical Trial) included men age 18->65. 56% were 40-65, leading some to conclude that the results are only applicable to white men 40-65. However, 32% were >65, and the authors did not make any statements about age criteria, so I believe watchful waiting applies.
At 11 years, crossover to repair was 62% for men <65 and 79% for men >66. Hernia-related adverse events were low, only 0.0018 per patient year. There was no difference in surgical complications between up-front surgery and delayed surgery.
As Sabiston discusses, probably a greater risk in the elderly is decreased activity which is necessary for maintaining quality of life. If the hernia is impairing activity, it should probably be fixed as the risk of repair is low compared to risk of functional decline. Also, M&M is much higher in emergent repairs.
Patient with parastomal hernia that is somewhat symptomatic. Hernia is fully reducible. What is the BEST management?
a. Repair with plication of tissues
b. Resite the colostomy
c. Mesh repair
d. biologic mesh
C
Doing a laparoscopic repair of a large incisional hernia you make a colotomy. Minimal fecal contamination. Best management? (Definitely said “minimal”.)
a. Laparoscopic repair of colotomy, washout, abandon hernia repair
b. Laparotomy, washout, repair colotomy, suture repair of hernia
c. Laparoscopic repair of colotomy and continue with hernia repair
d. Laparotomy, washout, repair colotomy, biological mesh repair
Answer key says B
A is reasonable if comfortable with laparoscopic repair of colon injury
An injury to the colon carries a much greater potential for a sinister outcome if the hernia repair is completed as planned. Therefore, the majority of surgeons will simply repair the injury and repair the hernia by a laparotomy. Some, however, have carried out the hernia repair with a simultaneous colon resection as a preplanned operation with an adequately prepared colon [18]. An unplanned event should terminate the laparoscopic repair. The further management of the injury and hernia repair should proceed similarly to that of the small bowel injury.
Best indication for laparoscopic inguinal hernia repair
a. Previous hernia repair as a child
b. Direct hernia
c. Bilateral hernias
d. Femoral hernia
C
Obese lady for ventral hernia repair, how do you best reduce wound infection rate?
a. Overlay mesh
b. Laparoscopic repair with mesh
c. Component separation
d. Primary repair
B
Randomized trials and observational studies have provided adequate evidence that the laparoscopic approach to incisional hernia repair has a lower incidence of surgical site and mesh infection compared with open mesh repair. Laparoscopic repairs are also less painful, and patients recover more quickly.
A systematic review involving 1003 patients from 11 trials found lower incidences of wound drainage (2.6 versus 67 percent) and wound infection (2.8 versus 16.2 percent) for laparoscopic incisional hernia repair compared with open hernia repair
POD 10 elective hysterectomy with peritonitis and sepsis from perforated diverticulitis has a 3cm incisional hernia from remote bladder surgery. Gets a Hartman for her diverticulitis. How do you deal with the hernia?
a. Repair with biologic mesh
b. Repair with absorbable mesh
c. Primary repair with secondary closure of skin
d. Primary repair with component separation and secondary closure of skin
C
POD 7 open inguinal hernia repair with mesh comes with pain, fluctuance and redness at incision, vitals normal, afebrile, WBC normal.
a. Drain and antibiotics
b. Mesh removal and biologic mesh
c. Mesh removal and absorbable mesh
d. Mesh removal
A
Alcoholic male presents with retrosternal pain after vomiting. Imaging shows small contained leak in mid esophagus. Stable. Improves on conservative mgmt..
a. Observe
b. Stent
c. Thoracotomy and primary repair
d. Laparotomy and drain mediastinum
A
Surgery is not indicated for every patient with a perforation of the esophagus and management is dependent on several variables—stability of the patient, extent of contamination, degree of inflammation, underlying esophageal disease, and location of perforation.
Scope screening for Barrett’s, distal esophageal perforation seen in distal mediastinum. Worsens with conservative management
a. Contune antibiotics
b. Stent
c. L thoracotomy
d. Laparotomy and
C
If it was upper do R thoractomy
Patient with paraesoph. Hiatus hernia with evidence of pain. Taken to the OR with contents reduced, no necrotic stomach. Mgmt?
a. Crural repair
b. Crural repair with prosthetic mesh and fundo
A
Roux en Y esophageal-jejunostomy, CXR post-operatively reveals large pleural Lt effusion. Best initial management.
a. Upper GI series, water soluble
b. Barium swallow
c. Endoscopy
A
Gastrograffin followed by thin barium if no leak seen.
Patient with submucosal lesion of esophagus, best management?
a. endoscopic biopsy
b. thoracic enucleation
c. esophagectomy
B
Biopsy not required and contraindicated b/c makes adequate pathologic material to exclude malignancy is impossible to obtain and violation of the mucosal layer may complicate subsequent surgical resection. Diagnosis best on barium swallow. Esophageal endoscopic ultrasonography (EUS) may further help in the diagnosis, planning of surgery, and follow-up of these tumors.
Resect if symptoms, >2cm, or cannot r/o GIST. EUS is recommended by some, biopsy is useful but makes eventual surgery more difficult.
Sabiston
Leiomyomas are slow-growing tumors with rare malignant potential that continue to grow and become progressively symptomatic over time. Although observation is acceptable in patients with small (<2 cm) asymptomatic tumors or other significant comorbid conditions, surgical resection is advocated for most leiomyomas; however, imatinib (a tyrosine kinase inhibitor), as targeted therapy used on other GIST tumors, may have some benefit for esophageal leiomyomas. Surgical enucleation of the tumor remains the standard of care and is performed through a thoracotomy or with video or robotic assistance. Lesions of the proximal and midesophagus are removed through the right chest; those of distal origin are removed through the left chest.
Endoscopy with biopsy for barretts. develops retrosternal CP in recovery. On imaging see a small contained leak in the distal mediastinum. start abx and resuscitate and continues to have temp of 38.2 and chest pain. what to do big boy?
a. continue resuscitation
b. transabdominal repair with fundo
c. left thoractomy and repair
d. endoscopic stent
C
35y M going for hellers for achalasia with a toupet. 1mm perforation 6cm proximal to GEJxn.
a. open repair with Thal
b. stent
c. lap repair and continue planned surgery
d. lap repair and Dor
A
Need to open to manage this complication. Thal brings up fundus as an onlay patch.
80 yr male POD #5 Total gastrectomy for ca with esophagojejunostomy. WBC elevated, dyspneic, febrile. Imaging shows large left pleural effusion. Drains 1600mL foul-smelling clear, Gram +ve organisms. Likely etiology of this presentation?
a. empyema
b. pneumonia
c. anastomotic leak
d. subphrenic abscess
C
Open fundo years ago with complete SBO. No radiologic improvement after 24 hours, symptoms improved with NG
a. Laparotomy and LOA
b. Continue non-operative management
c. Long intestinal tube
d. Lap LOA
B
Best repair method for recurrence of open inguinal hernia repair with mesh A. Laparoscopic B. Open C. Both D. Neither
A
If the initial repair was a tissue repair, either anterior or posterior approaches can be used for repair of the recurrent hernia. If the initial repair was a mesh repair, then the recurrent repair should preferably employ an approach in the space in which the tissues planes have not been violated previously.
Selecting one approach over the other also depends on the experience and comfort of the surgeon with that technique. Although a laparoscopic approach might be preferable in previously repaired anterior tissue repair, the absence of a mesh with its associated scarring and fibrosis makes a redo anterior approach with mesh more feasible and acceptable, especially if laparoscopic expertise is not available.
Best repair method for recurrence after initial tissue inguinal hernia repair A. Laparoscopic B. Open C. Both D. Neither
C
Recurrence after initial laparoscopic inguinal hernia repair in a 50 year old A. Laparoscopic B. Open C. Both D. Neither
B
Acute cholecystitis and septic shock on multiple pressors A. Lap Chole B. Perc Cholecystostomy tube C. Both D. Neither
B
Only 2 prospective trials have been conducted on percutaneous cholecystostomy in high surgical risk acute calculous cholecystitis. Both showed no significant difference in mortality and possibly reduced costs and morbidity. However, percutaneous cholecystostomy can be performed safely on patients who are not otherwise operative candidates, such as ASA class IV and V and patients with severe sepsis. Percutaneous cholecystostomy may be the only treatment required for acalculous cholecystitis in the critically ill, with success rates between 57% and 100%.
Management of acalculous cholecystitis A. Lap Chole B. Perc Cholecystostomy tube C. Both D. Neither
C Only 2 prospective trials have been conducted on percutaneous cholecystostomy in high surgical risk acute calculous cholecystitis. Both showed no significant difference in mortality and possibly reduced costs and morbidity. However, percutaneous cholecystostomy can be performed safely on patients who are not otherwise operative candidates, such as ASA class IV and V and patients with severe sepsis. Percutaneous cholecystostomy may be the only treatment required for acalculous cholecystitis in the critically ill, with success rates between 57% and 100%. However, successful laparoscopic cholecystectomy is reported in surgical candidates.
Management of asymptomatic gall bladder polyp 5 mm in diameter without stones or signs of malignancy A. Lap Chole B. Perc Cholecystostomy tube C. Both D. Neither
D
Gallbladder polyps that are 6–9 mm on ultrasound without signs of malignancy or risk factors for gallbladder cancer (e.g., age over 60, sessile morphology, gallstones, primary sclerosing cholangitis) can be safely observed with interval ultrasound studies.
Management of Acute cholecystitis and American Society of Anesthesiologists class III A. Lap Chole B. Perc Cholecystostomy tube C. Both D. Neither
A
Promotes early cellular and vascular infiltration A. Cross-linked biologic matrix B. Non-cross-linked biologic matrix C. Both D. Neither
B
Biologic matrices provide a scaffold for tissue and vascular ingrowth. Today they are popular in abdominal wall reconstruction. Cross-linking is a chemical step that attempts to render the collagen of the biologic matrix less prone to degradation in vivo by naturally occurring collagenases. Cross-linking also delays cellular infiltration and neovascularization in the short term.
A gradual remodeling of an implanted tissue graft is essential in abdominal wall repair, because too rapid of an absorption of the matric scaffold will result in graft failure and recurrence of the hernia. Because cross-linked matrixes are poorly incorporated by the host, they become encapsulated with fibrous tissue and act as foreign body material instead of serving as a scaffold for remodeling. Non-cross-linked matrices actually promote early vascularization and cellular ingrowth.
Intraperitoneal placement is acceptable for: A. Cross-linked biologic matrix B. Non-cross-linked biologic matrix C. Both D. Neither
C
Performs best when placed between 2 layers of vascularized tissue: A. Cross-linked biologic matrix B. Non-cross-linked biologic matrix C. Both D. Neither
C
Delayed or no remodeling A. Cross-linked biologic matrix B. Non-cross-linked biologic matrix C. Both D. Neither
A
Management of 5-cm focal nodular hyperplasia (FNH), asymptomatic A. Enucleation B. Hypertonic saline injection C. Observation D. Resection to negative margins E. Transarterial embolization
C
Focal nodular hyperplasia (FNH) is the second most common benign hepatic lesion. FNH has no potential for malignancy, spontaneous rupture, or hemorrhage. The only indications for resection of FNH are symptoms (abdominal pain) and inability to rule out malignancy. A definitive diagnosis, however, is usually possible with modern imaging techniques. Asymptomatic FNH does not require intervention. Annual ultrasound for 2–3 years is prudent for women who wish to continue oral contraceptive (OCP) use.
Management 5-cm hepatic adenoma, asymptomatic A. Enucleation B. Hypertonic saline injection C. Observation D. Resection to negative margins E. Transarterial embolization
D
Hepatic adenomas (HA) are benign liver masses that tend to be hormonally sensitive. The classic presentation is in young women (aged 20s–40s) with a history of OCP use and in men with a history of anabolic steroid use. Many HAs regress with discontinuation of OCPs and steroids. Management strategies of HA are more aggressive than for other benign liver masses, because they have a risk of hemorrhage and malignant degeneration to hepatocellular carcinoma (HCC). The risk of hemorrhage or an occult focus of HCC increases in HAs greater than 5 cm in size. Indications for surgery include (1) size at least 5 cm, (2) male patient, and (3) inability to rule out malignancy. Management of an acute rupture with hemorrhage is initial angioembolization followed by elective resection. If possible, resection to negative margins should be done in case underlying HCC is identified on final pathologic analysis.
Management 5-cm hepatic peripheral hemangioma with abdominal pain A. Enucleation B. Hypertonic saline injection C. Observation D. Resection to negative margins E. Transarterial embolization
A
Hepatic hemangiomas are the most common benign neoplasm of the liver. Hemangiomas have no risk of malignant degeneration and a very low risk of bleeding. Lesions smaller than 5 cm rarely cause symptoms. An asymptomatic hemangioma can be safely observed. Hemangiomas that bleed may be embolized. Indications for surgery are symptoms (recurrent abdominal pain) and extremely large lesions (>10 cm). For those requiring surgery, resection is the treatment of choice. Enucleation is possible in many cases, because the hemangioma tends to compress adjacent liver tissue, creating a plane of dissection.
Management 5-cm hepatic adenoma with active hemorrhage A. Enucleation B. Hypertonic saline injection C. Observation D. Resection to negative margins E. Transarterial embolization
E
Which has the shortest time for absorption A. Chromic gut B. Polyglactin 910 C. Polyglecaprone 25 D. Polydioxanone E. Silk
B
Gut = highly purified collagen processed from the submucosa layer of sheep intestine or the serosa layer of beef intestine. Absorbed based on whether it is plain or chromic, the tissue it is used in, and the general health status of the patient. Plain is absorbed by 70 days, with tensile strength maintained for 7–10 days. Plain gut treated with a chromium salt solution is called chromic gut. Prolongs absorption time to >90 days, with tensile strength retained for 10–14 days.
Polyglactin 910 or Vicryl = synthetic absorbable suture formed from a copolymer of lactide and glycolide (from lactic and glycolic acid). The lactide slows water penetration into the suture, which improves the duration of tensile strength. Retain ~ 65% of original tensile strength 14 days after implantation. Absorption is minimal for ~ 40 days and complete by 56–70 days.
Polyglecaprone 25 or Monocryl is virtually inert in time and absorbs predictably. At 7 days, 50–60% of initial strength remains, reduced to 20–30% at 14 days, with all original strength lost at 21 days. Absorption is complete at 91–119 days.
Polydioxanone or PDS is a polyester that retains 70% of tensile strength at 14 days postimplantation, 50% at 28 days, and 25% at 42 days. Absorption is minimal until approximately the 90th day postoperatively and essentially complete within 6 months.
Silk suture is braided raw silk. Surgical silk loses tensile strength when exposed to moisture. Although silk is classified as a nonabsorbable suture, long-term studies show that by 1 year, it has lost all of its tensile strength and usually cannot be detected in tissue after 2 years.
Intermediate absorption: A. Chromic gut B. Polyglactin 910 C. Polyglecaprone 25 D. Polydioxanone E. Silk
C
Absorbed by proteolytic enzymatic digestive process A. Chromic gut B. Polyglactin 910 C. Polyglecaprone 25 D. Polydioxanone E. Silk
A
Presence of 2 hyperplastic polyps on initial screening colonoscopy A. Repeat colonoscopy in 10 years B. Repeat colonoscopy in 3 years C. Repeat colonoscopy in 6 months D. Repeat colonoscopy in 1 year E. CT colonography in 6 months
A
Hyperplastic polyps do not portend any increased risk of developing colorectal malignancy and therefore require no increased surveillance from the standard 10-year follow-up.
Presence of 2 tubular adenomas, each less than 1 cm and without dysplasia, on initial screening colonoscopy A. Repeat colonoscopy in 10 years B. Repeat colonoscopy in 3 years C. Repeat colonoscopy in 6 months D. Repeat colonoscopy in 1 year E. CT colonography in 6 months
A
Patients with 1 or 2 adenomatous polyps, less than 10 mm in diameter, are considered low risk, and 10-year follow-up is recommended.
Presence of 1.1-cm tubular adenoma with dysplasia, margin of resection negative on initial screening colonoscopy A. Repeat colonoscopy in 10 years B. Repeat colonoscopy in 3 years C. Repeat colonoscopy in 6 months D. Repeat colonoscopy in 1 year E. CT colonography in 6 months
B
Patients with 3 or more adenomatous polyps, 1 or more polyps greater than 10 mm in size, or adenomatous polyps with villous or high-grade histology are considered high risk, and 3-year follow-up is recommended.
Presence of 0.5-cm tubulovillous adenoma without dysplasia, margin of resection negative on initial screening colonoscopy A. Repeat colonoscopy in 10 years B. Repeat colonoscopy in 3 years C. Repeat colonoscopy in 6 months D. Repeat colonoscopy in 1 year E. CT colonography in 6 months
B
A 65-year-old woman with a BMI of 45 presents with a midline incisional hernia defect after a total abdominal hysterectomy for fibroids. The defect is infraumbilical, 6 cm wide, and reducible. In the treatment of this ventral hernia, laparoscopic and open repair have similar rates of which of the following? A. Mesh infection B. Wound morbidity C. Cost of therapy D. Hernia recurrence E. Intraoperative complications
D
Incisional hernia is the most frequent complication after laparotomy, resulting in approximately 200,000 repairs annually. Both laparoscopic and open approaches are successfully used in the surgical treatment of incisional hernias.
Multiple comparisons have been made, examining each technique’s advantages and disadvantages vis-à-vis the other. The laparoscopic approach has a lower rate of wound complications, mesh infection, and hospital length of stay compared with the open approach. This advantage is offset by higher major or severe complications, particularly intraoperative enterotomy, and hospital costs.
Recurrence rates are similar for both approaches. The laparoscopic approach is especially recommended in obese patients or elderly patients with small to medium defects because of its minimal flap creation and soft tissue dissection.
Which of the following statements regarding the nonoperative management of uncomplicated acute appendicitis is true?
A. No randomized trials compare nonoperative versus operative management.
B. Most failures of nonoperative management occur within the first year after treatment.
C. Patients treated with surgery have fewer complications than patients treated nonoperatively.
D. Nonoperative management is associated with increased mortality.
E. Failure of initial nonoperative management is associated with an increased risk of developing complicated appendicitis.
B
Recently, multiple RCTs were conducted comparing nonoperative management with antibiotics to surgery in patients with uncomplicated appendicitis. Nonoperative management, had a success rate of 44–85% in individual studies. Most recurrences in the nonoperative group occur within the first year after treatment.
Surgery is associated with a higher rate of complications compared with nonoperative management. Mortality was similar between operative and nonoperative management. A meta-analysis of 4 randomized trials consisting of 900 patients found a relative risk reduction in complications of 31% in the antibiotic treatment group compared with the appendectomy group. Nonoperative management is associated with shorter length of stay and decreased sick leave time compared with surgical management. Antibiotic therapy may delay appendectomy in patients who are not improving, but this delay is not associated with an increased risk of developing complicated appendicitis, including perforation or peritonitis.
Which of the following statements is true regarding the diagnosis and management of hepatocellular carcinoma (HCC)?
A. PET scan is the preferred surveillance imaging in high-risk populations.
B. Alpha-fetoprotein greater than 3 times normal is diagnostic.
C. A tissue diagnosis is required before initiating therapy.
D. Screening reduces the overall mortality associated with HCC.
E. Triple-phase CT is diagnostic in high-risk populations.
E
In patients with cirrhosis, the diagnosis of HCC can be made with triple-phase CT scan. Liver Imaging Reporting and Data System (LiRADS) criteria include the presence of arterial-phase enhancement (hypervascularity), portal venous phase washout, and threshold growth if a historical comparison is available. The scale runs from 1 (definitely benign) to 5 (diagnostic of HCC). LiRADS 4 and 5 lesions can be treated without a biopsy. A tissue diagnosis may be necessary in low-risk patients and when there is nondiagnostic imagining. Serum alpha fetoprotein has a high positive predictive value when greater than 500 ng/mL but a low negative predictive value. Approximately 20% of patients with HCC have a normal alpha fetoprotein. Alpha fetoprotein levels are not used as a diagnostic criterion for HCC.
An effective screening approach that significantly reduces HCC-related mortality has not been identified. Serial serum alpha fetoprotein screening in more than 5000 chronic hepatitis B carriers in China identified more early-stage cancers than the unscreened population but did not significantly reduce mortality. The authors concluded this was likely a function of a lead-time bias—diagnosing the disease earlier but not changing the disease-specific mortality.
PET scans are neither sensitive nor specific for the diagnosis of HCC.
A patient with a cystic mass in the head of the pancreas undergoes cross-sectional imaging and cystic fluid sampling. Which of the following clinical presentations of a cystic mass in the pancreas has the highest risk of malignancy?
A Patient: 70-year-old woman Size: 8 cm Contents: Serous fluid Main duct width: 1 cm
B Patient: 80-year-old man Size: 1.5 cm Contents: Mucinous fluid Main duct width: 0.4 cm
C Patient: 65-year-old man Size: 5 cm Contents: Mucin Main duct width: 1.5 cm
D Patient: 32-year-old man with a recent history of acute pancreatitis Size: 6 cm Contents: Serous fluid Main duct width: 0.8 cm
E Patient: 55-year-old woman Size: 1.5 cm Contents: Mucin Main duct width: 0.5 cm
C
Determinants of the malignant potential of a pancreatic cyst include mucin in the cyst contents, cyst size, solid tumor in the cyst, and main pancreatic duct dilation. Loculation within the cyst does not predict malignancy.
Cysts containing serous fluid are generally benign and include serous cystadenomas and benign epithelial cysts. Accordingly, the patient with the largest mucin-containing cyst and greatest dilation of the main duct represents the patient with the highest risk of associated malignancy.
Mucin-containing cysts include intraductal papillary neoplasms (IPMN) and mucinous cystic tumors (MCN). MCN occur almost exclusively in women 55–65 years old, whereas IPMN occur in older men and women, with men predominating. Current recommendation from guidelines recommend resection of all MCN, main duct IPMN, branch duct-IPMN with a solid component, main pancreatic duct size at least 1 cm, obstructive jaundice, or cytology suspicious or positive for cancer. Although guidelines recommend resection of the unilocular, mucin-containing cyst in the 55-year-old woman as it is a presumed MCN, the risk of malignant cells within the lesion is less than the 65-year-old man with the large IPMN with main duct involvement.
A 56-year-old man with Child-Pugh B cirrhosis with hepatitis C is found to have a 6-cm mass in segments II and III with arterial enhancement and venous washout on triple-phase CT. His alpha-fetoprotein level is 450 ng/mL. An indocyanine green clearance (ICG) is obtained to assess his suitability for resection. The minimal percentage clearance at 15 minutes after injection of ICG needed to proceed with surgery is A. 50%. B. 60%. C. 70%. D. 80%. E. 90%.
D
Although only 30% of patients with HCC may qualify for operative treatment, surgery remains the best option available. Because the leading cause of death after resection for HCC is liver failure, proper preoperative assessment of hepatic reserve is essential to identify patients at risk. Both the Child-Pugh and Model for End-Stage Liver Disease classifications can aid in determining those patients at greatest risk. For example, Child-Pugh C patients have a greater than 25% perioperative mortality, and, consequently, resection is contraindicated.
Triphasic liver CT can be used to calculate the future liver remnant. A future liver remnant less than 40% is predictive of postoperative liver failure. Finally, indocyanine green (ICG) clearance can be used to assess the functional capacity of the liver. The dye is injected into the bloodstream, and clearance from the liver is measured 15 minutes after injection. Less than 10% of the dye should be detectable at this point in normal functioning livers. With 15–20% retention, a 2-segment resection is possible. With 21–30% retention, a single segment or wedge resection is indicated. Greater than 40% retention of dye at 15 minutes is predictive of postoperative liver failure regardless of resection size.
In this patient requiring a 2-segment resection for treatment of his 6-cm HCC, the minimum ICG clearance would be 80%, corresponding to a 20% retention rate, which would allow performance of a 2-segment resection without the development of liver failure.
A 55-year-old alcoholic man presents to clinic with signs and symptoms consistent with chronic pancreatitis. CT reveals enlargement of the pancreatic head with multiple stones and strictures in his proximal main pancreatic duct. In discussing treatment options with him, which of the following would result in the best long-term pain control?
A Endoscopic sphincterotomy and stone extraction alone
B Endoscopic sphincterotomy, stone extraction, and stent placement
C Longitudinal pancreaticojejunostomy
D Longitudinal pancreaticojejunostomy with limited resection head of pancreas
E Pancreaticoduodenectomy with reconstruction
D
Palliative options for long-term chronic pancreatitis-induced pain include procedures involving pancreatic head resections, such as the Beger or Frey procedure or pancreaticoduodenectomy (Whipple procedure), surgical drainage procedures, or endoscopic ductal decompression. Options for surgical drainage can include procedures such as the Puestow procedure or modifications of this classic operation such as the Izbicki operation or Hamburg or Berne procedure.
The Beger procedure includes duodenum-sparing resection of most of the pancreatic head with division of the pancreatic body over the portal vein and reconstruction via a side-to-side and side-to-end pancreaticojejunostomy to drain the remaining head and tail of the pancreas. The Frey procedure combines a duodenal-sparing pancreatic head resection with overlay pancreaticojejunostomy and longitudinal duct drainage without division of the pancreas. The Puestow procedure consists of opening the duct in a longitudinal fashion and draining it via a lateral pancreaticojejunostomy while preserving the head of the gland. Endoscopic treatment for pain related to chronic pancreatitis can be accomplished with sphincterotomy and stone extraction with or without ductal dilation or stent placement for main duct decompression.
A randomized controlled trial demonstrated that surgical drainage of the pancreas required fewer reinterventions and was more effective in controlling pain and improving quality of life than endoscopic decompression. Approximately 20–25% of patients required reintervention after endoscopic decompression versus approximately 5% of surgically drained patients. There were no statistically significant differences in reintervention rates between patients undergoing Beger versus Frey procedures.
Rates of exocrine insufficiency are typically more than 60% in patients undergoing either endoscopic decompression or surgery, and rates appear similar between Beger and Frey procedures. Rates of endocrine insufficiency appear to be lower than exocrine insufficiency and are similar between endoscopic and surgical drainage procedures. Endocrine insufficiency is lower with pancreatic head parenchymal-sparing surgical procedures than pancreaticoduodenectomy, but significant differences have not been seen between the different types of pancreatic parenchymal sparing operations.
Randomized controlled trials show superior results from duodenal-preserving pancreatic head resections compared with pancreaticoduodenectomy. Mortality from chronic pancreatitis may result from surgical complications if operative drainage is performed; other causes include pancreatic cancer, gastrointestinal hemorrhage, or cardiovascular disease as a result of diabetes from endocrine insufficiency. In fact, cardiovascular disease secondary to endocrine insufficiency is the most common cause of death after Beger and Frey procedures.
A 52-year-old man undergoes an uneventful open antrectomy with Roux-en-Y reconstruction for treatment of bleeding type II gastric ulcer. Optimal fascial closure to minimize hernia development in relation to closure technique, suture type, and suture-to-wound length ratio is A interrupted, monofilament, 3:1. B interrupted, braided, 3:1. C interrupted, monofilament, 4:1. D continuous, monofilament, 4:1. E continuous, braided, 4:1.
D
The use of nonabsorbable or slowly absorbable sutures has a lower hernia rate than quickly absorbable sutures. A continuous suture technique leads to a more rapid and stronger wound closure than interrupted sutures. Monofilament sutures result in fewer surgical site infections compared with braided sutures. A suture length-to-wound length ratio higher than 4-to-1 decreases the rate of incisional hernia by 4-fold. Finally, the use of 2-0 monofilament sutures mounted on a small needle (e.g., 20-mm tapered half-circle) with the placement of stitches within the aponeurosis 5–8 mm from the wound edge and 4–5 mm apart helps to minimize the rate of incisional hernia. Thus, for this patient, the use of a continuous, monofilament sutures with a 4:1 suture-to-wound length ratio is optimal.
Which of the following statements is true regarding necrotizing pancreatitis?
A. Contrast-enhanced CT is the imaging modality of choice to determine the extent of necrosis.
B. Enteral nutrition should be avoided for 10–14 days.
C. Broad-spectrum antibiotics should be given.
D. Pancreatic necrosis mandates immediate surgical intervention.
E. Minimally invasive and percutaneous approaches to pancreatic necrosis are contraindicated.
A
Necrotizing pancreatitis is seen in approximately 1 in 5 patients with pancreatitis and is associated with a mortality rate of 10–30%. Patients are at high risk for multiorgan failure and secondary infection of the necrotic pancreatic bed. The risk of secondary infection is 25–70%, adding to the already high rate of morbidity and mortality. After initial admission and resuscitation, patients with severe pancreatitis should undergo imaging to determine the extent of necrosis. Contrast-enhanced axial imaging is the modality of choice. The true extent of necrosis may not be apparent for several days. CT offers the additional benefit of assessing for further local complications associated with severe pancreatitis.
Enteral nutrition should be started within 72 hours. The goal is to achieve an early positive nitrogen balance. If gastric feeds are not tolerated, continuous, jejunal feeds should be used. In the setting of a paralytic ileus, parenteral nutrition can be used until resolution of the ileus.
Sterile necrosis must be distinguished from infected necrosis, because these 2 processes are managed differently. Antibiotics have no apparent role in the setting of sterile necrosis. Although some early studies supported the use of antimicrobial therapy in the setting of severe necrosis, 2 large, randomized, double-blinded, multicenter studies and a Cochrane meta-analysis determined prophylactic antibiotics were not protective.
Infected necrosis warrants debridement. There is no indication for surgical debridement of sterile necrosis.
Recently, several reports described the benefits of using a “step-up” approach to debridement. This approach advocates beginning with radiologic drainage, followed by minimally invasive retroperitoneal necrosectomy if drainage fails. This approach seeks to avoid the high morbidity and mortality associated with open necrosectomy.
A 67-year-old man presents with a several-month history of vague abdominal fullness and aching. He has episodes of flushing and hypertension. He has been on tricyclic antidepressants for 5 years. He was adopted as a child and does not know his relatives’ medical history. Preliminary laboratory values are normal for complete blood count, electrolytes, liver function tests, and clotting studies. A 24-hour urinary catecholamine collection is elevated. An MRI is obtained and shown in figure 1 (image shows large left sided RP mass above kidney). Which of the following statements is true?
A This mass arises from the kidney.
B Genetic testing is not recommended.
C Obtaining plasma-free metanephrine off medication is the next appropriate test.
D Determination of the molecular marker p53 predicts the biologic behavior of this mass.
E Hypertension is associated with elevated cardiac index.
C
This patient presents with an impressive mass in the retroperitoneal space on the left. Twenty-four-hour urinary catecholamine collection is elevated. Therefore, unless proven otherwise, this patient has a pheochromocytoma. The enlarged mass and the appearance of other nodules in the periaortic area strongly suggest a malignant pheochromocytoma (figure 2). This tumor does not arise from the kidney, and attention should be turned to the appropriate preoperative workup and management of this patient in preparation for surgery.
A pheochromocytoma is a tumor arising from the adrenomedullary chromaffin cells that commonly produce one or more catecholamines, including epinephrine, norepinephrine, and dopamine. In this case, urine catecholamines are strongly suggestive of pheochromocytoma, but this could be a false-positive. This patient has been on tricyclic antidepressants for 5 years. Therefore, checking peak values off the medication is clearly the best answer. Other medications that may cause falsely elevated tests for plasma and urine metanephrines include acetaminophen, labetalol, and cocaine.
The literature states that 80–90% of patients with pheochromocytoma have sustained or paroxysmal hypertension. This is mostly due to increased peripheral resistance. The hypertensive pattern in patients with pheochromocytoma is very similar to those with primary hypertension with the exception of the tachycardia these patients can experience. Hypertension in pheochromocytoma is usually characterized by a high peripheral resistance and low cardiac index.
Since 1990, 14 different susceptibility genes have been reported for patients with pheochromocytoma. Current guidelines recommend all patients with pheochromocytoma be engaged in “shared decision making for genetic testing.” This patient does not know his family history, and therefore, could possibly harbor a genetic predisposition. Identifying patients who may harbor the gene is of extreme importance for offspring. The p53 gene appears to have no role in predicting the behavior of pheochromocytoma.
A 75-year-old woman with familial adenomatous polyposis who has had a total colectomy complains of the new onset of intermittent abdominal pain, which is worse with eating. An MRI shows 2 masses in the mesentery. Which of the following is the most likely diagnosis? A Desmoid B Liposarcoma C Carcinoid D Lymphoma E Leiomyoma
A
The relative risk of developing desmoid tumors is much higher in patients with familial adenomatous polyposis (FAP) compared with the general population. Desmoids occur in 7.5–16% of FAP patients. Gardner syndrome, considered a variant of FAP, includes extracolonic manifestations such as osteomas and desmoid tumors. Given her history of FAP and the location of the masses, a diagnosis of desmoid is most likely.
Diagnosis of desmoids and other aggressive fibromatosis (AF) is made based on clinical, radiological, and histological parameters. AF/desmoids characteristically infiltrate deep tissue and muscles as opposed to pushing the adjacent tissue, as is seen in most sarcomas. MRI is an excellent tool for the identification and characterization of mesenteric masses (figure 1 and table 1). On MRI, the lesions are infiltrative with an irregular or lobulated contour. Homogeneous isointensity or mild hyperintensity on T1-weighted images and heterogeneous high signal on T2-weighted images is seen.
For symptomatic patients, treatment should be based on the location of the tumor and potential morbidity of treatment. Treatment options include resection, radiation, or systemic therapy. Radiation can be preoperative, intraoperative, or postoperative. Systemic therapies for desmoids include anti-inflammatory drugs, hormonal (tamoxifen) agents, biologic agents (the tyrosine kinase inhibitor imatinib), or chemotherapy (doxorubicin). The decision making for treatment recommendations should be multidisciplinary and should include the preferences and quality-of-life considerations of the patient.
For patients with complex abdominal wall hernias,
A. incisional hernias that result from a surgical site infection should be considered complex.
B. a retrorectus technique is equivalent to an overlay or underlay technique.
C. biologic mesh is superior compared with permanent mesh.
D. small-pore mesh incorporates tissue better than does large-pore mesh.
E. cross-linked biologic mesh degrades faster than non–cross-linked biologic mesh.
A
Complex abdominal wall hernias represent a therapeutic dilemma for surgeons. They are fraught with a high recurrence rate, and the best technique for their repair is not yet established. Complex abdominal wall hernias are defined by clinical and patient factors. Clinical factors include open abdomen management, previously infected wound, previously repaired hernia, or hernias with loss of domain. Patient factors include obesity, active smoking, diabetes, or chronic obstructive pulmonary disease. These hernias have a much higher rate of recurrence than do noncomplex hernias and, thus, require a more extensive operation.
Although the best operation for these hernias has yet to be determined, a retrorectus procedure is commonly used. In this repair, the mesh is placed between the rectus muscle and the posterior rectus sheath. The fascia is then closed above and below the mesh. This approach may require a release or separation of the muscular layers so the fascia can be closed without tension. The can be done using either an open or laparoscopic technique.
Other principles guiding the use of mesh in the repair of hernias are as follows:
The use of mesh seems to reduce recurrence rates and is recommended for the repair of complex hernias.
The use of synthetic mesh is associated with an increased risk of infection.
Mesh placed in the retrorectus position has lower complication rates, including recurrence, than does placement of mesh in the overlay or underlay positions.
Every attempt should be made to close the abdominal wall native tissue regardless of the positioning of the mesh.
Although the use of biologic mesh may reduce the infection rate, biologic meshes are thought to have a higher recurrence rate and are not considered superior in overall outcomes.
Large-pore or lightweight mesh allows better tissue incorporation of the mesh, which is thought to help reduce the recurrence rate and possibly infection rate.
Cross-linking is a feature of biologic mesh, and it is thought to prevent the degradation of the mesh, resulting in a lower incidence of recurrence.
Which of the following statements regarding inguinal hernias is true?
A. A transabdominal laparoscopic repair is preferred for asymptomatic patients.
B. Postoperative pain occurs less often when the ilioinguinal nerve is divided.
C. Urinary retention is a common complication after laparoscopic hernia repair.
D. The majority of asymptomatic patients become symptomatic from their hernia within 1 year.
E. Severe postoperative pain occurs in 10% of repaired patients.
C
Urinary retention is reported to occur in up to 22% of patients after laparoscopic hernia repair. Risk factors include age greater than 60, a history of benign prostatic hypertrophy, and operating time greater than 2 hours. Urinary retention can still occur in young patients and seems to be more prevalent after laparoscopic repairs compared with open ones.
When comparing laparoscopic repairs with open repairs, laparoscopic repairs tend to have longer operating times but similar recurrences and a faster return to work. Currently, there is no definitive evidence that one method (open, total extraperitoneal, or transabdominal) shows superiority over the others, so the approach can be determined by surgeon preference and patient choice, even in asymptomatic patients. Regarding asymptomatic patients, surgical repair is not mandatory, because only approximately 17% of patients will develop symptoms within 1 year from diagnosis, and this is typically not an incarcerated hernia.
Chronic postoperative pain is a concern after inguinal repairs. Chronic pain can occur in up to 11% of patients, but only about 3–4% of patients have severe or activity-limiting symptoms. There is no definitive evidence that dividing the ilioinguinal nerve routinely during an open repair results in a reduction in the incidence of this chronic pain syndrome.
Regarding idiopathic thrombocytopenic purpura, which of the following statements is true?
A The spleen sequesters platelets.
B Significant splenomegaly is usually present.
C Long-term response to steroids is more than 50% in adults.
D Diagnosis can be made using a peripheral blood smear.
E The disease is usually self-limited for children younger than 5 years.
Idiopathic thrombocytopenic purpura (ITP) is a disease in which antibodies are formed against platelets, which results in consumption of the platelets and thrombocytopenia. These antibodies are created in the spleen, and a splenectomy is curative in approximately 75–85% of patients. ITP is associated with mucosal bleeding, petechiae, purpura, and ecchymoses. The diagnosis is made by the presence of antiplatelet antibodies seen on serum testing.
For ITP, first-line medical therapy is the use of steroids to prevent the formation of antibodies; 50–75% of adults will respond to the initial use of steroid therapy, but long-term response rates are much lower at 15–20%. If steroids do not improve the symptoms, a recurrence occurs, or serious bleeding occurs, then anti-IgG antibody can be used. In pediatric patients, ITP is self-limited in the majority of patients, especially by the age of 5. Transfusion, steroids, and anti-IgG antibody therapy is usually avoided in pediatric patients unless the patient has life-threatening or symptomatic bleeding. When the platelet count is less than 20,000/mm3 (14,000–44,000/mm3), pediatric patients are at risk for intracranial hemorrhage.
By contrast, thrombocytopenic purpura (TTP) causes thrombocytopenia, because the spleen sequesters the platelets and results in splenomegaly. TTP is associated with a low platelet count, hemolytic anemia, neurologic complications, renal failure, mental status changes, and bilateral lower extremity petechiae. Here the diagnosis can be made by blood smear testing, which will show schistocytes, nucleated red blood cells, and basophilic stippling. The treatment is usually plasmapheresis and transfusion of fresh frozen plasma (not platelets), but splenectomy can be considered when the platelet count and symptoms are refractory to plasmapheresis. Splenectomy is curative in only approximately 40% of patients.
Which of the following statements is true regarding the management of sigmoid diverticulitis?
A Colonoscopy is not needed after a first episode of uncomplicated diverticulitis.
B Routine elective resection in patients younger than 50 years is recommended.
C Colectomy followed by primary anastomosis with ileostomy may be the optimal strategy for selected patients with perforated diverticulitis.
D Routine elective resection should be performed in patients with 2 or more episodes of uncomplicated diverticulitis.
E Laparoscopic lavage is indicated for feculent peritonitis from perforated diverticulitis.
C
Prior guidelines supported routine elective resection in patients younger than 50 and in patients with 2 or more episodes of uncomplicated diverticulitis. Newer evidence suggests that these groups are not at an increased risk of complications with successive episodes and that the decision to proceed with elective sigmoidectomy should be individualized. An additional recommendation is that colonoscopy be performed within 6–8 weeks after an acute episode of diverticulitis if it is the first episode or if colonoscopy was not performed recently. Although this practice is debated, the concern is that a small subset of patients (1–2%) will in fact have cancer, ischemia, or inflammatory bowel disease.
Laparoscopic lavage is gaining popularity as an alternative surgical approach for patients with Hinchey type II and III disease by CT imaging (table 1).
The procedure involves laparoscopic examination of the phlegmon and irrigation of the abdomen but no disturbance of the inflammatory process. Drains may or may not be placed. Patients receive antibiotics and are followed clinically. Critics argue that the risk of continuing or recurrent infection is a concern when the septic focus remains. Current data support laparoscopic lavage in Hinchey type III purulent peritonitis, but no supportive data exist for laparoscopic lavage in Hinchey type IV feculent peritonitis.
Traditionally, all patients who had an urgent operation for perforated diverticulitis underwent resection with a colostomy. This dogma is now challenged because of the high risk of complications associated with colostomy takedown and colostomy closure rates are much lower than ileostomy closure rates. Thus, colectomy with a primary anastomosis and proximal ileal diversion for Hinchey types III or IV disease may be optimal in select individuals versus subjecting all patients to a routine Hartmann procedure.
Compared with laparoscopic incisional hernia repair with synthetic mesh, open incisional hernia repair with synthetic mesh is associated with A similar seroma rates. B increased readmission rates. C increased reoperative rates. D decreased total costs. E decreased mortality.
A
Comparing open and laparoscopic repair of incisional hernias with mesh, there is no difference in the early outcomes of seroma, reoperation (about 13%), readmissions (2%), or mortality (<1%). The risk of recurrence is higher with open repair (approximately 20% vs. 15%). Laparoscopic hernia repair is associated with increased operating room costs (supplies and longer operative times) but has shorter total hospital length of stay compared with open repair. The net effect is that total costs for both procedures are similar.
Which of the following perioperative antibiotic regimens is associated with the lowest risk of surgical site infection after colorectal surgery?
A Oral antibiotics alone
B Oral antibiotics and intravenous fluoroquinolone and metronidazole
C Oral antibiotics and intravenous cefazolin and metronidazole
D Oral antibiotics and intravenous cefazolin
E Intravenous cefotetan alone
C
Colorectal procedures are associated with a relatively high SSI rate (5–30%).
In a Cochrane database analysis of 680 trials involving more than 43,000 patients, the investigators concluded that combined oral and intravenous antibiotic prophylaxis reduced SSI compared with intravenous antibiotics alone or oral antibiotics alone. In a review of elective colorectal procedures using the Veterans Affairs Surgical Quality Improvement Program, 12% of almost 6000 patients developed SSI. Oral plus intravenous antibiotics (n = 2426) had a lower SSI rate than intravenous antibiotics alone (n = 3324; 6.3% vs 16.7%, p 11%). Oral antibiotic use was associated with a lower SSI rate for every class of antibiotics used.
The most reliable method to detect small liver metastasis (<1 cm) from colorectal carcinoma is A transabdominal ultrasound. B multidetector CT scan. C fludeoxyglucose-PET scan. D PET-CT scan. E contrast-enhanced MRI.
E
Due to its availability, ease of use, and relatively low cost (compared with MRI), multidetector CT scan (MDCT) is generally the test of choice for screening and staging of liver metastases. These metastases are best detected by dynamic CT scanning during the portovenous phase.
Contrast enhanced MRI is the most sensitive imaging modality to detect small (<1 cm) liver lesions due to its superior soft tissue resolution and the ability to use diffusion-weighted imaging. The sensitivity of contrast-enhanced MRI to detect liver metastases is 90–95% compared with 70–75% for MDCT. However, the increased costs and limitations of MRI (e.g., metal implants, claustrophobia) limit its use as a screening tool. The main role of MRI is in detection and characterization of small lesions. In this respect, MRI outperforms both FDG-PET scan PET-CT scan in the detection of small liver metastases. Transabdominal ultrasound fails to detect more than 50% of liver metastases. Contrast-enhanced ultrasound may improve performance; however, this method is not yet available in the United States.
The lowest rate of pancreatic leak after distal pancreatectomy is achieved with
A preoperative pancreatic stent placement.
B suture ligation of the duct.
C fibrin glue sealant applied to the transected duct.
D stapler transection of the pancreas.
E postoperative somatostatin analogue.
B
The most common postoperative complication of distal pancreatectomy is a leak from the cut edge of the pancreas, occurring in 20–30% of patients. To avoid this leak and subsequent development of a pancreatic fistula, many closure techniques and adjuncts were investigated.
Direct suture ligation is the most reliable method to reduce pancreatic leak after pancreatectomy. Although initially thought to be promising, stapler transection of the pancreas (with or without stapler reinforcement) does not reduce leak rate. Postoperative pancreatic stent placement might be useful treatment for a known leak; however, prophylactic use of this technique does not reduce leak rate. Routine use of fibrin glue sealant, closed suction drains, and postoperative somatostatin analogue do not reliably reduce leak rates after distal pancreatectomy.
The patient shown in figure 1 becomes hypotensive and hypoxic during fascial closure after a splenectomy. Which of the following is the optimal surgical management at this time?
A Vacuum closure of the abdominal wall
B Progressive fascial closure with closure device
C Bridge closure with biologic mesh
D Polytetrafluoroethylene (PTFE) patch closure of the abdominal wall
E Component separation and primary abdominal wall closure
A
Patients requiring temporary abdominal closure have significant morbidity and an in-hospital mortality rate of 33%. Complications of open abdomens include abdominal abscesses, enterocutaneous fistulas, and chronic hernias. Eventual closure techniques may include tissue rearrangement and use of biosynthetic agents.
Temporary closure of an open abdominal wound is commonly accomplished with a vacuum dressing. When a patient requires open abdomen management, negative pressure therapy is becoming the method of choice, especially in trauma patients and for planned re-exploration in other general and vascular surgery patient populations. Temporary negative pressure abdominal wound closure is associated with patient comfort, low complication rates, and low costs. Eventual primary closure of open abdominal wounds after negative pressure treatment exceeds 70%.
Which of the following statements regarding overwhelming postsplenectomy infection (OPSI) is true?
A Patients undergoing emergency splenectomy for injury have worse OPSI outcomes than patients having splenectomy for a hematologic disorder.
B Asplenic patients are at risk for OPSI that is fatal in up to 25% of cases.
C Appropriate initial antibiotic coverage for an asplenic patient with a fever includes empiric treatment with vancomycin and ceftriaxone.
D Splenic implants after splenectomy ensure protection against OPSI.
E OPSI is most commonly caused by a Gram-negative organism.
C
Asplenic or hyposplenic patients have increased risk for infection and death from encapsulated organisms, often pneumococcus, Haemophilus influenza type b, or meningococcus. However, patients who have a splenectomy to treat an underlying hematologic outcome are at greater risk for postsplenectomy infection. Splenic implants are not considered adequate protection against overwhelming postsplenectomy infection (OPSI).OPSI can progress rapidly from a mild flu-like illness to fulminant sepsis that is fatal in up to 70% of cases with delayed or inadequate treatment. Because of the high mortality and fulminant course associated with OPSI, vaccination and antibiotic prophylaxis are the basis of the management of asplenic or hyposplenic patients
Pneumococcal, H. influenzae type b, meningococcal, and influenza virus vaccinations are recommended for asplenic patients, preferably 2 weeks before an elective surgery. A booster dose should be administered every 5 years. Children who receive their dose before age 7 should receive a booster dose 3 years later with subsequent doses every 5 years.
Asplenic patients with a fever should receive empirical antimicrobial therapy. Rapid identification of patients at risk for OPSI and administration of vancomycin and ceftriaxone to cover Streptococcus pneumonia, H. influenzae, Neisseria meningitidis, and many community-acquired Gram-negative bacilli can improve survival. Prophylactic antimicrobial therapy is recommended for asplenic children younger than age 5 and may be considered for older children and adults for 1–2 years after splenectomy. Patients who have had postsplenectomy infection should have lifelong prophylaxis. Asplenic patients should be educated about the risk of life-threatening infection with any illness with fever or severe flu-like symptoms without fever.
A 38-year-old woman presents with right upper quadrant pain, postprandial gas, and bloating associated with nausea. Gallbladder ultrasound shows a distended gallbladder with no stones. Which of the following statements is most accurate regarding cholecystokinin cholescintigraphy?
A It is useful when patients describe recurrent epigastric or right upper quadrant pain episodes lasting 30 minutes or longer.
B It is useful in determining the etiology of atypical abdominal pain.
C Symptom reproduction is the most useful predictor of right upper quadrant symptom relief by cholecystectomy.
D H2 receptor antagonists increase gallbladder contractility and may produce a false-negative test.
E An abnormal gallbladder ejection fraction is highly specific for gallbladder disease.
A
Cholecystokinin (CCK) cholescintigraphy is an appropriate step in diagnosing the patient with typical gallbladder symptoms who has a negative ultrasound evaluation. CCK or a synthetic derivative (sincalide) is injected 30–60 minutes after the administration of Tc-99m labeled disofenin (DISIDA, 2,6-diisopropyl acetanilidoiminodiacetic acid) or mebrofenin (BRIDA, bromo-2, 4,6-trimethyl acetanilidoiminodiacetic acid). Imaging starts after the injection and continues for 60 minutes. When acute cholecystitis is suspected and the gallbladder is not seen within 60 minutes, imaging should be continued for up to 3–4 hours. Imaging for 18–24 hours may be necessary in some cases (e.g., a severely ill patient, severe hepatocellular dysfunction, or suspected common bile duct obstruction). If the patient is being studied for a biliary leak, 2- to 4-hour delayed imaging should be obtained.
The Society of Nuclear Medicine defines an ejection fraction of less than 38% as abnormal. This is a calculated number designated as 2 standard deviations from the mean and there will be “normal” patients who have ejection fractions less than 38% and abnormal patients who have ejection fractions greater than 38%. Thus, specificity of this test for gallbladder disease is low.
The use of CCK cholescintigraphy for diagnosis of biliary tract disease should be limited to those patients who meet ROME III criteria (table 1) for functional gallbladder disorders. It should not be used to evaluate atypical abdominal pain, because many medical conditions, including diabetes, irritable bowel disease, and celiac disease, can produce an abnormal gallbladder ejection fraction.
Several medications may reduce gallbladder contractility and should be withheld before the test. These include atropine, calcium channel blockers, octreotide, progesterone, indomethacin, theophylline, benzodiazepines, H2 antagonists, and all opioids. Although many physicians rely on reproduction of the patient’s symptoms by the CCK infusion to recommend cholecystectomy, this is a very nonspecific finding. Several small retrospective studies recommended removal of the gallbladder for symptoms induced by CCK infusion, but at this time it is not considered a reliable test for the effectiveness of cholecystectomy in curing symptoms. Reproduction of symptoms is quite often the result of rapid infusion of the CCK analog.
A 22-year-old woman develops acute cholecystitis during the 32nd week of pregnancy. Which of the following statements regarding her management is true?
A Premature labor is less common than in the second trimester.
B Intraoperative cholangiography is associated with an adverse fetal outcome.
C Open cholecystectomy is safer than laparoscopic cholecystectomy.
D Postoperative endoscopic retrograde cholangiopancreatography is safe.
E Gallstone pancreatitis is associated with a fetal loss of 20%.
D
The most appropriate time to (semi-) electively operate on a pregnant women with cholecystitis is in the second trimester (13–26 weeks), because there is a lower incidence of premature labor compared with the third trimester. At 32 weeks, conservative management of her cholecystitis should be attempted initially. If an operation is mandated, the mother should be pretreated with betamethasone 24 hours and 12 hours preoperatively. This approach enhances surfactant production and reduces the need for neonatal respiratory support.
At 32 weeks, any radiographs that are clinically needed are safe. This includes intraoperative cholangiography as well as endoscopic retrograde cholangiopancreatography (ERCP). The fetus should be shielded when possible. External fetal monitoring is indicated.
Laparoscopic surgery is as safe or safer than an open procedure in pregnancy. An open cutdown to the peritoneal cavity is indicated (Hasson technique). Gallstone pancreatitis is associated with a fetal loss of 4.7% compared with a fetal loss of 2.8% with non–gallstone pancreatitis. A patient with gallstone pancreatitis should be treated with a cholecystectomy and clearing of stones with operative common duct exploration or postoperative ERCP. Both are associated with lower rates of fetal loss than conservative management.
A 48-year-old woman with known cirrhosis from prior alcohol abuse develops increasing confusion. Her blood ammonia levels are elevated. A diagnosis of hepatic encephalopathy is made secondary to hepatic failure. The best definitive treatment is
A portacaval shunt.
B protein restriction.
C splenorenal shunt.
D transjugular intrahepatic portosystemic shunt.
E liver transplantation.
E
The only definitive treatment for end-stage hepatic failure is liver transplantation. Portacaval shunting, splenorenal shunting, and a transjugular intrahepatic portosystemic shunt (TIPS) will all reduce portal hypertension, which will reduce gastrointestinal bleeding, but they do not treat the underlying liver failure and will exacerbate encephalopathy.
Although protein restriction was previously recommended for hepatic encephalopathy, it is no longer indicated and may be harmful. High-protein diets are well tolerated in patients with cirrhosis; they should receive 1–1.5 g/kg of protein and 25–40 kCal/kg per day. Most patients with cirrhosis have a deficiency of branched chain amino acids. These are commonly found in dairy products and vegetables. Augmentation of the diet with these proteins may be useful. Although there is anecdotal evidence in favor of the use of purgative agents such as nonabsorbable disaccharides (such as lactulose), no comprehensive meta-analysis has shown them to be effective.
A 34-year-old man is diagnosed with a transverse colon cancer in the setting of multiple colon polyps. There is no prior family history of colon cancer. He has a total proctocolectomy with an ileal pouch anal anastomosis. His tumor is a T3N0M0, and more than 100 other polyps were noted throughout his colon and rectum. Two years later, he has a surveillance CT scan, and a 3-cm mesenteric mass is noted. A core needle biopsy shows very bland-appearing spindle cells. Which of the following is the next step in management of this patient?
A Sulindac
B Imatinib
C FOLFOX (5-fluorouracil, leucovorin, and oxaliplatin)
D Radiation
E Surgical excision
A
This patient has clinical evidence of familial adenomatous polyposis (FAP). Patients with classic FAP are at high risk of developing abdominal desmoid tumors, occurring in approximately 10–15% of patients with FAP. Attenuated FAP is a subtype and recognized by fewer polyps, presentation at an older age and less risk for developing desmoid tumors.
Desmoid tumors are considered benign, because they do not metastasize. However, despite having a benign designation, abdominal desmoid tumors can cause significant local problems as they grow into surrounding tissues. Therefore, while considered “benign,” some abdominal desmoid tumors clearly have a “malignant” effect on patient’s lives and can result in death.
The natural history of these tumors is unpredictable, with a small number regressing completely, some waxing and waning, many staying stable over time, and a small number rapidly progressing. The management of these lesions is multidisciplinary. Surgery is reserved for tumors that are symptomatic and resectable, because recurrence is common. Simple observation is reasonable for small, asymptomatic lesions. As part of this strategy, serial CT scans to identify complications, such as ureteral obstruction, is reasonable.
First-line pharmacologic agents include nonsteroidal anti-inflammatory medications (e.g., sulindac) and antiestrogens (e.g., tamoxifen). Cytotoxic agents can be used for advanced disease that is not responding to these other, less-toxic choices. Surgery is often complicated and associated with frequent complications. Therefore, it is done only for significant symptoms. When necessary, resection is the best option if possible. However, many patients will have unresectable disease, so a bypass can be done to relieve a bowel obstruction.
This patient has an incidental finding of a mesenteric mass. While at risk for metastatic colon cancer, the biopsy is most consistent with a desmoid tumor. Resection is not indicated at this time and should not be done. Imatinib is a tyrosine kinase inhibitor used to treat gastrointestinal stromal tumors. FOLFOX (5-fluorouracil, leucovorin, and oxaliplatin) is a combination chemotherapeutic regimen used for the adjuvant treatment of colon cancer. Radiation is not used in the treatment of desmoid tumors, because it appears ineffective and is associated with a high rate of complications. Therefore, sulindac is the best option at this time.
A 47-year-old woman presents with acute onset of abdominal pain. CT shows evidence of acute portal vein thrombosis (PVT). Which of the following is true regarding acute PVT?
A In most patients, only the portal vein is involved.
B A myeloproliferative disorder is rarely associated with PVT.
C Clinically significant ascites is uncommon.
D Recanalization occurs frequently without anticoagulation.
E Common initial presentation is gastrointestinal bleeding from varices.
C
Acute portal vein thrombosis often presents with onset of vague abdominal pain. Imaging studies such as CT will typically define the significance of the abnormality. In most patients, the portal vein and its tributaries and branches are involved. One study found that more than 50% of the patients had a prothrombotic state, and 35% had a myeloproliferative disorder as a precipitating factor in the development of portal vein thrombosis.
Imaging often shows ascites, but it is rarely clinically significant. Gastroesophageal varices develop in up to 50% of patients without recanalization, although this is rarely an initial presentation. Studies suggest that gastroesophageal varices may develop as early as a month after thrombosis but more commonly develop later over time. Early anticoagulation is associated with recanalization in 40% of patients studied, but no patients recanalized when anticoagulation was not used. Another study found similar findings in patients who underwent early anticoagulation that continued for a mean of 8 months; 39% patients developed recanalization. Early anticoagulation of patients with acute portal vein thrombosis leads to a better outcome with a reduced risk of gastrointestinal complications.
A 57-year-old woman presents for acute symptoms of right upper quadrant pain. The CT scan obtained is shown (large simple appearing cyst in L lobe of liver). Which is the following is the most appropriate treatment?
A Estrogen therapy B Somatostatin C Sclerotherapy D Fenestration E Hepatic resection
D
This patient presents with acute symptoms related to her polycystic liver disease. Indications for intervention include acute pain from potential bleeding into the cyst, vague mass effect from the size of the cyst, and, rarely, liver dysfunction. Fenestration is most effective, can be done laparoscopically, and often is performed as an outpatient procedure. There is minimal risk of bleeding or postoperative complications with this approach.
Hepatic resection is not necessary in the management of this disease. Sclerotherapy is ineffective with a high rate of recurrence. Observational studies suggest that pregnancy and estrogen replacement therapy can increase the number and volume of cysts. Somatostatin analogues may reduce the overall volume of the liver but have no effect on the size of the cysts.
A 35-year-old man develops acute necrotizing pancreatitis related to a drug reaction. The CT scan shown in figure 1 and figure 2 (large RP collection in R sided of abdo, small bubble of air) was obtained 1 month after the acute process. Although he is afebrile, he is having trouble eating, has decreased stamina, and has moderate discomfort to palpation of the right lower quadrant. His white blood cell count is mildly elevated. Which of the following is the most appropriate management in this patient?
A Expectant management for continued resolution B Cyst gastrostomy C Percutaneous aspiration D Transabdominal debridement E Retroperitoneal debridement
E
The management of acute necrotizing pancreatitis continues to evolve. A multitude of different modalities are now available that affect the timing and management approach for patients with walled-off pancreatic necrosis (WOPN). This patient continues to be symptomatic a month after his acute process. The CT scan suggests a small fleck of air in one of the pockets of the WOPN
Expectant management of this patient may continue to work, although his recovery is likely to be slow. The gas within the fluid collection raises concern for bacterial contamination and infection. There is not an obvious endoscopic site for transluminal drainage of this fluid collection, and the multiple loculated areas would make this less likely to work effectively. Percutaneous aspiration is a diagnostic test to exclude an infected fluid collection. Percutaneous drainage is possible and may expedite resolution of the patient’s symptoms, but often the material cannot be completely evacuated. Laparoscopic debridement runs the risk of further seeding infected fluid throughout the peritoneal cavity, which would be less than ideal.
Direct percutaneous endoscopic debridement, also referred to as direct retroperitoneal debridement, offers an excellent option for management of this process. The procedure uses laparoscopic instruments passed in a retroperitoneal plane into the cavity for debridement and irrigation or direct percutaneous endoscopic debridement. This procedure is usually facilitated by first passing image-guided catheters into the cavity and using these catheter tracks to guide the laparoscopic instruments. Endoscopic debridement was used in this patient with prompt resolution of the symptoms and no need for subsequent operative intervention.
Which of the following is a component of both the Child-Pugh and Model for End-Stage Liver Disease scoring systems? A Albumin B Aspartate transaminase C Creatinine D Encephalopathy E International normalized ratio
E
The Child-Turcotte-Pugh (CTP) score was originally developed to evaluate the risk of portocaval shunting procedures in patients with portal hypertension. It is also used to evaluate surgical risk in other intra-abdominal procedures performed in cirrhotic patients. Overall surgical mortality is 10% for patients with class A cirrhosis, 30% for those with class B cirrhosis, and 75–80% for those with class C cirrhosis. Components of the CTP score are international normalized ratio (INR), bilirubin level, albumin level, encephalopathy, and ascites (table 1). The CTP score is criticized for its 2 subjective variables (degree of encephalopathy and ascites), narrow range (5–15), and the equal weight given to each variable.
The Model for End-Stage Liver Disease (MELD) is a linear regression model based on 3 laboratory values (INR, bilirubin, and creatinine level). It was initially developed as a model to predict mortality after transjugular intrahepatic portosystemic shunt but has now become the method used for liver transplant allocation in the United States. The MELD formula is shown in figure 1.
MELD is an excellent predictor of postoperative mortality. One study demonstrated an increase in mortality of 1% for each MELD point, up to a score of 20, and by 2% for each MELD point above 20. Cirrhotic patients undergoing urgent surgery or major operations have a relative risk of mortality increase of 14% for each 1-point score in their MELD score (figure 2). Both the CTP and MELD scoring systems use INR and total bilirubin in their calculation.
A 74-year-old woman presents with 4 days of nausea, feculent vomiting, and medial thigh pain. Her last bowel movement was 2 days ago. She has no history of abdominal surgery. Her abdomen is distended and mildly tender. Her white blood cell count is 14,000/mm3 (3600–11,200/mm3). A CT scan is shown in figure 1 (Shows an obturator hernia). Which of the following is the most appropriate next step?
A Nasogastric tube, intravenous fluids, and observation B Preperitoneal groin exploration C Femoral groin exploration D Inguinal groin exploration E Midline laparotomy
E
This patient presents with a small bowel obstruction secondary to an obturator hernia. An obturator hernia is a rare pelvic hernia with an incidence of approximately 1%. A weakening of the obturator membrane can result in an enlargement of the obturator canal and herniation of small bowel contents adjacent to the obturator vessels and nerve, leading to an obstruction. Obturator hernias are most commonly seen in elderly women with prior weight loss and no history of abdominal surgery. Patients can present with signs of a small bowel obstruction: nausea, vomiting, obstipation, and abdominal pain. Thigh or knee pain on the affected side are commonly seen. The Howship-Romberg sign refers to ipsilateral groin pain radiating down the thigh caused by irritation of the obturator nerve. The referred pain is relieved by flexion of the thigh and aggravated by extension, abduction, and medial rotation.
This patient presents with signs of compromised bowel, tenderness on exam, and an elevated white blood cell count. Therefore, observation would not be appropriate management. Surgery through a midline incision is preferred for an obturator hernia, because it gives the best exposure, allows reduction of hernia content, and facilitates bowel resection if necessary. Resection of the involved portion of the bowel is sometimes required because of gangrenous changes or perforation. The defect in the membrane can usually be closed with interrupted nonabsorbable sutures. On occasion, a mesh repair is required.
An obturator hernia cannot be repaired through a femoral or inguinal groin exploration. In this patient with signs of compromised bowel, a preperitoneal approach is not recommended.
A 56-year-old woman had a robotic-assisted thoracoscopic left lower lobectomy 12 weeks ago for lung cancer complicated by a diaphragmatic injury, which was repaired. She presents with intractable nausea, vomiting, and abdominal pain for 2 days. She is tachypneic, tachycardic, and ill appearing. Her white blood cell count is 23,700/mm3 (3600–11,200/mm3). A CT scan of the chest is shown in figure 1 (diaphragmatic hernia with small bowel in thorax). Which of the following is the most appropriate next step?
A Left chest tube B Upper endoscopy C Laparotomy D Left anterolateral thoracotomy E Left posterolateral thoracotomy
C
The CT scan of the chest reveals a left diaphragmatic hernia most likely through the previous site of injury. The patient presents with symptoms of a bowel obstruction, abnormal vital signs, and an elevated white blood cell count. The suspicion for strangulated bowel is high.
The patient will need an exploration to reduce the bowel, a resection if the bowel is indeed gangrenous, and repair of the diaphragm. This can best be accomplished through an abdominal incision using a midline exploratory laparotomy.
The thoracotomy incision, whether anterolateral or posterolateral, would make abdominal exploration of the bowel with possible resection quite challenging.
There is no role for placement of a chest tube or upper endoscopy in this patient, who presents with a strangulated diaphragmatic hernia.
An 82-year-old man presents with a 7-day history of right upper quadrant pain with nausea and vomiting. His medical history is remarkable for hepatitis C and cirrhosis. His white blood cell count is 18,500/mm3 (3600–11,200/mm3) with 13% bands. Total bilirubin is 5.5 mg/dL (0.2–1.9 mg/dL), and alkaline phosphatase is 325 units/L (<95 IU/L). CT scan of the abdomen and pelvis demonstrates severe cholecystitis with pericholecystic fluid, cholelithiasis, and a normal common bile duct. The most appropriate next step should be
A magnetic resonance cholangiopancreatography.
B hepatobiliary iminodiacetic acid scan.
C endoscopic retrograde cholangiopancreatography.
D percutaneous cholecystostomy.
E open cholecystectomy.
D
Acute calculous cholecystitis is a common surgical emergency, often affecting elderly patients and those with serious medical comorbidities. Emergency cholecystectomy in this high-risk group of patients is associated with increased morbidity up to 41% and with mortality up to 4.5%. An alternative treatment is image-guided percutaneous cholecystostomy under local anesthetic, which has less morbidity and mortality than open or even laparoscopic cholecystectomy in the emergency setting. Once the acute episode is resolved, cholecystectomy may or may not be considered as an elective intervention.
Which of the following statements regarding colorectal liver metastases is true?
A Colorectal metastases represent the third most common malignant tumor of the liver.
B Resectability is based on the volume of liver remaining after resection.
C Four or more hepatic metastases are a contraindication to liver resection.
D Radiofrequency ablation is appropriate for metastatic tumors larger than 5 cm.
E Neoadjuvant chemotherapy yields best results when used for single colorectal metastases.
B
Liver metastases from colorectal cancer are the most frequent hepatic malignancies in the United States. Radiofrequency ablation (RFA) is a local therapy that uses a form of alternating electrical current to achieve thermal destruction. RFA is performed using several techniques: percutaneous, open, or laparoscopic. Tumor size and location can preclude effective RFA when used as curative treatment. Tumor sizes larger than 4–5 cm are associated with an increased incidence of recurrence.
Traditionally, the presence of 4 or more hepatic metastases was a contraindication to hepatectomy. Current data reveal that hepatectomy for 4 or more metastases is associated with an approximate 5-year survival rate of 33%. Resectability is based on volume of liver remaining after resection and not the actual number of tumors. In a patient with normal liver function, a 20% remnant is recommended. In a patient who has undergone neoadjuvant chemotherapy, a 33% remnant is recommended.
Neoadjuvant chemotherapy can yield good results when used for multiple colorectal metastases. Chemotherapy and complete resection of hepatic metastases are associated with long-term survival in up to 50–60% of patients.
An 80-year-old woman is admitted to the medical intensive care unit with pneumonia and sepsis. She is intubated and on norepinephrine and vasopressin for 5 days without improvement. Current therapy includes piperacillin/tazobactam and vancomycin. Her liver function tests are elevated, with a total bilirubin of 1.8 mg/dL (0.2–1.9 mg/dL), aspartate aminotransferase of 60 U/L (women 8–40 IU/L), alanine aminotransferase of 50 U/L (7–55 IU/L), and alkaline phosphatase of 100 U/L (<95 IU/L). An ultrasound of her right upper quadrant demonstrates an enlarged gallbladder with pericholecystic fluid and wall thickening, but no gallstones. The common bile duct measures 4 mm. Which of the following is the most appropriate next step in the management of this patient?
A Open cholecystectomy
B Percutaneous cholecystostomy tube placement
C Endoscopic retrograde cholangiopancreatography
D Observation
E Magnetic resonance cholangiopancreatography
B
This patient has acalculous cholecystitis, and no further imaging is necessary to confirm the diagnosis. Acalculous cholecystitis is typically associated with critical illness, and most of these patients are poor surgical candidates. Mortality associated with this disease is 41%. Acalculous cholecystitis is generally the result of biliary stasis and gallbladder ischemia, although the pathophysiology is incompletely understood. Stasis is frequently due to prolonged fasting or hyperalimentation. Progression of disease with gallbladder complications, such as perforation, gangrenous cholecystitis, or emphysematous cholecystitis, occurs in 40–100% of these patients; therefore, observation alone is not recommended.
Magnetic resonance cholangiopancreatography or endoscopic retrograde cholangiopancreatography would not benefit this patient, because the common bile duct is normal in caliber and gallbladder inflammation and ischemia is the source of her sepsis. Percutaneous cholecystostomy is recommended as definitive therapy or as a bridge to ultimate cholecystectomy when the patient is medically stable. In a hemodynamically normal patient, most authors recommend a laparoscopic approach initially rather than an open approach given the proven safety and decreased hospital length of stay of the former.
Which of the following statements is true about inguinal hernia?
A Male patients without signs of indirect inguinal hernia have an obliterated processes vaginalis.
B Direct hernias develop lateral to the inferior epigastric vessels.
C The most common nerve injured during inguinal hernia repair is the genitofemoral.
D Femoral hernias are the most common hernia in female patients.
E Chronic pain is a long-term complication of hernia repair.
E
Indirect hernias are associated with a patent processus vaginalis. Patients with a patent processus vaginalis have a lifetime incidence of inguinal hernia of 5%. Direct hernias occur in an anatomic area called Hesselbach triangle, which is bordered by the inguinal ligament inferiorly, the inferior epigastric vessels laterally, and the lateral edge of the rectus medially. Femoral hernias occur inferior to the inguinal ligament and medial to the inferior epigastric vessels and are more common in women than men. The most commonly occurring hernia in female patients, however, is an indirect inguinal hernia.
During inguinal hernia repair, the most commonly injured nerve is the ilioinguinal. This nerve is close to the external inguinal ring and is a sensory nerve to the pubic region and the upper portion of the scrotum or labia majora. The genital branch of the genitofemoral passes with the spermatic cord to the scrotum, and the lateral branch forms the lateral femoral cutaneous nerve and provides sensation to the lateral portion of the anterior thigh.
Although most series report chronic pain limiting quality of life as occurring in only 5% of postoperative hernia repair patients, up to 30% of patients report some long-term pain or discomfort when asked on a confidential questionnaire. Chronic pain may persist for several years and is the most serious long-term complication.
A 32-year-old woman presents with a moderately symptomatic, reducible, left groin hernia. Which of the following statements is true regarding repair of groin hernias in women?
A The incidence of femoral hernias is the same regardless of sex.
B A traditional, open, tension-free, mesh repair (Lichtenstein) addresses both inguinal and femoral hernias.
C The increased incidence of femoral hernias may explain why female sex is a risk factor for hernia recurrence.
D Laparoscopic extraperitoneal herniorrhaphy is contraindicated in female patients
E A preoperative diagnosis of femoral hernia precludes the laparoscopic approach.
C
Inguinal herniorrhaphy is one of the most commonly performed surgical procedures today. Incidence of “groin hernias” is substantially higher in male patients (2–5% vs 0.3% in women). Although femoral hernias comprise only a small minority of groin hernias in men, they represent nearly one-third of the groin hernias in women. After open mesh repair, a “recurrence” in women is often seen in the femoral canal. The most likely explanation is that these are not recurrences but rather errors in diagnosis leading to the incorrect operation.
A traditional, open, tension-free, mesh repair (Lichtenstein) will address both the direct and indirect defects but does not cover the femoral canal. Both McVay and open preperitoneal repairs will address the femoral canal, but these are less commonly performed than open Lichtenstein procedures. A laparoscopic, extraperitoneal herniorrhaphy allows for exploration and coverage of the entire myopectineal orifice, addressing both inguinal and femoral hernias. In women, laparoscopic preperitoneal repair is associated with a lower recurrence rate than open mesh repair.
A 75-year-old man presents with new onset left groin bulge. He reports that he noticed the bulge 2 months ago and experienced no pain or other symptoms. He has normal bowel habits and no previous hernia surgery. He is otherwise healthy, except for hypertension. He lives alone and works as a hospital volunteer 3 days a week. In this patient, watchful waiting is
A contraindicated due to the acute nature of the hernia.
B contraindicated due to the presence of a bulge.
C contraindicated due to the age of the patient.
D at high risk of bowel obstruction.
E likely to fail due to progression of symptoms.
E
Management of asymptomatic or minimally symptomatic hernias is evolving as long-term randomized controlled study data for watchful waiting are reported. Since their original report in 2004 demonstrating safety of watchful waiting during short-term follow-up (median 3.2 years), Fitzgibbons et al. followed patients up to 11.5 years and found that the overall crossover rate to surgery is 68%. Men older than 65 are significantly more likely to crossover to surgery than are younger men (79% vs 62%). The most common reason for surgery was pain (54%). Only 2.4% of watchful waiting patients required emergency operation for complications such as strangulation or obstruction.
The authors concluded that watchful waiting is therefore safe, regardless of the timing of onset of the hernia, presence of a bulge, or age of the patient, but most patients will experience a progression of symptoms and ultimately need surgical repair. A similar study in the United Kingdom found that 72% of the watchful waiting cohort came to operative therapy at 7.5 years of follow-up.
Which of the following statements is true regarding nonparasitic splenic cysts?
A Management should consist of splenectomy for cysts greater than 5 cm in diameter.
B Percutaneous needle aspiration is often definitive therapy.
C Symptom relief after percutaneous needle aspiration may predict response to operative management.
D Most nonparasitic splenic cysts are symptomatic.
E There is a high incidence of spontaneous or posttraumatic rupture of splenic cysts.
C
In the United States, nonparasitic splenic cysts may be posttraumatic or primary (figure 1). Rupture of splenic cysts either spontaneously or posttraumatically is rare. Studies suggest that fewer than 60% of posttraumatic cysts are symptomatic.
Few data support the use of cyst size as an indication for splenectomies in asymptomatic patients. Postaspiration cyst and symptom recurrence are very common; however, symptomatic relief with aspiration can suggest a benefit of operative management of splenic cysts. Because of the benign nature of the majority of splenic cysts, partial splenectomies or partial cystectomies (unroofing) are reported in most patients.
26 y.o. patient with 6 wk hx of enlarging, painless mass in distal L thigh. MRI shows 7 x 6 x 5 cm mass in gracilis muscle, near femoral neurovasc bundle. Bx shows low grade sarcoma. CT Chest shows a few calcified, sub centimeter pulmonary nodules, This patient should have which of the following?
A. Radiation B. Pre Op Chemo C. Post Op Chemo D. Amputation E. Lung Bx
A
These lesions are too small for bx and may be granulomas. Monitor lung findings closely with follow up CTs
For high or low grade soft tissue sarcomas of the extremities, combined modality local therapy of surgery with radiation is standard of care. Radiation improves both local recurrence and relapse free survival compared with surgery alone. Particularly true when resection margins are close (<1 cm) b/c they are likely to be based on proximity of tumor to the femoral neurovascular bundle. Local control and survival with limb sparing surgery is similar whether radiation is given preop or postop. Pre op radiation allows for smaller, more precise fields, but this may come at the trade off of more wound complications.
35F presents with substernal CP 5 days after lap chole. She is 5 wks postpartum. A PE student is obtained. An incidental posterior mediastinal mass is noted. Which is the most likely dx?
A. Thymoma B. Ganglioneuroma C. Neuroblastoma D. Schwanomma E. Teratoma
D
Predicted by location
Anterior mediastinal masses–thymic neoplasms, germ cell tumors, lymphoma and teratoma as well as thyroid and parathyroid pathology
Middle mediastinal masses–lymphomas, granulomas, and cystic processes such as congenital aerodigestive tract cysts of pericardial cysts. ~30% malignant
Posterior mediastinal masses–neurogenic, esophageal masses and neurenteric cytsts, with relatively few of these being malignant. Nerve sheath tumors compreise 40-65% of neurogenic tumors in chest, and ~75% are schwanommas. Typically firm and encapsulated. On CT they are round and well circumscribed. Low malignant potential. Observation is appropriate unless high risk features including symptoms, >5 cm, hx of radiation in field or evidence of invasion on CT or MRI. Neuroblastoma is much more common in childhood and would be rare in this patient
74F presents with acute pancreatitis. US shows no stones. ERCP is performed to evaluate the cause of her pancreatitis. This shows a fish mouth pancreatic duct opening with mucinous fluid emanating from it. Which of the following dx is most likely assoc with this presentation?
A. Pancreatic adenoca B. Mucinous cystadenoma C. IPMN D. Choledocholithiasis E. Distal CBD cancer
C
Up to 5% of pancreatic tumors in referral centers
Symptoms of acute, chronic or relapsing pancreatitis
Classic view on ERCP is fish mouth pancreatic duct opening with mucinous fluid emanating from it.
Pancreatic adenoca would not present with these ERCP findings. S &S are non specific but can include new abdo/back pain, wt loss (often significant), steatorrhea, and loss of appetite. Pain can be sign of locally advanced disease. Painless jaundice is the classic presentation for lesions in the head of the pancreas. 4th leading cause of cancer death in US. Only 15-20% of patients dx with panc adenoca are candidate for panc resection. CT for diagnosis and preop staging. In addition to determining primary tumor size, CT is used to evaluate invasion into local structures and for evidence of mets
Cystadenoma can cause pacnreatitis but these are more commonly found incidentally when imaging is done for abdo pain. These cysts are typically found in the 5th-6th decade and they are located in the body or tail of the pancreas.
Which of the following is characteristic of IPMN?
A. It is often multifocal
B. Clinical course is benign
C. Branch duct IPMNs are less likely malignant than main duct
D. Most patients present with jaundice
E. Enucleation is adequate for main duct IPMN
C
Three main types of IPMN– Main branch, mixed and branch type. Main duct has far greater malignant potential than branch duct. In diffuse main duct, total pancreatectomy is required for cure. When resection is done for branch type, local recurrence is 8%. Further, after resection of branch type, development elsewhere in N pancreas is uncommon. Although diffuse involvement can be seen, multifocal presentation is uncommon.
If left unresected, IPMNs follow a dysplasia carcinoma sequence undergoing malignant transformation and invasion, which ultimately disseminate and spread.
WU includes MRCP, EUS with aspiration of fluid and assessment of specific duct types involved. Observation may be appropriate for in patients without consensus indication for resection. Consensus indication include main duct or mixed type IPMN, b/c they have a risk of malignancy approaching 70%. Further branch type IPMN with the presence of symp, mural nodules or cysts >3 cm
If abN appear focally limited, local resection with pancreatic head or tail resection can be considered. Frozen section should be done of the margin of resection. If there is carcinoma or carcinoma in situ, additional resection or completion pancreatectomy should ensue. If resection margins show only adenomatous changes or less, observation may ensue.
54M presents with epigastric discomfort. His CT scan is neg except for the finding of a mass in greater curve. Upper endoscopy shows a submucosal gastric mass, the bx shows spindle cells, and staining is positive for c-kit. Which of the following is the most appropriate therapy for this lesion?
A. Imatinib for 3 mos followed by resection if there is no progression
B. Radiation therapy followed by imatinib for 3 mos
C. Subtotal gastrectomy with D1 lymphadenectomy
D. Partial gastrectomy with D1 lymphadenectomy
E. Laparoscopic wedge resection with grossly neg margins
E
Arise from interstitial cells of Cajal, the intestinal pacemaker cells. Most common connective tissue tumors affecting GI tract. Most commonly located in stomach and proximal small bowel. Standard of care is complete surgical resection, either open or laparoscopic. No evidence that large organ resections are of benefit. Avoid rupture and tumor spillage b/c this increases the risk for recurrence. Lymphadenectomy not indicated b/c GISTS rarely met to locoregional nodes. Spread is hematogenous and most common site is the liver. In addition, peritoneum is a common site of locoregional mets, presumably due to drop mets. Resection remains mainstay and initial choice of treatment for GISTs that cab be completely removed
Preop imatinib not indicated for resectable disease but may be useful in marginally resectable patient in an effort to make them resectable or ensure margin neg resection. Risk of recurrence and need for adjuvant therapy is estimated by the size of the tumor, its mitotic indexx, and the site of origin of the tumor. Radiation is not indicated in patients with resectable disease or in other patients b/c there is no evidnece that it is radiosensitive. Standard cytotoxic chemo is inefffective.
Which of the following is the most common primary malignant tumor found within a Meckel’s diverticulum?
A. GIST B. Carcinoid C. Adenoca D. Pancreatoblastoma E. Lymphoma
B
Meckels is a congenital remnant of omphalomesenteric duct. 1-2%. Most common anomaly of GI tract. Incidence of malignancy in asymptomatic Meckel’s is 0.5% and 3% in symptomatic. Most common primary malignant tumor found within a Meckel’s diverticulum is carcinoid (76.5%). Remaining types include adenoca (11.4%), GIST (10.8%), and lymphoma (1.3%). Although heterotopic gastric or pancreatic tissue may be found in a Meckel’s diverticulum, pancreatoblastoma is not reported
Surgical management of asymp Meckel’s is controversial. Factors assoc with symp included age <50, male, diverticulum length > 2 cm, and palpable ectopic tissue or abN features of diverticulum. Most common site of ileal malignancy is a Meckel diverticulum and survival is related to stage of disease. Overall incidence rate of Meckel diverticular malignancy is low, an incidentally discovered Meckel diverticulum represents a high risk anatomic region whose risk increases with age. Therefore a Meckel diverticulum discovered intraop should be resected as a method of absolute cancer risk reduction give the high possibility of curative resection and negligible operative mortality, especially in younger pts.
Compared with emergency open operative procedures for obstructing rectosigmoid carcinoma, initial endoscopic stenting of the obstructing lesion followed by elective laparoscopic resection (“bridge to surgery”) is assoc with which of the following?
A. Higher complication rates B. Delay in chemo administration C. Equivalent rate of primary anastomosis D. Higher intraop blood loss E. Greater number of LNs harvested
E
Stenting used as definitive palliative procedure or as a bridge to surgery. Technical success with stenting was 96.2% with a perforation rate of 4.5% and reobstruction rate of 12%. Shorter hospital stays with stent placement than emergency surgery. Primary anastomosis rates were 2x greater with stenting than emergency operation. Complications were higher in emergency surgery group than stent group. Stenting followed by lap resection of obstructing colorectal ca demonstrated lower blood loss, lower post op analgesia requirements, lower surgical site infection rate and lower rate of anast leak after primary anast. More LNs were present in the specimens in pts treated with stenting followed by elective lap resection compared to open surgery.
Stenting does not delay initiation of chemo. However when stenting alone is used as definitive palliation in patients also receiving high dose chemo, there may be an increased risk of delayed perf secondary to tumor shrinkage
In addition to tumor thickness and ulceration, the most powerful predictor of overall survival outcome in patients with pathologic stage I melanoma is
A. Clark level B. Mitotic rate C. Age <60 D. Lower extremity primary site E. Female
B
In patients with localized melanoma, tumor thickness, mitotic rate, and ulceration were the most significant prognostic factors.
Assoc betwee patient age, tumor thickness and ulceration: with incr age, tumor thickness and ulceration incr. In addition, proportion of male patients with melanoma incr with age. However, age by itself, is not as powerful of predictor of survival as is tumor thickness, ulceration and mitotic rate.
Anatomic sites for melanoma assoc with least favorable prognosis include the scalp and neck. Women have better overall survival rates than do men with melanoma
55M post sigmoid colectomy for stage II B colon cancer develops a 4.5 cm solitary L sided liver met 10 months after his initial resection. PET is neg for any other disease. Colonoscopy is neg for recurrence or metachronous cancer. His CEA is 150. This represents a 10x increase over the last 6 months. Which of this patient’s characteristics is an independent predictor of poor prognosis with liver resection of the met?
A. Disease free interval B. Current CEA C. Size of liver met D. Number of liver mets E. Nodal status of primary lesion
A
In up to 1/4 of patients undergoing resection of their primary, the liver is the only site of initial recurrence. Median survival in untx pts with hep mets is < 1 yr, 5 yr survival can approach 25% after resection in select pts.
FONG criteria >Five cm 200 Extrahepatic disease Postive margin
39 y.o. fair skinned man presents to clinic with a flat, plaque like 2.1 cm lesion on his R cheek. Bx reveals elongating strands of basal cell carcinoma infiltrating the dermis. Appropriate tx of this lesion would be
A. Topical imiquimod B. Curettage and electrodissection C. Mohs micrographic surgery D. Wide local excision E. Radiotherapy
C
BCC is most common form of skin cancer, predominantly in men > 40 y.o. Arises most often in sun expsoed skin (eg. face) in susceptible populations (i.e. fair skinned, blonde hair, blue eyed people) living in regions of high sun exposure. Slow growing tumor that rarely mets but its insidious nature can lead to extensive regional infiltration and destruction. Many variants exist. 2 main classifications: circumscribed and diffuse. Nodular BCC, variant of circumscribed, is the most common. Presents as dome-shaped, pearly lesion with distinct borders. Morpheaform, a diffuse form, is particularly aggressive and can be a tx challenge. Presents as whitish, plaque like lesion with poorly defined margins. On path, it has elongating strands of BCC infiltrating the dermis
Tx tailored to type of lesion and its location. Small lesions, curettage and electrodisssection in the office with 2-3 mm margins often renders excellent results. It cannot be used for morpheaform BCC or other deeply infiltrating lesions. Surgical excision with or without reconstruction is indicated for large tumors, invasion of tumor into surrounding structures, and aggressive histologic types (e.g. morpheaform, infiltrative, and basosquamous BCC)> Margins should be 5-10 mm. With BCC of favorable histology located in areas of aesthetic concern, Mohs sx can be performed. Form of minimal resection with immediate pathologic frozen confirmation of neg margins that minimizes the defect created. Radiotherapy is an option in tumors that cannot be excised or in tumors that have incomplete excision. Should not be used in patients < 40 y.o. except in unusual circumstances. Topical therapy with imiquimod is an option in patients with superficial or nodular BCC, but it requires long term administration and excellent patient compliance
50F undergoes endoscopic bx of gastric nodule in the antrum. Pathology reveals low grade mucosa assoc lymphoid tissue (MALT) lymphoma. EUS reveals only submucosal invasion. CT of C/A/P are neg for other sites of disease. Initial therapy of this patient should include:
A. Clarithromycin, amoxicillin, and omeprazole
B. Cyclophosphamide, vincristine and prednisone
C. External beam radiation therapy
D. Wedge excision
E. Antrectomy
A
Rare form of gastric neoplasm (4% of all gastric cancers), it remains the most common form of extranodal lymphoma (up to 20% of all such malignancies). N stomach is typically devoid of lymph cells; chronic inflamm from H pyloir infection, however, leads to acquisition of mucosa assoc lymphoid tissue (MALT) within the stomach, with subsequent degeneration into initial low grade then subsequent high grade lymphoma. Typically B cell tumors. Dx is made by endoscopic bx of lesions. WU include EUS, CT CAP and bone marrow bx to r/o extragastric disease or secondary gastric involvement.
Low grade MALT lymphoma isolated to the stomach is tx through eradication of gastric H pyloric infection. With it, the lymphoma is often cured. Therapy must combine acid suppression with antibiotics. PPIs, H2 receptor blockers and bismuth slats are used to help control gastric acid. Administration of 2 Abx is more effective than 1 alone. Most common Abx used include clarithryomycin, amoxicillin, metronidazole, or tinidazole. In patients with persistent MALT lymphoma after successful eradication of H pylori, radiation is often used. High grade primary gastric lymphomas are tx with chemorads.
Surgery tx of primary gastric lymphoma, once the mainstay, is now reserved for tx of complications such as perforation, bleeding or obstruction. If undertaken in patients with disease limited to stomach and regional LNs, a radical subtotal gastrectomy with D2 dissection can be performed. Palliative gastrectomy is also an option in patients with extragastric disease. Wedge excisions are not performed.
In selecting patients with peritoneal carcinomatosis for surgical peritonectomy and intra-abdominal chemo, which of the following is true?
A. Mucinous subtype of adenoca of the colon PC is a favorable prognostic factor
B. Optimal debulking is assoc with improved survival
C. Solid organ mets are a contraindication to peritonectomy
D. Survival benefits are similar for patients with primary appendiceal PC compared with patients with colon PC
E. Pseudomyxoma peritonei is synonymous with disseminated peritoneal adenomucinosis
C.. although not true these days. HIPEC is done with resection of CRC liver met
GI peritoneal carcinomatosis is rare. Natural hx is variable and dependent on subtype. Role of peritonectomy and peritoneal chemo remains unclear, and benefits cannot be generalized across all diagnoses. Therapy can be morbid and the palliation and survival advantages are obscured by non standard diagnostic definitions, tx protocols and lack of prospective studied. Presence of solid organ mets is a contraindication to surgical mgmt of peritoneum, b/c this presentation defines a malignant and aggressive natural hx. Std definition for optimal debulking is elusive and not assoc with improved survival
Pseudomyoxma peritonei should be classified into at least 2 general subtypes: disseminated peritoneal adenomucinosis (DPAM) and peritoneal mucinous carcinomatosis (PMCA). These 2 descriptions represent a spectrum and there is room for intermediate tumor types.
Appendiceal primary tumors tend to fall into a more indolent natural hx than colon primaries. Median survival for appendiceal PC can excess 6 yrs. Dx of DPAM is best used to describe these lesions. DPAM is characterized histologically by abundant extracellular mucin containing scant simple to focally proliferative mucinous epithelium with little cytologic atypia or mitotic activity and is usually associated with appendiceal mucious adenoma.
Colon primaries tend to be more sinister, with a median survival of less than 2 years. PMCA is the more typical histological subtype here. PMCA is composed of more abundant mucinous epithelium with the architectural and cytologic features of carcinoma
45 y.o. M with 3 metachronous R lobe liver mets of colon cancer is tx with systemic chemo for 3 cycles and has a complete response on PET and CT. Which of the following statements is true?
A. Complete response on PET predicts complete pathologic response
B. Chemo liver toxicity will significantly incr the risk of post op liver failure
C. Resection before administration of chemo would have maximized his chances of cure
D. Resection is still indicated
E. Response to chemo does not alter this patient’s prognosis
D
Effective chemo regimens have led to improvements in medial survival of patients with stage IV colon ca (approx 12 months 20 yrs ago to 24 months now).
For patients with mets confined to liver, these chemo regimens used in the neoadj setting have incr the number of patients with borderline disease who may benefit from liver resection. First line multiagent therapy FOLFOX is assoc with an expected response rate of 70%. Response to therapy is an important stratification tool for patients with advanced but potentially resectable disease. Pts who fail to respond to chemo have poor prognosis. Multiagent chemo is not assoc with incr morbidity after resection
PET are metabolic studies and depend on avidity of tumor cells to concentrate the glucose agent. Tumors can be viable yet not sufficiently metabolically active to show on PET. Complete response on PET is not indicative of complete tumor resolution. CT is highly sensitive, even for small tumors. Some patients (<10%) demonstrate a complete radiographic response–no detectable disease on PET or CT/MRI
Several small retrospective studies of patients with a radiographic complete response show fewer than 15% achieve a complete pathological response. Therefore resection is still recommended, even for pts with a complete radiographic response. The resection should use anatomic landmarks from the tumor pretreatment studies to guide resection with a goal of obtaining an R0 outcome.
70 y.o. F presents deeply jaundiced with central abdo and back pain caused by a pancreatic head mass. Stage IV adenoca of the pancreas is dx after a percutaneous bx of a liver met. She has no other symptoms. Optimal management would include which of the following?
A. MRI to determine whether the liver lesion can be resected at the time of Whipple
B. PET scan to monitor response to neoadjuvant chemo
C. Surgical double bypass (gastrojej and hepaticojej)
D. Endoscopic metal stent and celiac nerve block
E. Percutaneous transhepatic biliary catheter placement
D
60M with Child A cirrhosis and hx of EtOH abuse is found to have 7 cm vascular mass. His AFP is 500. Which of the following is assoc with the greatest survival advanage?
A. Sorafenib B. TACE C. Cytotoxic Chemo D. RFA E. Surgical resection
E
Liver directed therapies in patients with cirrhosis are often limited by degree of underlying liver disease. Resection is gold standard for pts with preserved liver function. Patient has stage II HCC lesion amenable to resection of seg 6 and 7, which represent 25% of total liver volume
Liver transplant would be considered for this clinical stage II HCC patient but the tumor size and assoc cirrhosis are relative contraindication. ETOH use would be relevant as well and abstinence < 6 months is a contraindication to transplant.
Other liver directed therapies should be considered but none can be offered with curative intent. RFA is unreliable in tumors >3 cm with an expected locatl tx failure rate >30% for 5 cm tumors. Role for TACE would be to tx a locally advanced, unresectable tumor and manage lesions with assoc pain or rupture. Sorafenib is a tyrosine kinase inhibitor that modestly improves survival in patients with advanced liver disease and multifocal HCC. Cytotoxic chemos are of limited benefit in HCC
Incidentally identified gallbladder adenoca is found after lap chole. Completion extended chole (liver resection and portal lymphadenectomy) is indicated for which of the following findings?
A. Tumor confined to mucosa B. Celiac lymphadenopathy on staging CT C. Regional hepatic mets in the gallbladder fossa on staging CT D. Tumor invading the serosa E. Peritoneal studding
D
Incidental gallbladder cancer confined to mucosa is T1a and cured by the chole alone. However, if the gallbladder is removed piecemeal, risk of peritoneal dissemination is high.
Tumor invading serosa is T2. These patients have a demonstrated survival benefit after completion extended chole–resection of seg IVb and 5 of liver and portal lymphadenectomy.
Patients with more advanced disease, remote lymphadenopathy, peritoneal studding or hepatic mets do not derive benefit from complete radical resection. Peritoneal or liver mets define stage IV disease and are contraindications to further surgery
72M is receiving adjuvant cytotoxic chemo with FOLFOX. After the 3rd cycle, he develops fever, RLQ pain, WBC 4.1 and ANC of 500. Physical exam demonstrates tenderness in RLQ with equivocal signs of peritonitis. A CT was obtained and shows thick walled right colon, no free air. Which of the following is the best management option?
A. Continued chemo B. IV Abx and observation C. Colonoscopic decompression D. Appendectomy E. R hemicolectomy
E
Neutropenic enterocolitis or typhlitis is a relatively uncommon complication of chemo. It is transmural inflammation of the bowel, usually affecting the ileocecal region. Pathogenesis not fully elucidated, may be related to an acute mucosal injury with secondary infection. Usually assoc with chemo for hematologic malignancies, bone marrow transplant, and high dose chemo in solid tumors. Pooled incidence 5%. Agents include taxanes, cisplatin, oxaliplatin, irinotecan, anthracyclines, gemcitabine, vinorelbine, and 5-FU. Typical presentation is neutropenic patients with fevers and abdo pain. Most reliable diagnostic test is CT scan. Usually show colonic mural thickening, pericolic inflammation, pneumatosis intestinalis and ascites
Without absolute indications for sx, such as perforation or peritonitis, management is usually conservative, with broad spectrum Abx and observation, bowel rest, NG tube if N/V, hydration and TPN. Some physicians have recommended GCSF to reverse neutropenia. B/c chemo is the underlying cause, continued chemo is contraindicated. Sx is indicated in perforation, such as free air on AXR, or continued clinical deterioration. If sx is indicated, removal of all gangrenous bowel is necessary and extent of resection will depend on findings. In this patient, operation is not indicated, therefore, neither appy nor R hemi is appropriate. Colonoscopic decompression is used for Ogilvie or colonic volvulus and is not treatment fore neutropenic enterocolitis.
36M presents with intermittent BRBPR. Rectal exam reveals a friable fixed mass at the upper margin of the prostate. Bx confirms adenoca. CT scans of pelvis and abdomen show locally advanced rectal ca, mesenteric disease and a liver met. Which of the following would you recommend as the next step in management?
A. APR B. LAR with coloanal anastomosis C. Diverting colostomy D. Neoadjuvant chemoradiation E. Endoscopic US to determine therapy
D
He has T3N1M1 or Stage 4 disease. Even an uncomplicated immediate surgical resection will delay the initiation of systemic therapy and given the locale, would necessitate an APR with permanent colostomy. Diverting colostomy is not necessary since he is not obstructed. In the absence of obstruction, tx should focus on treating his systemic disease; initiation of neoadjuvant chemoradiation is first step in management. Potential benefits include decr toxicity and incr likelihood of patient completion of full tx course. Rationale for concurrent therapy lies in potentiation of tumor radiosensitivity. Addition of oxaliplatin or irinotectan to 5FU and leucovorin has led to clinically significant downsizing of inititally unresectable disease.
Which of the following is true regarding clinical use of genetic markers in colorectal cancer?
A. Chromosomal instability is associated with a better prognosis
B. Microsatellite instability is assoc with a worse prognosis
C. K ras status can predict responsiveness to cetuximab therapy
D. Lynch syndrome is characterized by overexpression of specific mismatch repair genes
E. Adenomatous polyposis coli and beta catenin mutations are rare
C
Most sporadic tumors result from chromosomal instability and acquired loss of specific genes. >90% of CRC are sporadic and can be traced to erros in cell cycle transcription due to loss of adenomatous polyposis coli (APC) gene and dysregulation of beta catenin. Small percent of inherited colorectal cancers (1%) arise secondary to germline mutations in the APC gene and are assoc with FAP
Microsatellites are simple repetitive sequences scattered throughout the genome; failure of mistmatch repair genes result in microsatellite instability. Identification of MSI is generally assoc with a better prognosis than chromosomal instability. Lynch syndrome is an example of inherited germline mutation in 1 such mismatch repair gene. Polyps in these patients are endoscopically indistinguishable from conventional adenomas but have complete loss of expression of a single mismatch gene in up to 88% of adenomas. Although a dx of Lynch cannot be made w/o genetic testing, screening by immunohistochemistry for MSH2 confers a presumptive dx. In addition, BRAF mutations are present in 40-50% of sporadic tumors due to MSI but are absent in Lynch
K-ras mutation are less responsive to epidermal growth factor receptor antibodies (anti-EGFR therapy), such as cetuximab or panitumumab, compared with patients with wild type K rase tumors. 30-40% of CRC harbor a K ras mutation. BRAF mutations are also assoc w/resistance to anti-EGFR regimens but are mutually exclusive of K ras mutations.
59M presents with progressive dysphagia. His EGD, EUS and CT scan show locally advanced but surgically resectable lesion. Bx demonstrates adenoca. Which of the following is the best course of management?
A. Surgical resection
B. Neoadjuvant chemorads and surgical resection
C. Chemoradiation
D. Surgical resection and adjuvant radiation
E. Surgical resection and adjuvant chemorads
B
Significant survival benefit for neoadj chemorads followed by surgical resection compared to surgery alone in patients with stage 1-3 adenoca of the esophagus.
60F has ulcerated 1 cm pigmented lesion on her R forearm. No assoc lymphadenopathy. Punch bx reveals a 0.8 mm thick melanoma. Which of the following is the next best step in her management?
A. WLE with 1 cm margin B. WLE with neg margin C. WLE with 1 cm margin and SLNB D. WLE with 2 cm margin and SLND E. WLE with 0.5cm margin
C
Tx of cutaneous melanoma incl excision of primary lesion along with assessment of nodal status in select pts. LN involvement is most important prognostic predictor of outcome in patients with clinically localized melanoma. Appropriate margins of excision are dictated by the depth of the lesion. Thin melanomas (<1 mm) should be excised with 1 cm margins. Intermediate melanomas (1-4 mm) should be excised with 1-2 cm margins. Thick melanomas (>4 mm) should be excised with 2 cm margins. Wider excision margins do not improve disease free or overall survival
SLNB is recommended for intermediate thickness melanomas with clinically neg nodal basins. Given the low risk of nodal mets, SLNB is not routinely recommended for thin melanomas. Likelihood of nodal mets increases in the setting of primary tumor ulceration, mitoses >1 and Clark level IV or V. Thus SLNB is recommended for lesions with these characteristics regardless of thickness <1 mm
70F undergoes L hemi for colon cancer. Although she denied famhx of GI or gyne malignancies during her preop work up, molecular pathology was performed on a tumor sample with the following results: MSI = high, absent staining of the MLH1 and PMS2 proteins by immunohistochemistry, BRAF V600E mutation and Kras wild type. Which of the following is true?
A. Patient has sporadic MSI
B. Patient has de novo germline mutation in MLH1 gene
C. Patient has germline mutation in PMS2 gene
D. Patient would not be candidate for anti-EGFR therapy
E. Family members should be notified they have Lynch syndrome
A
De novo germline mutation is an alteration in a gene that is present for the first time in 1 family member as a result of a mutation in a germ cell (egg or sperm) of 1 of the parents. De novo germline mutations in MLH1 are rare and assoc with a younger age at CRC dx. PMS2 staining is often absent when there is loss MLH1 expression. However, germline mutations in PMS2 gene are most often assoc with absence of PMS 2 only as seen in immunohistochem testing. ~15% of sporadic CRC display high level MSI, mostly due to somatic BRAF V600E mutation. This is not inherited but rather an acquired mutation. Further, presence of BRAF V600E mutation suggests a non inherited process
Lynch syndrome is an autosomal dominant cancer predisposition syndrome with mutations in various DNA mismatch repair genes (MSH2, MLH1, MSH6, PMS2). In addition to CRC, it infers incr risk for endometrial ca, ovarian ca, stomach ca, and other types of ca. Screening includes MSI and IHC testing to detect MMR defects in tumor, which determines whether further germline genetic testing is required. BRAF V600E is not observed in tumors from patients with Lynch
Colorectal tumors with Kras mutations are not responsive to anti-EGFR based tx. Therefore this patient who is wild type for KRAS may be a candidate for anti-EGFR therapy.
An incr risk of liver malignancy is assoc with which of the following lesions?
A. FNH > 5 cm B. Giant hemangiomas C. Echinococcal liver disease D. Glycogen storage disease E. Peliosis hepatis
D
EtOh and chronic hep C are the most commonly recognized RF contributing to cirrhosis and subsequent HCC, less common hepatic abN are also assoc with potential development of liver malignancy. Glycogen storage diseases have a reported incidence of 22-75% for hepatic adenomas. This incidence appears to incr with patient age but has been reported for patient 3-54 years. Adenomas carry the risk of malignant degeneration and most series report a 5-7% rate of malignancy. Patients with GSD should therefore be surveyed with liver US. Optimizing metabolic control with intensive dietary treatment may prevent occurrence or progression of adenomas and subsequent risk of malignancy
FNH has a reported freq of 3% in adults. Typically found incidentally in young women (8:1 F:M). Asymp in 80% of cases and hold no malignant risk. Hemangioma is most common benign lesion of liver with 5-20% prevalence. Few are symptomatic however when they are large >4cm, they may cause pain from capsular stretch. Rupture is rarely reported and they have no malignant potential. Echinococcal liver disease (hydatid) is caused by the larval cestode Echinococcus granulosus. Dogs are the definitive host, and sheep are the intermediate host, with man incidentally infected. Patients are infected early in childhood, and the infection may go undetected for years. Lesions may present with central necrosis and resemble HCC or large hemangiomas. Does not display delayed peripheral asym enhancement on CT. Surgical resection is preferred. No malignant risk. Alternative to sx is percutaneous aspiration combined with oral albendazole or mebendazole for patients who are poor surgical candidates.
Peliosis hepatitis is a benign disorder characterized by blood filled cystic cavities. Etiology unknown. Assoc with anabolic steroids and OCP use. Clinical course is benign. Typically resolved once offending drug is d/c’ed. May rupture, hemorrhage and liver failure but carries no malignant risk.
60 y.o. healthy male undergoes a CT scan of the abdo and is found to have an asymp 1.5 cm side branch IPMN in the body of the pancreas. Which of the following would you recommend?
A. Distal panc with splenectomy B. Observation and repeat imaging C. Total pancreatectomy D. Enucleation E. ERCP and stent
B
IPMNs account for 50% of incidentally found panc cysts and 7% of panc malignancies. Divided into main and side branch groups based on imaging with CT or MRCP.
Dilation of main duct >1 cm strongly suggests main duct IPMN, whereas a panc cyst communicating with the duct without main duct diln sugguests side branch IPMN. Main duct and side duct have significant diff in prevalence of cancer. Main duct IPMN should be resect b/c of high incidence of malignancy. Side branch is based on level of suspicion for malignancy. Suggestive RF include >3.0 cm, mural nodules, symptoms concerning for malignancy, calcifications, + cytology on bx.
When considering the safety of liver resection, which of the following is true regarding the future liver remnant (FLR)?
A. Portal vein ligation is preferable to portal vein embolization (PVE) in augmenting the FLR
B. An estimated FLR of >20% of preop liver is sufficient in normal liver
C. An estimated FLR of >30% of preop liver is recommended in compensated cirrhotic liver
D. PVE has a 25% rate of morbidity primarily from assoc pain
E. An absorbable gelatin sponge is superior to polyvinyl alcohol for PVE
B
Calculate FLR using CT measurements and calculated total liver volume on the basis of body surface area, which is independent of liver disease or tumor burden. Safe limits of resection are based on degree of underlying liver disease. >20% of preop liver volume in pts with N liver, >30% of preop liver volume in pts with early diseased liver and >40% in patients with well compensated cirrhosis
Selective PV occlusion before operation may impeded tumor progression while allowing time for FLR to hypertrophy, potentially expanding the number of patients who may undergo resection. Although occlusion of the PB may be accomplished by ligation at open operation or by selected percutaneous venous embolization, of the 2 methods, embolization is preferable.
Complications assoc with PVE occur in <5% and no specific embolization substance has emerged as superior.
Anal intraepithelial neoplasia in HIV postive men is
A. Pathologically similar to cervical cancer in women
B. Not preventable with current therapies
C. Readily curable with appropriate surgical care
D. Decr in incidence with current antiviral regimens
E. Rapidly fatal despite aggressive therapies
A
AIN is almost certainly the precursor in many, if not most, cases of anal SCC in HIV infected pts. Natural hx and management strategies are becoming more clear and common as more HIV pos patietns live with their chronic viral disease. HPV is understood to be carcinogenic in this area, as with cervical cancers in women. Pathology of SCC of the genital and anal regions is very similar. This disease may be largely preventable for men or women at risk, especially those who will have multiple partners in their lifetime with vaccinations. Antiviral regiments for HIV infections have not decr the rate of finding this disease, although active surveillance strategies may be uncovering more disease than known prev. AIN, which is essentially an in situ diseaase, is rarely fatal, even when it progresses to invasive anal cancer
When suspected, thorough EUA is warranted to determine the extent of disease with lesion directed bx. Anal pap smears are non specific for invasive disease and do not help manage patients with known AIN. Close follow up is best strategy, Bx of ulcerated or concerning lesions but not try to eradicate a chronic viral infection with radical extirpation
40 y.o. HIV + homosexual male is evaluated for hemorrhoids and anoscopy identifies a mass. Bx confirms the presence of SCC of the anus. For this condition, in comparison to HIV neg patients, HIV pos patients have
A. Decr survival rates
B. Incr rates of hematologic morbidity with chemorads
C. Decr local recurrence rates
D. Similar perineal wound complications with surgical tx
E. Older age at initial dx
B
Demographics of anal SCC are evolving with emergence of high risk group of patients: HIV + male homosexuals prone to develop AIN that rapidly progress toward invasive anal SCC. Caused by oncogenic HPV.
Neoadjuvant chemoradiation using 5-FU and mitomycin C before APR, several key points regarding tx have been established: 1) Anal SCC is a radiosensitive tumor, 2) Most anal cancers are cured with chemorads, 3) Surgical excsion is restricted to pts who fail to respond or who experience anal cancer recurrence after chemorads
HIV positive pts with evidence of persistent HPV infection within the anal canal progress more quickly toward invasive carcinoma adn are on avg 15 yrs younger that the time of initial dx. Overall survival of HIV postiive pts with anal SCC is similar to HIV neg patients, although major cause of death in such HIV pos patients is anal SCC. In addition, severe hematologic/skin toxicity can occur in up to 50% of HIV + pts tx with current chemo protocols.
Local recurrence after chemorads remain a problme in HIV + pts, developing in 50% of patients at 5 yr follow up. Finally, HIV + pts who do not respond to chemorads adn require operative tx have assoc higher rate of noncurative R1/R2 resections and perineal wound complications
Which of the following is a benign liver tumor with an incr risk of spontaneous hemorrhage?
A. FNH B. Biliary hamartoma C. Cystadenoma D. Adenoma E. Giant hemangioma
D
FNH 3% in adults. Found incidentall in young women (8:1 F:M). Asymp in 80% with no incr of spont hemorrhage.
Mesenchymal biliary hamartoma is a rare benign lesion seen most often in children with a slight male predominance w/o an incr risk of bleeding. Present with hepatomegaly. Lesion may mimic radiographic appearance of met dx and may require bx.
Cystadenoma is rare benign liver tumor that orginates from intrahepatic bile duct. May become symp by causing obstruction of bile duct or causing pain by capsular stretch. Although these tumors may degenerate to malignancy as cystadenocarcinoma, they carry no incr risk of rupture
Adenoma is rare benign liver tumor typically seen in yojng women. Reports of hemorrhage range from 30-50% and may be sudden and life threatening.
Hemangiomas are the most common benign liver lesion, with a 5-20% prevalence. Few hemangiomas are symptomatic, however when they are large (>4 cm) they may cause pain by capsular stretch but are not assoc with spontaneous rupture
Strategies to reduce medical exposure to radiation include all of the following except:
A. Using CT scanners with automatic exposure control
B. Substituting MRI for CT to evaluate soft tissue masses
C. Substituting barium enema for CT to evaluate diverticular disease
D. Substituting US for CT to follow aortic aneurysms
E. Substituting plain films and US for CTs in patients with flank pain and suspected kidney stones
C
Barium enemas involve higher radiation doses and therefore, risk are comparable with CTs
Recommended interventions for surveillance for patients after curative resection of stages II and III rectal cancer include all of the following except:
A. H&P B. Liver function tests C. CEA D. CT scan E. Colonoscopy
B
Majority of recurrences occur in first 3-5 years and involve liver and local recurrences.
Which of the following is not a factor used in the current AJCC TNM staging for melanoma
A. Breslow thickness B. Tumor mitotic rate C. Tumor ulceration D. Clark level of invasion E. Serum lactate dehydrogenase
D
Components of the Child Pugh score include all of the following except
A, Serum total bilirubin B. Serum albumin level C. Platelet count D. Encephalopathy E. Ascites
C
Includes Albumin, Bilirubin, INR, Ascites, and Encephalopathy
52M with known EtOH cirrhosis stopped drinking 6 yrs ago and is enrolled in screening program. Developed a 22 mm HCC. Labs show total bili 0.9, albumin 4.1 INR 1.2, plt 90, and cr 120. He is a child pugh A patient.
Which of the following statements regarding the tx of this patient’s HCC is true?
A. Percutaneous alcohol ablation and RFA have equivalent overall survival and tumor response
B. Overall survival is equivalent between RFA and resection
C. Overall survival is superior to liver transplant vs partial hepatectomy
D. There is no survival advantage for TACE over best supportive therapy
E. Overal survival is equivalent between TACE and radioembolization
C
Ideal tx in pt with cirrhosis an an HCC within Milan criteria (single < 5 cm or <3 tumors <3 cm), who is not a candidate for resection is liver transplant. In pts with advanced liver disease, overall survival with liver transplant is better than resection
Tumor <2 cm in diameter who are not resection or transplant candidates, ethanol ablation or RFA have equivalent outcomes but ethanol approach requires more tx sessions bc the necrotic effect ablation is not as predictable as with RFA. Tumors > 2 cm, the efficacy of RFA is better than ethanol.
Some data show equivalent outcomes with HCC <2cm tx either with resection or RFA. Other studies show that resection is superior to RFA. This finding remains controversial and resection continues to be the recommended tx in pts who are candidates for resection. For HCC at least 2-3 cm, RFA is inferior to resection.
Tx of HCC by TACE postively affects survival compared with best supportive therpay and is a good choice for patients with tumors who cannot be treated for curative intent by resection, transplant or ablation.
Radioembolization, the hepatic intra-arterial injeciton of yttrium-90 bound microspheres, can induce tumor necrosis but there are no data comparing its efficacy to TACE
All of the following are risk factors for Merkel cell except
A. UV radiation B. Immunosupression C. Polyomavirus D. Radiation exposure E. AIDS
D
Primary neuroendocrine tumor of the skin. Rare. Very high propensity for local recurrence and poor prognosis. Excision is primary tx, usually followed by radiation
UV exposure may contribute to MCC b/c it is accepted that UV exposure leads to down regulation of immune response. This immune suppresion can lead to an increase in MCC
HIgher incidence in immunocompromised pts, such as HIV and AIDS. Further supported by mean age of dx in transplant pts being decr by almost 20 yrs.
Merkel cell polyomavirus is implicated in pathogenesis of MCC, which may account for its higher incidence in immunosuppressed pts.
Incidence of MCC is not increased in pts exposure to radiation tx
All of the following are true regarding sporadic desmoid tumors except:
A. Spontaneous regression may occur
B. They commonly metastasize after resection
C. Surgery is the mainstay of therapy
D. Site of tumor is assoc with recurrence
E. Occur more commonly in women
B
Rare. Slow growing, mesenchymal proliferations with benign histologic features and virtually no malignant potential. Arise sporadically or in assoc with FAP. FAP assoc desmoids tend to occur more commonly in intra-abdo site or the anterior abdo wall. Estrogens have been of interest in the pathogenesis of sporadic desmoids, b/c women have a higher incidence, particularly those of childbearing age.
Sx is mainstay of therapy. High local recurrence rate, even after apparent complete sx resection; however there is no capacity for tumor mets. Variables assoc with local recurrence include site of tumor, b/c pts with tumor located at extremities or girdles having a higher recurrence rate, regardless of microscopic margin status. Effect of microscopic margin status is not clearly defined. Well documented cases of regssion, even after no tx, complicate tx dilemma and lead some to recommend expectant observation in some pts.
No current role exists for chemoprophylaxis . If disease is considered unresectable, NSAIDs and antiestrogens are considered first line therapies with cytotoxic chemo or radiation usually administered for those who progress on a noncytotoxic regimen.
The following statements regarding gallbladder carcinoma are true except
A. half of pts who undergo lymphadenectomy during radical resections have nodal mets
B. evaluation of <3 LNs during radical resection is assoc with worse survival
C. ability to obtain a microscopically neg (R0) margin of tissue is the key determinant of outcome
D. adjuvant radiation or chemo after curative resection reduces locoregional recurrence and improves survival
E. simple chole is adequate tx for pt with T1a tumor
D
Rare and aggressive. Radical resection, incl liver resection and lymphadenectomies are recommended for pts with resectable disease ad T1b (muscularis invasion) primary tumors. Pts with lymphadenectomyies that recover at least 3 LNs have sig improvement in long term survival compared with those who have fewer nodes recovered, particularly in T2/T3 tumors. Key determinant in outcome is ability to obtain a microscopically neg margin (R0).
Role of adjuvant therapy in gb carcinoma is debated with chemo and radiation being advocated as adjuvant tx b/c of high 5 yr recurrence in majority of pts with advanced dz.
The following statements about malignant panc neuroendocrine tumors are true except
A. Tumor size of at least 5 cm is assoc with a higher risk of recurrence and death
B. Among pts with advanced pancreatic neuroendocrine tumors who are not candidates for resection, sunitinib improves survival
C. Pts who undergo potentially curative liver resection have improved survival over those who undergo chemoembolization of met lesions
D. absence of LN mets is assoc with improved disease free survival after curative panc resection
E. functional neuroendocrine tumors have a worse prognosis than non functional
E
Prevalence of panc neuroendocrine tumors has incr, perhaps b/c of improvements in dx. Present challenges regarding prognosis b/c of risk of tumor recurrence and mortality correlates poorly with the usual gross and microscopic markers of malignant tumor behavior.
In study of pts with panc neuroendocrine tumors who underwent curative resection, tumor size > 5 cm and LN mets were assoc with worse disease free survival. Among pts with advanced tumors not amenable to curative resection, the tyrosine kinase inhibitor sunitinib improves progression free and overall survival. A majority of pts w/neuroendocrine tumors develop liver mets. Resections of these met tumors offer a survival advantage over chemoembolization. Controversy surrounds an association between neuroendocrine tumor fcn and prognosis, with the majority of studies showing no association.
Which of the following statements is true regarding nonparasitic splenic cysts?
A. Management should consist of splenectomy for cysts greater than 5 cm in diameter.
B. Percutaneous needle aspiration is often definitive therapy.
C. Symptom relief after percutaneous needle aspiration may predict response to operative management.
D. Most nonparasitic splenic cysts are symptomatic.
E. There is a high incidence of spontaneous or posttraumatic rupture of splenic cysts.
C
In the United States, nonparasitic splenic cysts may be posttraumatic or primary (figure 1). Rupture of splenic cysts either spontaneously or posttraumatically is rare. Studies suggest that fewer than 60% of posttraumatic cysts are symptomatic.
Few data support the use of cyst size as an indication for splenectomies in asymptomatic patients. Postaspiration cyst and symptom recurrence are very common; however, symptomatic relief with aspiration can suggest a benefit of operative management of splenic cysts. Because of the benign nature of the majority of splenic cysts, partial splenectomies or partial cystectomies (unroofing) are reported in most patients.
Which of the following statements is true regarding the management of patients with achalasia?
A. Overall symptom relief is higher with Heller myotomy and partial gastric fundoplication than with per-oral endoscopic myotomy (POEM).
B. The incidence of technical complications after POEM exceeds that seen after laparoscopic Heller myotomy and partial gastric fundoplication.
C. POEM and Heller myotomy achieve complete ablation of the lower esophageal sphincter complex.
D. POEM can achieve similar rates of postprocedural symptomatic esophageal reflux to those obtained with laparoscopic Heller myotomy and partial gastric fundoplication.
E. Laparoscopic Heller myotomy is combined with a partial gastroesophageal fundoplication to eliminate postoperative gastroesophageal leaks.
D
Per-oral endoscopic myotomy (POEM) divides only the circular muscle fibers of the lower esophagus and stomach. It is proposed that this incomplete ablation of the lower esophageal complex compared with the Heller procedure lowers the 20–100% incidence of gastroesophageal reflux that accompanies the Heller procedure and necessitates a gastroesophageal fundoplication in combination with a full-thickness myotomy.
A 2014 study compared consecutive groups of patients treated with the 2 procedures. Technical complication rates were similar with the 2 procedures. Successful treatment of dysphagia with POEM, and rarely symptomatic gastroesophageal reflux, was comparable to the results with Heller myotomy and partial gastric fundoplication. The purpose of adding partial gastroesophageal fundoplication to a Heller myotomy (circular and longitudinal muscle layers) is to prevent gastroesophageal reflux after this procedure
Men with asymptomatic or minimally symptomatic inguinal hernias are most likely to
A. experience a high rate of bowel obstruction or strangulation of hernia contents.
B. experience symptoms of pain some time in their lives if they choose “watchful waiting.”
C. fail “watchful waiting” if they are younger compared with older patients.
D. undergo operative therapy with a frequency of 20% over their lifetimes.
E. remain asymptomatic throughout their lives.
B
Seventy-two percent of patients who chose “watchful waiting” in the British trial went on to operative management over a median of 7.5 years, and 68% of patients who chose “watchful waiting” in the US trial went on to operative management over a median of 10 years.
Among patients who were 65 years of age or older, the operative conversion rate was 79% in the US study. The main reason for choosing operations was pain. Both studies demonstrated less than 3% rates of hernia complications (bowel obstruction or strangulation of hernia contents) during periods of “watchful waiting.”
Which of the following statements is true about carcinoid tumors of the appendix?
A. The majority present with carcinoid syndrome.
B. The depth of tumor invasion is the most accurate predictor of lymph node metastasis.
C. The status of regional lymph nodes is the single most important factor influencing 10-year survival.
D.The tumor size is the most accurate predictor of lymph node metastasis.
E. Appendectomy alone is adequate treatment for tumors less than 4 cm in diameter.
D
The majority of appendiceal carcinoid tumors are small, and they are discovered incidentally. Lymph node metastases from the tumors are predicted most accurately by size, with tumors less than 1 cm in diameter and confined to the appendix not associated with lymph node metastases and treated with appendectomy alone. Tumors greater than 2 cm in diameter are treated with right hemicolectomy. Controversy surrounds the management of patients with tumors 1–2 cm in diameter as well as carcinoids at the base of the appendix. The status of regional lymph nodes does not influence 10-year survival. Because the majority of tumors are less than 1 cm in diameter when they are discovered and unlikely to be associated with metastatic disease, the incidence of carcinoid syndrome is very rare in patients with appendiceal carcinoid tumors.
A 44-year-old man with poorly controlled diabetes was diagnosed with severe pancreatitis and associated pancreatic necrosis (figure 1). He was managed without intervention and discharged after 6 days. He re-presents 2 weeks later with increasing diffuse abdominal pain. He is normotensive with a heart rate of 105 beats per minute and a temperature of 38.3°C. He has diffuse abdominal tenderness without peritoneal signs. His white blood cell count is 24,000/mm3 (3600–11,200/mm3), his hemoglobin is 10 g/dL (men 13.5–17.5 g/dL), and his serum glucose is 380 mg/dL (70–100 mg/dL). Repeat imaging is seen (showed large retroperotoneal collection with bubbles of gas). Which of the following is the next most appropriate step in this patient’s management?
A. Admission with nasogastric tube placement, bowel rest, and octreotide
B. Retroperitoneal debridement
C. Percutaneous drain placement
D. Exploratory laparotomy
E. Endoscopic placement of distal feeding access for enteral nutrition
C
This patient has evidence of infected pancreatic necrosis (figure 3). Up to 30% of patients develop infection after necrotizing pancreatitis, with presentation on average 3–4 weeks after the original management of pancreatitis. In this case, the extraluminal gas evident on CT imaging is suggestive of the diagnosis, even without culture results.
Management of infectious complications after pancreatitis and pancreatic necrosis has evolved in recent years from mandated laparotomy and open necrosectomy. Recent reports demonstrate that percutaneous drainage may control sepsis and avoid surgery in more than 50% of patients; if the patient has ongoing sepsis uncontrolled by percutaneous drainage, surgical intervention should be entertained. In those cases, the percutaneous drain can serve as a “road map” to direct a minimally invasive approach, such as the step-up approach or video-assisted retroperitoneal debridement. Minimally invasive surgical approaches have lower morbidity (e.g., multiple organ failure, incisional hernia, new onset diabetes) and mortality rates compared with open necrosectomy.
Enteral nutrition tends to improve outcomes for pancreatitis, so indefinite bowel rest is not advocated. Antibiotics alone for a fluid collection of this size, particularly in a patient with sepsis, is not advocated. Distal feeding access may be desirable, particularly for those patients with a gastric ileus, but that would not be the first step in management.
A 60-year-old man presents with epigastric pain radiating to his back and nausea. On exam, he has focal epigastric tenderness. He has elevated transaminases, a lipase of 1800 units/L (reference is 10–40 units/L), and normal bilirubin. Right upper quadrant ultrasound demonstrates cholelithiasis and a common bile duct measuring 9 mm. After admission to the hospital, intravenous fluid resuscitation, and pain medication, his pain resolves and laboratory values normalize. Which of the following is the next step?
A. Laparoscopic cholecystectomy
B. Endoscopic retrograde cholangiopancreatogram
C. Discharge home with outpatient follow-up
D. Magnetic resonance cholangiopancreatogram
E. Laparoscopic cholecystectomy at 6 weeks
A
Gallstones and alcohol are the most common causes of acute pancreatitis. The presence of high serum lipase and typical abdominal pain suggests the diagnosis in the absence of a CT scan. The presence of gallstones by abdominal ultrasound suggests the etiology, which can be further supported by the presence of a dilated common bile duct and the finding of elevated serum transaminases. The vast majority of stones pass spontaneously within 48 hours of presentation.
Mild cases can be treated with hospital admission, fluid resuscitation, and pain control. In patients with more severe disease, cholangitis, persistent hyperbilirubinemia, clinical deterioration, or detection of a persistently impacted common bile duct stone, endoscopic retrograde cholangiopancreatogram (ERCP) is warranted within 24–48 hours. In patients with resolving symptoms of gallstone pancreatitis, cholecystectomy is indicated to remove the source of stones and prevent recurrence. Delayed cholecystectomy is associated with a high rate of recurrence of pancreatitis.
A randomized trial demonstrated that early laparoscopic cholecystectomy is associated with a shorter overall length of hospital stay without increasing operative complications or conversion rates, compared with delayed cholecystectomy. Magnetic resonance cholangiopancreatogram (MRCP) does not offer therapeutic options and is reserved for cases that have an equivocal diagnosis or anatomy.
A 60-year-old man with known chronic liver disease secondary to alcohol abuse presents with acute onset right upper quadrant abdominal pain. He is diaphoretic. His blood pressure is 90/40 mm Hg, his heart rate 120 beats per minute, and his respiratory rate 20. Resuscitation is initiated. After improvement of his vital signs, he undergoes a CT scan that shows hemorrhage from a 4-cm liver mass (figure 1). Therapy should begin with which of the following?
A. Biopsy of the mass B. Transjugular intrahepatic portosystemic shunt C. Embolization D. Laparotomy and packing E. Mass resection
C
This individual is in a high-risk group for hepatocellular carcinoma (HCC), and this presentation is likely a ruptured HCC. A biopsy is unnecessary and unsafe at this point. The immediate goal is control of the bleeding. The diagnosis may be confirmed based on the result of the recommended therapeutic arteriography.
The patient’s abnormal hemodynamics and active bleeding warrant urgent intervention. HCCs have a rich arterial supply, and embolization is the preferred first intervention in the setting of acute rupture. The expected success rate would be better than 80%.
The bleeding is clearly arterial, as evidenced by the blush seen on CT scan (figure 2), so transjugular intrahepatic portosystemic shunt (TIPS) would not be indicated. TIPS would be considered in the setting of bleeding from esophageal varices refractory to endoscopic intervention. Surgical approaches to this presentation—resection or packing—carry a significant risk of morbidity and mortality in the acute setting in patients with chronic liver disease and should be reserved for patients failing embolization.
A 55-year-old man had an esophagectomy for adenocarcinoma of the esophagus (stage IIa). Six months later, he is found to have 3 technically resectable liver metastases 3–6 cm in diameter. Which of the following statements is true about liver-directed therapy for this patient’s presentation?
A. Resection confers a survival advantage over chemotherapy.
B. Radiofrequency ablation has a low local failure rate.
C. External beam radiation is curative.
D. Systemic chemotherapy should be offered.
E. Transarterial chemoembolization is the preferred care for hepatic metastases of noncolorectal, nonneuroendocrine cancers.
D
In this patient, there are several contraindications to liver-directed therapy. The short disease-free interval and presentation with multiple lesions suggests poor tumor biology, or inadequate initial staging, or both. Metastatic esophageal adenocarcinoma is rarely confined to the liver; therefore, systemic therapy is indicated. There is no evidence that resection has added benefit over systemic chemotherapy.
Noncolorectal, nonneuroendocrine cancers metastatic to the liver are a heterogeneous group of diagnoses with highly variable biology. Few tumor types, other than colorectal carcinoma, are known to metastasize to the liver in a manner amenable to curative intent intervention. Most studies show no benefit of resection or ablation of liver metastases in the treatment of gastrointestinal malignancies.
Radiofrequency ablation and other percutaneous thermal ablation techniques enjoyed popularity in the past decade due to ease of delivery and generally favorable safety profiles. The limitations of the technology include tumor number and size. The ability to reliably and safely destroy tumors larger than 3 cm is in question, with local failure rates in tumors larger than 5 cm being greater than 30%.
External beam radiation is an attractive option given recent advancements in tumor localization and delivery. The literature supports a role in palliation of symptomatic lesions but not in curative intent treatment. Likewise, transarterial chemoembolization has a theoretical role in palliation of vascular lesions but is not a standard for any broad set of diagnoses.
A 41-year-old man with Child class A alcoholic cirrhosis undergoes his first screening upper endoscopy. He has no history of upper-gastrointestinal bleeding. Upper endoscopy identifies the presence of large varices with no red wheals. Initial primary prophylaxis against variceal hemorrhage for this patient is
A observation.
B beta-blockade.
C endoscopic variceal sclerotherapy.
D endoscopic variceal ligation and beta-blockade.
E spironolactone.
B
The initial diagnosis and classification of nonbleeding esophageal varices is usually made on esophagogastroduodenoscopy. In most centers, varices are classified by a semiquantitative morphological assessment into 3 sizes or grades:
- Small (grade I), generally defined as minimally elevated veins above the esophageal mucosal surface normal in color, straight, and compressible.
- Medium (grade II), medium varices defined as tortuous veins occupying less than one-third of the esophageal lumen, with or without red wheals, and noncompressible.
- Large (grade III) defined as those occupying more than one-third of the esophageal lumen, with or without red wheals, and noncompressible.
Nonselective beta-adrenergic blockers are the foundation of therapy for primary prophylaxis for preventing variceal bleeding. Patients with cirrhosis and varices that have not bled should be started on primary prophylaxis. A meta-analysis of 11 trials (1189 patients) evaluated nonselective beta-blockers (e.g., propranolol, nadolol) versus nonactive treatment or placebo in preventing first variceal hemorrhage. In patients with large- or medium-sized varices, risk of first variceal bleeding was significantly reduced by beta-blockers (30% in controls vs. 14% in patients treated with beta-blockers). One bleeding episode was avoided for every 10 patients treated.
Two invasive endoscopic approaches to manage esophageal varices are available: sclerotherapy and banding. Endoscopic sclerotherapy is associated with a higher risk of side effects compared with variceal band ligation (VBL). Therefore, variceal band ligation is the preferred method for endoscopic prophylaxis of variceal bleeding. A meta-analysis of 5 randomized clinical trials comparing VBL with no treatment showed a decreased risk of first variceal bleeding and a lower mortality rate in the VBL group.
Spironolactone in combination with beta-blockade shows no benefit over beta-blockade alone for primary prophylaxis of variceal bleeding.
Nonselective beta-blockers or variceal band ligation may be an appropriate first-line choice for primary prophylaxis of variceal bleeding. However, the combination of both therapies is not more effective and is associated with increased side effects.
A 24-year-old man undergoes an ultrasound of the abdomen for nonspecific generalized abdominal pain. Three 7- to 8-mm polyps are identified in the gallbladder. The next step in the management of this finding would be
A CT scan of the abdomen.
B endoscopic ultrasound.
C laparoscopic cholecystectomy.
D open cholecystectomy with lymph node sampling.
E repeat ultrasound in 6 months.
E
Gallbladder polyps are identified in 1.5–6.9% of the population. They can be classified as “pseudotumors” (cholesterol polyps, adenomyomas, or inflammatory polyps), epithelial (adenomas), mesenchymal (leiomyomas), and malignancies (adenocarcinomas). The main concern when a polyp is identified is whether it is malignant. Single polyps are more likely to be malignant than multiple polyps. The feature most predictive of malignancy is the size of the polyp. A polyp larger than 1 cm in size has a risk of malignancy of 43–77%, whereas polyps larger than 2 cm are nearly always malignant. Age also needs to be considered; patients older than 50–60 years have a higher risk of malignancy. Finally, the presence of gallstones along with polyps or a diagnosis of sclerosing cholangitis and polyps increases the likelihood of malignancy.
With the knowledge of these risk factors, a care plan algorithm can be designed. In a patient with true biliary symptoms, cholecystectomy is indicated based on the symptoms. The presence of polyps does not influence that decision. In patients with no symptoms and polyps 5 mm or smaller, a follow-up ultrasound in 6–12 months is indicated. If the polyps have not increased in size, no further imaging is needed. Polyps 6–9 mm in size can be followed with ultrasound. Typically, a follow-up study is obtained at 6 months and 1 year and then yearly thereafter if polyp size is stable. If the polyps decrease in size, no further imaging is needed. If they increase in size, a cholecystectomy is indicated. An exception could be a patient over the age of 50 with a single polyp larger than 5 mm. In a patient older than 50, cholecystectomy is indicated.
All polyps 10 mm or greater in size require surgery. CT scanning should be considered in patients with polyps greater than 10 mm and age younger than 60 and in all patients with polyps greater than 20 mm to evaluate for invasive cancer.
The role of endoscopic ultrasound in the management of gallbladder polyps is not well defined. Studies show it can be used to predict pathology at the time of cholecystectomy in 97% of patients. However, it is more invasive and expensive, and it requires endoscopists who have advanced endoscopic ultrasound training. Thus, although it may be useful in evaluating and staging larger polyps, it may not be the most cost-effective way to follow patients with smaller polyps. Open, nonradical cholecystectomy with lymph node sampling is not a procedure used for benign or malignant gallbladder pathology.
A 47-year-old woman presents with fever and left upper quadrant pain. A CT scan is obtained and shows a splenic abscess. The most appropriate treatment for this lesion is
A intravenous antibiotics. B intravenous antifungals. C percutaneous drainage with intravenous antibiotics. D splenectomy. E splenic embolization.
C
Splenic abscesses are rare, but if not treated appropriately, they have a high mortality rate. The most common etiology is hematogenous spread, such as from infective endocarditis. Splenic trauma is another common etiology, as is splenic infarction. Up to 50% of these infections are polymicrobial. For this reason, empiric broad-spectrum antibiotics should be initiated as soon as the diagnosis is made. Primary fungal splenic abscesses are rare and antifungal therapy would not be initiated as part of empiric therapy. Antibiotics alone are not considered definitive treatment.
Percutaneous drainage is now the procedure of choice for splenic abscess. Aspiration allows for culture-directed antibiotics in addition to preserving the spleen. Contraindications to percutaneous drainage include multiple abscesses and cysts that have features that make a diagnosis of Echinococcus more likely, such as a calcified cyst wall and the presence of other cysts in the abdominal cavity, particularly in a patient from the Mediterranean basin or Eastern Europe. Additionally, patients who are coagulopathic or those without safe percutaneous access to the abscess are not candidates for percutaneous drainage.
Splenectomy is reserved for patients who are not candidates for or who have failed percutaneous drainage. There is no indication for the use of splenic embolization in splenic abscess, and it is one of the etiologies of splenic abscess.
A 38-year-old man underwent an open right inguinal hernia repair. The Lichtenstein technique was used. He had significant pain immediately after the procedure. This pain has persisted for 6 months. He describes the pain as a shooting or “electric shock” type pain that starts in the right lower quadrant and radiates to the right testicle. On exam, palpation over the internal ring reproduces his symptoms. He does not have a recurrent hernia. Which of the following would be the next step in his treatment? A Pulse steroids with rapid taper B Physical therapy for scar release C Mesh excision D Triple neurectomy E Diagnostic laparoscopy
D
Pain is to be expected after any operation. It is important to recognize when pain has transitioned from acute to chronic. For an inguinal hernia repair, pain that exists beyond 3–6 months, which would be expected for the normal postoperative inflammatory response, is defined as chronic pain. It is estimated that 10–12% of patients will suffer chronic pain after inguinal herniorrhaphy (CPIH) and that 0.5–6% of patients will have pain that affects their daily life or employment.
Evaluation of CPIH requires the surgeon to know the different types of pain that patients experience. Nociceptive pain is acute pain associated with acute stimulation or damage to sensory nerves. Touching a hot stove or a pain associated with a surgical incision are common causes. This type of pain often transitions to inflammatory nociceptive pain, in which inflammation leads to stimulation of the sensory nerves and the sensation of pain. An example would be pain that persists after spraining an ankle. These 2 types of pain represent the normal response to surgical injury.
Etiologies of chronic pain can be simple to complex. Inflammatory pain can persist. This can occur if there is ongoing inflammation of the tissue and or a nerve in the region. Nerves injured at the time of surgery can form a neuroma that is hypersensitive. Ongoing pain stimuli can lead to a peripheral neuropathy, where there is peripheral amplification of normal nociceptive signals. Ongoing pain stimuli can lead to a central neuropathy where spinal cord neurons send increased pain signals to the brain. This is seen in allodynia (normal stimulus leads to the sensation of pain) and hyperalgesia (amplified response to a painful stimulus).
For CPIH, it is important to differentiate hernia recurrence, direct nerve injury associated pain, chronic inflammatory pain, and neuropathic pain. The first 3 have surgical options, whereas a pain management specialist best treats the fourth.
The pain associated with hernia recurrence is typical of an inguinal hernia. If physical exam is negative, an ultrasound exam done with the patient standing and performing a Valsalva maneuver can help to identify an occult recurrence. If the patient seems to be describing a recurrence, and evaluation is negative, diagnostic laparoscopy can be performed. Treatment is repair of the hernia.
Direct nerve injury can lead to neuroma formation. Stimulation of the neuroma will cause an electric shock type pain. In the groin, the ilioinguinal, iliohypogastric, and genitofemoral nerves are all susceptible to injury. Because of the overlapping areas these nerves supply sensation to, any history of this type of pain or physical exam that elicits this type of pain (Tinel sign) should lead to nerve injury as the cause of CPIH. Nerve blocks can be attempted to confirm the diagnosis, but failure of the block could be based more on injection technique than wrong diagnosis. For this reason, with this type of pain, the surgeon can make a direct recommendation for triple neurectomy for treatment.
Chronic inflammatory pain can occur due to the mesh, inciting chronic inflammation. Additionally, the mesh can contract, forming a “meshoma,” which can lead to chronic tissue inflammation and irritation. This pain typically remains in the groin and can be worse with movement. There is usually pain on palpation of the inguinal floor, but no electric shock type pain. Treatment can be initiated with anti-inflammatory agents, including steroids, in severe cases. If the mesh is stiff on exam, physical therapy, including scar release, can be attempted. If these fail, then mesh excision is considered.
The most difficult diagnosis is that of chronic neuropathic pain. On exam, the physician will find a severe reaction to normal stimuli or to minimally painful stimuli. Pain will not be localized to the groin and may be described as the entire lower abdomen. Investigation for hernia recurrence is important. If a recurrent hernia is not identified, these patients are best treated by a pain specialist.
A 23-year-old male soccer player presents with left groin pain. On exam, he does not have an inguinal or femoral hernia. There is pain with palpation of the inguinal floor and the pubic tubercle. Dynamic ultrasound is normal. MRI shows edema of the marrow under the tubercle and mild inflammation at the insertion of the conjoint tendon. Initial treatment for these findings is
A monthly steroid injections for 3 months.
B monthly platelet rich protein injections for 3 months.
C no weight bearing with the left lower extremity for 6 weeks.
D physical therapy to address muscle imbalances and flexibility.
E surgery with mesh insertion and adductor tenotomy.
D
Pain in the inguinal region without evidence of hernia is known by many names. Sports hernia and athletic pubalgia are used interchangeably in the literature, even though they are different entities. The term pubic inguinal pain syndrome (PIPS) is now used to describe the various forms of non-hernia-associated inguinal pain.
Most important in any discussion of PIPS is to ensure that the patient does not have an inguinal or femoral hernia. This is done by exam and, if negative, a standing Valsalva ultrasound (dynamic ultrasound) of the groin. If a hernia is not identified with either, then PIPS should be considered.
PIPS has 2 main etiologies. The first is weakness or laxity of the inguinal floor. This is the classic “sports hernia.” Patients will describe more classic hernia symptoms and on exam, with palpation of the inguinal floor, pain is reproduced. Dynamic ultrasound can show weakness and bulging of the floor. Surgical reinforcement of the inguinal floor is the treatment of choice.
The second etiology is inflammation and injury at the pubic tubercle and the associated musculotendinous insertions. Patients describe pain in the proximal medial thigh, below the scrotum, in the lower abdomen, and at the pubic bone. On physical exam, there is tenderness to palpation of the pubic tubercle and there may be tenderness along the thigh adductor tendons, the lower rectus muscle, or the medial aspect of the conjoined tendon. Diagnosis can be confirmed with MRI scanning, which will demonstrate inflammation of these areas. The etiology of these findings is thought to be muscular imbalances of the abdomen, pelvis, and proximal lower extremity. For this reason, physical therapy is the initial treatment of choice. If physical therapy fails, surgery to off-load tension on the pubic tubercle in addition to adductor tenotomy is indicated.
Steroid and platelet-rich protein injections are used to treat osteitis pubis and would not be indicated for PIPS. If MRI identified a pelvic stress fracture, limitation of weight bearing would be considered. It is not used for PIPS.
Two weeks ago, a 69-year-old man with a remote history of right hemicolectomy for a T2N0 colon cancer was admitted with right upper quadrant pain, fever, and jaundice. On presentation, he was hemodynamically abnormal but responsive to fluid resuscitation. Laboratory values were as follows: white blood cell count of 24,000/mm3 (3600–11,200/mm3), bilirubin of 14 mg/dL (0.2–1.9 mg/dL), alkaline phosphatase of 670 U/L (<95 IU/L), glutamic-oxaloacetic transaminase of 250 U/L (<42 U/L), gamma-glutamyl transferase of 175 U/L (10–70 U/L). Gallbladder ultrasound showed a thickened gallbladder with a positive Murphy sign but without cholelithiasis and a normal caliber common bile duct. The patient initially underwent magnetic resonance cholangiopancreatography (showing bead on string appearance), followed by a cholecystostomy that drained bile and pus. Despite broad-spectrum antibiotics for 7 days, his white blood cell count remains at 15,000/mm3 and his bilirubin is 9 mg/dL. Which of the following would you recommend?
A Laparoscopic cholecystectomy B Liver biopsy C Serum IgG4 D Cancer antigen 19-9 level E Endoscopic retrograde cholangiopancreatography with stent placement
C
This patient presents a picture of cholangitis without evidence of the distal bile duct obstruction that would be expected with choledocholithiasis, a stricture complicating pancreatitis, or obstructing pancreatic/common duct tumor. The incongruity of history/physical and laboratory values should prompt further investigation before considering operative intervention. The degree of hyperbilirubinemia and elevated alkaline phosphatase do not support a diagnosis of acalculous cholecystitis. Because of its high sensitivity and specificity, noninvasive magnetic resonance cholangiopancreatography (MRCP) has largely replaced endoscopic retrograde cholangiopancreatography (ERCP) as the initial diagnostic test of choice. The MRCP images for this case show the classic “bead-on-a-string” appearance of sclerosing cholangitis (figure 2). Although placement of a stent might be useful for biliary decompression in the face of a dominant extrahepatic stricture, significant clinical improvement is unlikely in the presence of intrahepatic strictures. Because this patient may ultimately be considered for liver transplant, cholecystostomy drainage and a prolonged course of antibiotics is the most appropriate initial management for this patient.
Primary sclerosing cholangitis is the classic hepatobiliary manifestation of inflammatory bowel disease, leading to cirrhosis and end-stage liver disease caused by chronic and progressive biliary strictures. Disease with no known precipitant is deemed primary sclerosing cholangitis, whereas a preceding identifiable biliary injury is designated secondary disease. Autoimmune diseases, including IgG4-associated autoimmune pancreatitis, ischemia, toxins, and inherited disorders, mimic the historical and radiologic features shown in the MRCP images. If confirmatory, a trial of corticosteroid immunosuppression is appropriate, and, in the absence of hepatocellular carcinoma, suitable patients may be considered for liver transplantation. Although cancer antigen 19-9 (CA 19-9) is an indicator of pancreatic carcinoma, it has poor sensitivity for early-stage cholangiocarcinoma. Because increased CA 19-9 concentrations may result from cholangitis alone, routine measurement in this setting is less valuable. Diagnosis of small-duct disease necessitates liver biopsy when a potential overlap with autoimmune hepatitis is suspected (10% of patients).
A 48-year-old man with known cirrhosis due to hepatitis C is being screened for hepatocellular carcinoma. His CT scan shows portal vein thrombosis. Which of the following would you recommend? A Splenectomy B Immediate liver transplant C Observation D Splenorenal shunt E Anticoagulation
E
Portal vein thrombosis (PVT) is a complication of cirrhosis commonly identified on imaging of patients with advanced liver disease and is associated with portal hypertension and hypersplenism. Chronic partial PVT may be asymptomatic, whereas acute complete obstruction may induce intestinal congestion with severe continuous or colicky abdominal pain and occasionally nonbloody diarrhea. Although PVT and its attendant portal hypertension pose a significant risk of variceal bleeding, a treatment algorithm combining anticoagulation and transjugular intrahepatic portosystemic shunting (TIPS) offers the best chance for restoring portal flow, reducing portal pressures, reducing thrombosis extension, and reducing the risk of intestinal infarction.
Although appropriate for management of bleeding gastric varices due to splenic vein thrombosis, splenectomy decreases portal vein flow, thus increasing the risk of PVT. Splenorenal shunt, an option for management of bleeding varices, also decreases portal blood flow. PVT alone is not an indication for liver transplantation nor is it necessarily a contraindication. Its presence, however, is an independent risk factor for recurrent PVT after transplant and decreased perioperative survival. Complete or partial recanalization is associated with better survival rates and, therefore, anticoagulation is recommended in all patients with PVT.
A 54-year-old woman presented to her primary care provider with intermittent, right upper quadrant pain associated with fatty foods. Ultrasound showed a distended gallbladder without evidence of cholelithiasis. Blood work included an alkaline phosphatase of 245 U/L (<95 U/L) associated with a normal bilirubin and serum glutamic-oxaloacetic transaminase of 28 U/L (10–55 U/L). Based on the magnetic resonance cholangiopancreatography (MRCP) scan showing an extrahepatic cyst, which of the following would you recommend?
A Endoscopic retrograde cholangiopancreatography with biopsy and stent
B Laparoscopic cholecystectomy with cholangiogram and common duct exploration
C Whipple (pancreaticoduodenectomy)
D Cholecystectomy with bile duct excision and Roux-en-Y hepaticojejunostomy
E Cholecystectomy with choledochoduodenostomy
D
This patient’s workup was initiated by a concern for biliary colic. The ultrasound did not show cholelithiasis. Her laboratory picture of normal bilirubin and transaminase values with a marked elevation in alkaline phosphatase supports further imaging of the biliary tree. In this case, the magnetic resonance cholangiopancreatography (MRCP) shows a type I choledochocyst (figure 2). There is no evidence for distal or common duct obstruction warranting endoscopic retrograde cholangiopancreatography (ERCP) with stent or cholecystectomy with common duct exploration. A pancreaticoduodenectomy or Whipple procedure is generally reserved for a distal bile duct or pancreatic head malignancy. In this case, cholecystectomy with bile duct excision and Roux-en-Y hepaticojejunostomy is the procedure of choice.
Choledochocysts are single or multiple dilations of the intrahepatic or extrahepatic biliary tree (figure 3). The classic triad of jaundice, right upper quadrant pain, and an abdominal mass is less common in adults than in children. Untreated, patients are at risk of developing recurrent cholangitis, liver abscesses, and cholangiocarcinoma. Those confined to the common bile duct, as in this patient, are well managed by cholecystectomy with complete cyst excision to include any abnormal common duct from the hepatic plate to the intrapancreatic portion of the duct for fusiform dilations and hepaticoenterostomy reconstruction. The hepaticoenterostomy can be performed as a hepaticoduodenostomy or a Roux-en-Y hepaticojejunostomy. The latter approach is preferred, because it excises all potentially premalignant cyst tissue and separates the biliary tree from the pancreatic duct, eliminating any mixture of pancreatic and biliary secretions that may contribute to disease pathogenesis.
Hepaticoduodenostomy carries the risk of bilious gastric reflux, gastritis, and esophagitis. Overall complications are substantially lower with Roux-en-Y reconstruction (7%) compared with hepaticoduodenostomy (42%). If the cyst cannot be completely excised, stripping or ablation of the mucosa is recommended and the patient should be followed with regular ultrasound surveillance for evidence of malignancy development.
A 75-year-old woman on warfarin for management of atrial fibrillation presents to the emergency department complaining of abdominal pain. Her blood pressure is 100/70 mm Hg, her heart rate is 110 beats per minute, and she is afebrile. Physical examination demonstrates a tender mass in the right lower quadrant, which does not change in character when she flexes her abdominal wall musculature. Which of the following is the next most appropriate step in management?
A Ultrasound of the right lower quadrant B Broad-spectrum intravenous antibiotics C CT scan of her abdomen and pelvis D Mesenteric angiogram E Urgent exploratory laparotomy
C
Rectus sheath hematoma (RSH) is a condition characterized by abdominal pain and an abdominal wall mass. Fothergill sign is an abdominal wall mass that does not cross the midline and remains present with contraction of the abdominal wall muscles. It is useful to differentiate between a mass arising in the rectus muscle and intra-abdominal masses. This sign would be positive with RSH.
Important risk factors for RSH include female sex, older age, anticoagulation therapy, and cough or other abdominal trauma. A high index of suspicion in the appropriate clinical setting can lead to rapid diagnosis via CT scan (figure 1) of the abdomen and pelvis, which is the diagnostic modality of choice for RSH and is more accurate than ultrasonography for this clinical entity. Most patients are successfully treated with symptom management. Anticoagulation should be reversed, if at all possible.
There is no indication for antimicrobial treatment. Criteria for blood transfusion do not change with RSH, and few patients will require this. Mesenteric angiography would be appropriate in the setting of mesenteric ischemia, but the presentation would not be consistent with this diagnosis. Angiography might be needed if bleeding does not stop with reversal of anticoagulation. The need for operative intervention is even rarer than angiographic intervention, and typically arises when the patient is hemodynamically abnormal and other therapies have failed, or if the hematoma becomes secondarily infected and requires drainage
A 50-year-old man with alcoholic cirrhosis and ascites presents with abdominal pain, fever, and leukocytosis. His blood pressure is 88/50 mm Hg, and his heart rate is 100 beats per minute. Laboratory data are as follows:
Blood urea nitrogen = 45 mg/dL (7–20 mg/dL)
Creatinine = 1.7 mg/dL (0.40–1.30 mg/dL)
International normalized ratio = 2.5 (0.8–1.2)
Albumin = 2.0 g/dL (3.5–5.2 g/dL)
Diagnostic paracentesis demonstrates 500 neutrophils/mm3 of ascitic fluid. Which of the following therapies is indicated as a first step in treatment?
A Vancomycin B Metronidazole C Fresh frozen plasma D Cryoprecipitate E Third generation cephalosporin
E
Spontaneous bacterial peritonitis (SBP) is a bacterial infection of ascitic fluid without an intra-abdominal source of infection. SBP is associated with underlying conditions such as cirrhosis, nephrotic syndrome, and congestive heart failure. The patient will have abdominal pain, fever, leukocytosis, and an ascitic fluid sample demonstrating more than 250 neutrophils/mm3.
Broad-spectrum antibiotics, typically a third-generation cephalosporin, are started immediately. Gram-positive and anaerobic organisms rarely cause SBP in cirrhotic patients making vancomycin or metronidazole inappropriate. Because the patient is not bleeding, neither fresh frozen plasma nor cryoprecipitate is necessary.
Laparoscopic cholecystectomy carries a higher risk of a bile duct injury than open cholecystectomy. One approach to mitigate this risk is to achieve the “critical view of safety.” Which of the following best describes the critical view of safety?
A Visualizing contrast in the duodenum on cholangiogram
B Visualizing 2 tubular structures entering the neck of the gallbladder and the liver/cystic plate behind
C Visualizing the cystic duct–common bile duct junction
D Dissecting the gallbladder free from the liver bed in a top-down fashion
E Visualizing 2 tubular structures in the triangle of Calot
B
Laparoscopic cholecystectomy is associated with a 0.4% incidence of bile duct injury. Errors in identification are the most common cause of bile duct injuries during laparoscopic cholecystectomy. The “critical view of safety” is advocated as a means of ensuring the cystic duct and artery are clearly identified and that no other structures are adherent to or mistaken for these structures before dividing them. The essential elements of this critical view are to visualize 2 and only 2 tubular structures entering the neck of the gallbladder with a view of the liver and cystic plate behind these cystic structures.
Although intraoperative cholangiography can be complementary to the critical view of safety, it is not required. The cystic duct–common bile duct junction should not be routinely dissected, because this maneuver can actually increase the risk of a bile duct injury. At times, a top-down mobilization of the gallbladder may be necessary, but this is not part of or required for the critical view of safety. Visualizing 2 tubular structures in the triangle of Calot alone does not constitute this critical view.
A 52-year-old woman with ulcerative colitis underwent a total colectomy with end ileostomy many years ago. She was subsequently diagnosed with primary sclerosing cholangitis, and she developed cirrhosis. She now presents with copious bleeding from her ileostomy with bright red blood and clots in the bag. She reports intermittent episodes of bright red bleeding into her ileostomy bag over the past 3 months. She is pale and diaphoretic. Her heart rate is 130 beats per minute, and her blood pressure is 80/30 mm Hg. On abdominal examination, she has a caput medusae and moderate ascites by percussion and bedside ultrasound. After successful resuscitation, she undergoes an upper endoscopy, which is negative for esophageal varices. What is the next best step in her management?
A Transjugular intrahepatic portosystemic shunt
B Stomal enteroscopy
C Laparotomy and resection of the terminal ileum
D Portacaval shunt
E Beta-blockade
A
This patient is most likely bleeding from her peristomal varices given the negative upper endoscopy. The patient should be aggressively resuscitated with blood products, and any underlying coagulopathy should be corrected. A temporizing measure to consider would be to apply traction to a balloon-tipped catheter inserted into her ileostomy to temporarily control the bleeding. Although endoscopic management is often a first-line therapy in esophageal variceal bleeding, enteroscopy through the stoma is not likely to be effective in managing bleeding from peristomal varices, because they arise at the mucocutaneous junction.
If transjugular intrahepatic portosystemic shunt (TIPS) is available, this intervention will significantly reduce the risk of current and future bleeding from peristomal varices. Operative intervention with portacaval shunt carries a significantly higher mortality in this case. Re-siting the ileostomy with or without ileum resection can be considered but should be performed only if TIPS is not available or other factors indicate a need for a new ileostomy site. Nonselective beta-blockers are useful in preventing initial variceal bleeds but are not indicated for a patient who is actively bleeding and in hemorrhagic shock.
An 18-year-old Afghan woman presents with upper abdominal fullness and pain. A chest x-ray (large structure in left lung with calcified walls) and CTs of her chest and abdomen are obtained (large thin walled structure with hyperintense layering posteriorly and two cysts in liver with heterogenous enhancement). What is her diagnosis?
A Echinococcal cysts B Amoebic cysts C Polycystic disease D Cystadenomas E Retained fragments/foreign bodies
A
Echinococcus is endemic in much of the underdeveloped world, including Afghanistan. Four different species of this tapeworm infect humans, with Echinococcus granulosis being the most common. The x-ray and CT scan demonstrate classic findings of a calcified cystic wall in the lungs and liver (figure 4). Amoebic cysts, polycystic disease, and cystadenomas do not typically have calcified walls. Retained fragments or foreign bodies would have more beam-hardening artifact scatter around the radiodense structures.
This patient underwent successful resection of her multifocal echinococcal disease in a staged fashion, starting with a left lower lobectomy, followed by a nonanatomic resection of her right upper lobe and excision of her hepatic cysts (video 1). During each procedure, the operative site was surrounded with laparotomy pads soaked in hypertonic saline, which is toxic to the parasite, and great care was taken during the cyst excision to avoid spilling the cyst contents. Spillage of cyst contents is associated with significant morbidity and mortality.
A 52-year-old man is referred for evaluation of progressive fatigue, 2.7-kg weight loss, jaundice, and pruritus. His medical history is significant for ulcerative colitis diagnosed at age 28, which was followed by a total proctocolectomy with ileoanal pouch anastomosis at age 46. After his operation, he has done well with 1 episode of “pouchitis” 8 months ago. This episode resolved promptly with conventional therapy. Physical examination is negative other than some mild scleral icterus. Laboratory serum abnormalities include the following: Alkaline phosphatase = 422 IU/L (<95 IU/L), serum glutamic oxaloacetic transaminase = 70 IU/L (<42 IU/L), and serum alanine transaminase = 65 IU/L (7–55 IU/L). The most likely study to confirm the diagnosis is
A cholescintigraphy (HIDA scan).
B transabdominal ultrasound.
C magnetic resonance cholangiography.
D upper gastrointestinal endoscopic ultrasound.
E serum antinuclear cytoplasmic antibody determination (p-ANCA).
C
Fatigue, weight loss, and pruritus are the classic presenting symptoms of primary sclerosing cholangitis (PSC). A history of ulcerative colitis, which is present in 75–80% of patients with PSC, strongly suggests the diagnosis. PSC has a slight male predilection and can involve both intra- and extrahepatic ducts. Confirmation of the diagnosis is by cholangiography, preferably magnetic resonance cholangiography, which has the advantage of being noninvasive and cost-effective and has a sensitivity of better than 90%.
Endoscopic retrograde cholangiography, which has a sensitivity approaching 100%, is currently reserved for equivocal finding by magnetic resonance cholangiography. Cholescintigraphy (HIDA scan) is not helpful in confirming the diagnosis, because this imaging modality lacks sufficient detail to show the classic “beads-on-a-string” anatomic configuration of the ducts. Percutaneous liver biopsy is not necessary for the diagnosis. Endoscopic or transabdominal ultrasound, while beneficial in the identification of choledocholithiasis or pancreatic pathology, is not recommended for suspected PSC, because it cannot evaluate the intrahepatic ductal system. The determination of serum antibodies (e.g., p-ANCA and ANA) is useful in suggesting autoimmune disorders but lacks the ability to differentiate the many autoimmune conditions, such as rheumatoid arthritis, Sjögren syndrome, and celiac disease.
The elevation of serum alkaline phosphatase is the primary laboratory abnormality in PSC patients. Most patients present with normal bilirubin levels; the marked elevation of bilirubin in this case is suggestive of advanced disease or malignancy. Patients with a history of ulcerative colitis have an increased risk of cholangiocarcinoma, and those with a history of “pouchitis” have an even greater risk. For patients with PSC, intact colons, and ulcerative colitis, the risk of colorectal cancer, preferentially the right colon, is increased by some reports 160-fold over normal individuals. The relationship between PSC and ulcerative colitis remains obscure, and theories include genetic (variants in HLA genes), geographical (prevalence in the United States and European countries 100-times greater than in Asia), bacterial toxins (colonic bacterial overgrowth with exaggerated immune response), and environmental factors (tobacco smoke and environmental toxins), or combinations of these.
Endoscopic stenting of the bile ducts, preferentially with self-expanding metallic stents, has some role in palliation in PSC. Some studies suggest that stenting of just a few of the stenotic ducts provides symptomatic relief in most cases. Sphincterotomy and cholecystectomy have no therapeutic or even palliative benefit. Corticosteroids and immunosuppressive agents are not of any value, either in providing symptomatic relief or in delaying progression of the disease. However, in patients with symptoms overlapping autoimmune hepatitis, corticosteroids may have a role.
The only effective therapy for end-stage PSC is liver transplantation. In fact, PSC is the most common indication for liver transplantation in Scandinavia and the fifth most common indication in the United States. Unfortunately, transplanted livers have a 20% chance of developing a recurrence of PSC, and this may ultimately lead to graft rejection.
A 35-year-old man presents to your office with complaints of discomfort and dysesthesia at the base of his penis after open mesh–based inguinal hernia repair. These symptoms are most consistent with the entrapment of which of the following nerves?
A Ilioinguinal nerve
B Lateral femoral cutaneous nerve
C Genital branch of the genitofemoral nerve
D Femoral branch of the genitofemoral nerve
E Obturator nerve
A
Chronic pain after inguinal hernia repair is reported to occur in 5–30% of patients. Although there is overlap in the different entrapment syndromes, detailed anatomic knowledge of the innervation is critical to avoid entrapment and assist in diagnosis and management of postherniorrhaphy neuralgia when it occurs.
The ilioinguinal nerve and iliohypogastric nerves have similar anatomic origins, courses, and sensory innervation. They provide sensory innervation to the proximal and medial thigh, base of the penis, and upper scrotum.
The lateral femoral cutaneous nerve should be well away from the operative field. Its sensory innervation is to the anterolateral thigh. The genital branch of the genitofemoral nerve provides sensation to the scrotum. The femoral branch of the genitofemoral nerve innervates the anterolateral thigh. The obturator nerve passes through the obturator canal well away from the operative field (figure 1).
The most consistent predictor of response to splenectomy for idiopathic thrombocytopenic (ITP) purpura is
A response to steroids. B younger age. C larger spleen size. D degree of thrombocytopenia. E duration of ITP.
B
Idiopathic thrombocytopenic purpura (ITP) occurs when autoantibodies against platelet glycoprotein complexes result in varying degrees of thrombocytopenia. The spleen is the predominant site of antibody-induced platelet sequestration and destruction. Splenectomy is advocated in adults with thrombocytopenia from ITP with reported long-term remission rates of 66–85% compared with much lower remission rates reported from medical management (<30%). The benefits of laparoscopic splenectomy reinvigorated the interest in splenectomy as a treatment for ITP.
Many studies examined factors that predict a sustained improvement in platelet counts after splenectomy. Although the degree of response to steroids and shorter duration of disease are positive predictors of response, the most consistent, independent predictor is younger age. There is no specific age cut off; the younger the patient is, the more likely the chance of complete remission. Lower platelet counts are a negative predictor. A few studies show that spleen size does not predict response. In general, patients with ITP have normal to slightly enlarged spleens. Splenomegaly is uncommon.
A 56-year-old man presents with fatigue, right upper quadrant abdominal pain and tenderness, and leukocytosis. Amoebic serology is negative. CT scan of the abdomen with intravenous contrast shows a 5-cm, round, fluid-filled lesion with an enhancing rim in the right lobe of the liver. Appropriate treatment includes
A antibiotics only. B antibiotics and percutaneous drainage. C needle aspiration. D right hepatectomy. E percutaneous drainage only.
B
Liver abscesses can be caused by fungus, bacteria, and amebae. Patients commonly present with constant, dull right upper quadrant abdominal pain, probably due to stretching of the liver capsule. Fever is common. Abscesses tend to occur in the right lobe, presumably due to the larger hepatic volume and predominant flow of blood from the superior mesenteric vein draining the gastrointestinal tract as opposed to the splenic vein. The presence of multiple abscesses suggests a bacterial or mixed source.
The treatment of liver abscesses has changed with the emergence of medical technology, particularly imaging and image-guided percutaneous interventions. A century ago, patients with multiple hepatic abscesses had a nearly universal mortality. Abscesses not amenable to percutaneous drainage may be successfully treated with antibiotics alone. In this case, the 5-cm fluid collection in the right lobe would be accessible by percutaneous means. The enhancing rim suggests characteristics of a pyogenic abscess. The negative amoebic serology eliminates amebae as the cause. Percutaneous drainage, with or without a catheter placement, improves the likelihood of treatment success. Occasionally, repeated aspirations are needed and can still result in treatment success. The morbidity of surgical drainage or even hepatectomy precludes its use.
In the Western world, bacterial hepatic abscess is most common. In developing countries, particularly southeast Asia and Africa, amebic infection is the most frequent cause. Amebic liver abscess is caused by the protozoa Entamoeba histolytica, and most can be treated with amebicidal drug therapy alone.
A 43-year-old woman with a history of 3 midline cesarean sections and diabetes presents with a 10-cm reducible hernia defect just below her umbilicus. She undergoes workup and is brought to the operating theater for an open repair of her defect. During the case, a small enterotomy is made. It results in minimal spillage and is repaired primarily. Which type of implant should be used to repair the hernia in this clinical scenario?
A Polyester B Polypropylene C Polyglactin D Polytetrafluoroethylene E Biologic
E
To assist with risk stratification of surgical site events and operative decision making, the Ventral Hernia Working Group (VHWG) developed a 4-tiered grading system (figure 1). VHWG recommends the use of synthetic mesh for low-risk hernias (i.e., grade 1) and bioprosthesis for potentially contaminated and infected hernias (i.e., grades 3 and 4, respectively). Biologic meshes placed in between two layers of vascularized tissue allow rapid host cell and vascular infiltration, which is believed to help them resist infection more effectively than synthetic meshes. In addition, biologic implants have high salvage rates in the setting of infection and enteric contamination.
In this patient with a recurrent incisional hernia and violation of the integrity of the small bowel (i.e., a VHWG grade 3 hernia), biologic mesh placement is the suggested therapy. Placement of synthetic mesh is not recommended. Polyester, polypropylene, and polytetrafluoroethylene are all permanent synthetic materials. Polyglactin is an absorbable synthetic material, which is prone to recurrent hernias.
A 32-year-old woman presents with a moderately symptomatic, reducible, left groin hernia. Which of the following statements is true regarding repair of groin hernias in women?
A The incidence of femoral hernias is the same regardless of sex.
B A traditional, open, tension-free, mesh repair (Lichtenstein) addresses both inguinal and femoral hernias.
C The increased incidence of femoral hernias may explain why female sex is a risk factor for hernia recurrence.
D Laparoscopic extraperitoneal herniorrhaphy is contraindicated in female patients
E A preoperative diagnosis of femoral hernia precludes the laparoscopic approach.
C
Inguinal herniorrhaphy is one of the most commonly performed surgical procedures today. Incidence of “groin hernias” is substantially higher in male patients (2–5% vs 0.3% in women). Although femoral hernias comprise only a small minority of groin hernias in men, they represent nearly one-third of the groin hernias in women. After open mesh repair, a “recurrence” in women is often seen in the femoral canal. The most likely explanation is that these are not recurrences but rather errors in diagnosis leading to the incorrect operation.
A traditional, open, tension-free, mesh repair (Lichtenstein) will address both the direct and indirect defects but does not cover the femoral canal. Both McVay and open preperitoneal repairs will address the femoral canal, but these are less commonly performed than open Lichtenstein procedures. A laparoscopic, extraperitoneal herniorrhaphy allows for exploration and coverage of the entire myopectineal orifice, addressing both inguinal and femoral hernias. In women, laparoscopic preperitoneal repair is associated with a lower recurrence rate than open mesh repair.
A 75-year-old man presents with new onset left groin bulge. He reports that he noticed the bulge 2 months ago and experienced no pain or other symptoms. He has normal bowel habits and no previous hernia surgery. He is otherwise healthy, except for hypertension. He lives alone and works as a hospital volunteer 3 days a week. In this patient, watchful waiting is
A contraindicated due to the acute nature of the hernia.
B contraindicated due to the presence of a bulge.
C contraindicated due to the age of the patient.
D at high risk of bowel obstruction.
E likely to fail due to progression of symptoms.
E
Management of asymptomatic or minimally symptomatic hernias is evolving as long-term randomized controlled study data for watchful waiting are reported. Since their original report in 2004 demonstrating safety of watchful waiting during short-term follow-up (median 3.2 years), Fitzgibbons et al. followed patients up to 11.5 years and found that the overall crossover rate to surgery is 68%. Men older than 65 are significantly more likely to crossover to surgery than are younger men (79% vs 62%). The most common reason for surgery was pain (54%). Only 2.4% of watchful waiting patients required emergency operation for complications such as strangulation or obstruction.
The authors concluded that watchful waiting is therefore safe, regardless of the timing of onset of the hernia, presence of a bulge, or age of the patient, but most patients will experience a progression of symptoms and ultimately need surgical repair. A similar study in the United Kingdom found that 72% of the watchful waiting cohort came to operative therapy at 7.5 years of follow-up.
Each of the following statements about cutaneous melanoma is true except:
A Regional LN status remains the most important prognostic indicator in early stage melanoma
B Mitotic index, ulceration and Breslow thickness are significant factors for predicting SLN mets
C Patients with melanomas of Breslow thickness of at least 0.75 mm or ulceration should be considered for SLNB
D Shave bx is a safe and accurate method for the initial evaluation of a skin lesion thought to be a melanoma
E A majority of completion LN dissections in patients with positive SLNB will yield additional positive nodes
E
Only 12% of patients with melanoma who had occult LSN positive were found to have additional positive LNs are completion lymphadenectomies
51F is undergoing lumpectomy for 0.9 cm grade II invasive ductal carcinoma. Her axillary exam is N. The proper management of the axillary LNs should consist of which of the following?
A. Observation
B. SLNB with permanent pathology
C. SLNB, frozen section and CLND if any nodes are positve
D. Partial breast irradiation
E. Axillary US and no further therapy if no abN nodes are seen
B
Patient has early stage breast cancer (T1-2, clinically node neg) and is undergoing BCT
Z011 study had similar patients which randomized pts who were found to have 1-2 positive SLNs to either no further axillary sx or ALND. Study showed no difference in 5 yr survival or DFS for the 2 groups
Omission of ALND may be safe b/c these patients receive axillary radiation as standard adjuvant therapy after lumpectomy. Avoid overextending the results in patient who had mastectomy, who do no receive axillary radiation for early stage breast cancer, and patients tx with partial breast irradiation who do not receive any dose to the axilla. For at least 3 positive nodes on permanent pathology, further axillar tx should be considered. Study also excluded whole breast radiation in prone b/c those patients do not receive any radiation to the low axilla
Compared with sporadic breast cancers, breast cancers in women who have a deletrious BRCA 1 gene mutation are more likely to be:
A. ER postive B. Her 2 positive C. Assoc with a better prognosis D. Lower grade E. Triple
E
Breast cancer in BRCA1 patients are more likely to be “basal” type tumors (ER -, PR -, HER-2 -) . These triple neg breast cancer have a particularly worse outcome compared with other breast cancers. B/c BRCA-1 related cancers tend to occur in younger women, this greater tendency for such cancers to be basal type explains in part the long observed worse prognosis of breast cancer in younger women
Conversely BRCA 2 mutation carrier tned to have similar histopathologic features as sporadic cancers. Accordingly, a meta analysis showed decreased OS for BRCA 1 assoc breast cancers vs no effect for BRCA 2 mutations on survival
Preop neoadjuvant chemo for locally advanced breast cancer
A. Significantly improves OS
B. Decreases breast conservation rates
C. Results in an overall pathologic complete response rate in the minority of patients
D. Uses different drugs and regimens than post op chemo
E. Has highest response rates for ER + tumors
C
Freq downsizing of tumors seen with upfront chemo allows increase use of BCT, resulting in improved cosmesis. B/c tumor response can be directly monitored, neoadjuvant therapy provides an in vivo test of the tumor’s sensitivity to treatment
Significant improvement in DFS and OS. Trends in improved survival were seen in NSABP B 18 and B 27 for women younger than 50 years. Further 13-26% of patient who achieved complete pathologic response had improved survival compared with those who did not have a pathologic complete response. Seen more commonly in patients who have neg prognostic factors: <40 yrs old, high grade tumors, Her2 positive, and ER -]
Most chemo drugs and regimens approved for post op can be used pre op (NSABP used doxorubicin, cyclophosphamide, and docetaxel).
52F referred with a 2 cm mass in the region of the axilla and a normal breast exam. Her family hx reveals a paternal aunt with breast cancer dx at age 34. Which of the following is the most likely etiology of the mass?
A. Lymphoma B. Melanoma C. Breast cancer D. Gastric cancer E. Reactive lymphadenopathy
E
Isolated axillary adenopathy is a concerning physical finding requiring complete evaluation. Fortunately, it is assoc with malignancy in only a minority of cases. When subjected to bx, nonspecific inflammatory changes or findings consistent with granuloma are typically identified. Malignancy accounts for <25%, lymphoma accounting for 14% and metastatic carcinoma for 10%
Work up: Complete H&P including breast exam and skin exam. If >35 yrs then mammogram. US is reliable for differentiating benign and malignant lymphadenopathy and useful in the management of patients with unilateral axillary adenopathy with otherwise N findings. US directed DNA should be considered for LNs with abN morphology. Asymp pts with normal appearing LNs on US are managed with a period of watchful waiting.
63F with a breast mass undergoes a lumpectomy and a SLNB. Final path shows tumor is a 1.0 cm invasive lobular breast cancer, ER +, Her2 +. Which of the following regimens would be the best chemo for this patient?
A. Cyclophosphamide, methotrexate and 5 FU (CMF)
B. Doxorubicin, cyclophosphamide, paclitaxel, trastuxumab (AC-TH)
C. Doxorubicin, cyclophosphamide, plus paclitaxel (AC-T)
D. Doxorubicin, cyclophosphamide (AC)
E. Docetaxal, cyclophosphamide (TC)
B
Trastuzumab is highly effect for breast cancer patients with HER 2 neu amplified tumors; it reduced the risk of death by at least 39% in several prospective randomized trials.
72F scheduled to undergo BCT and SLNB. She has a hx of HTN that is well controlled with single medications. Otherwise has no hx of cardiac or resp problems. She is able to carry out her ADLs without assistance (ECOG 0 and Karnofsky 100). Her physical exam is unremarkable, and her EKG shows an occasional PVC. Which of the following should be included in the pre op cardiac evaluation?
A An echocardiogram B Traditional exercise stress testing C Coronary angiography D Dipyridamole myocardial stress perfusion imaging E No further cardiac evaluation
E
Regarding women known to be at increased risk of breast cancer development, which of the following statements is TRUE?
A. BSO decreases breast cancer risk by 25%
B. Bilateral ppx mastectomy eliminates breast cancer risk
C. Tamoxifen decreases breast cancer risk by 50%
D. Annual MRI is recommended if lifetime risk is >15%
E. Anastrozole decreases risk of breast cancer by 50%s
C
BSO decreases breast cancer risk by 50%. Bilat ppx mastectomy decreased breast cancer risk by 90-95% by removing the majority of the breast tissue. It does not completely eliminate breast cancer risk. Tamoxifen decreases breast cancer risk by 50% and is an effective chemoprovention agent.
Annual MRI for screeing is recommended for women whose lifetime risk of breast cancer is 20-25% or higher.
Anastrozole has not been evaluated for chemoprevention, and its use is limited to post menopausal patients with breast cancer
63F undergoes a lumpectomy for a 1.2 cm invasive ductal carcinoma of the breast resected to neg margins. One of 2 sentinal LNs is found to be positive. Which of the following is the most appropriate initial management option?
A. No further axillary surgery, whole breast radiation
B. No further axillary surgery, partial breast radiation
C. Axillary dissection, whole breast radiation
D. Axillary dissection, partial breast radiation
E. Mastectomy, no further axillary surgery
A
Z011 evaluated women with T1 or T2 tumors who were clinically node negative at presentation undergoing BCS, who were found to have 1 or 2 positive SLNs and who were receiving whole breast radiation. Pts with at least 3 postive SLNs. with gross extranodal extension, receiving noeadjuvant therapy, undergoing mastectomy or not receiving whole breast radiation were not eligible.
No difference in in-breast recurrence, axillary recurrence, 5 yr DFS or 5 yr OS between women undergoing ALND and those women who did not undergo ALND. Most women in the study were postmenopausal with low grade, ER +, and 96% received adjuvant systemic therapy.
Women who meet Z11 criteria and have 1 or 2 positive SLNs now can be spared Ax dissection. Whole breast radiation is recommended for these patients and these data should not yet be extrapolated to pts undergoing partial breast radiation, no radiation, neoadj endocrine or chemo, mastectomy or 3 or more postive LNs
ALND can be avoided for patients who meet all of the following criteria:
- Breast conservation
- Whole breast radiation (not partial breast radiation)
- Clinical T1 or T2 and N0
- No neoadjuvant chemo
- 1 or 2 postive SLNs
Regarding lymphatic mapping for breast cancer, which of the following statements is TRUE?
A. Blue dye alone is recommended for mapping
B. Isosulfan blue can interfere with pulse oximetry readings
C. Peritumoral injections provide the quickest drainage
D. Internal mammary nodes seen on lymphoscintigraphy should be resected
E. SLNB is not recommended for multicentric breast cancer
B
Highest SLN identification rates and lowest false neg rates have been reported with the use of dual mapping agents: radioactive colloid and blue dye.
LIfe threatening anaphylaxis have been reported in 1% of cases with use of isosulfan blue. It can also interfere with pulse oximetry, resulting in falsely lowered peripheral oxygen saturation readings.
Subareolar injection of mapping agents results in the fastest drainage to the ipsilateral axillary LNs and can be used for many breast cancer cases; however for patients with recurrent breast cancer, peritumoral injection is recommended.
Detection of lymphatic drainage to internal mammary LNs allows the option for resection; however, this finding remains controversial and for primary breast cancer, most surgeons do not resect internal mammary LNs as long as an axillary SLN is identified. SLN surgery is used in male breast cancer, multifocal breast cancer, multicentric breast cancer and patients after neoadj chemo with clinically neg nodes at presentation
Which feature is associated with increased local recurrence after breast conservation?
A. Partial breast radiation B. Invasive lobular carcinoma C. Estrogen receptor positivity D. Multifocal disease E. Young age
E
7 prognostic factors for local recurrence: patient age, tumor size, tumor grade, margin status, LVI, chemo use and hormone therapy use
Incr risk of local recurrence after breast conservation include those with a young age at dx (< 40 yrs). Young age is a poor prognostic factors for breast cancer. Incr age is assoc with decr risk of local recurrence independent of breast cancer subtype.
Local recurrrence is higher among HER2 postive and triple neg. Lower recurrence in ER+ breast cancer.
Which of the following statements is TRUE regarding atypical lobular hyperplasia?
A. Breast cancer risk is increased 8 fold
B. Tamoxifen decreases the risk of breast cancer by 1/3
C. An annual MRI is recommended
D. Lumpectomy and SLN staging are recommended
E. The risk of breast cancer increases with multifocality
E
ALH is a factor for incr risk of breast ca development (4x). Not thought to be a precursor lesion to breast ca. # of foci correlates with risk–multifocal atypia have higher risk than single focus of atypia
ALH on core needle bx–> consider surgical excision especially if pathologic-radiologic discordance or any residual imaging abN. Ax nodal staging not recommended.
Chemoprevention with tamoxifen, raloxifene and exemestane is recomneded for consideration for women with atypia. Tamoxifen decr risk of breast cancer by 50% or more.
Regarding a 55F with 2 cm intermediate grade DCIS who undergoes surgical excision, which of the following statements is TRUE?
A. Aromatase inhibitor is recommended for adjuvant therapy
B. Tamoxifen is recommended after bilateral mastectomy
C. Postmastectomy radiation is recommended if invasive disease is identified
D. SLNB at time of lumpectomy is recommended
E. Adjuvant breast radiation is recommended after lumpectomy
E
Adj radiation decr local recurrence rates after lumpectomy for pts with invasive breast cancer and DCIS.
Post mastectomy radiation is recommended for pts with invasive disease >5 cm, chest wall invasion, persistent positive margins, or postive LNs (clasically >4 LNs and increasingly also for 1-3 + LNs). Not recommended if invasive disease <5 cm, neg margins and neg LNs.
SLNB for DCIS tx with BCS is not recommended. SLN sx for axillary staging should be used only for cases of DCIS that are > 5cm in size, palpable, assoc with microinvasion or tx with mastectomy.
Adj therapy for DCIS consists of tamoxifen and is used to decr the risk of future breast cancer event, most commonly recurrence or new primary in the ipsilateral or contralateral breast. AIs have not been studied in women with DCIS. Women with DCIS tx with bilat mastectomy are not recommended to receive adjuvant endocrine therapy.
Which of the following conditions is suitable for consideration of partial breast radiation?
A. Young age B. LVI C. Tumor > 3 cm D. ER negative E. Medial quadrant lesion
E
Partial breast radiation delivers radiation to the lumpectomy bed and surrounding breast tissue.
After lumpectomy, in breast recurrence can be classified as tumor bed recurrences (near lumpectomy cavity) and elsewhere recurrences. Majority after BCT and whole breast radiation are in the region of the tumor bed
Partial breast radiation allows radiation to be delivered over a shorter time course. Suitable candidates are women > 45 y.o. with invasive ductal carcinoma, tumore < 3 cm, neg margins, node neg. Location within the breast is not a limiting factor regarding use of partial breast radiation.
Which of the following patients is recommended to undergo annual screening MRI for breast cancer detection?
A. Prior breast augmentation B. Lifetime risk of breast ca of 14% C. Prior breast cancer tx with lumpectomy and radiation D. Personal hx of LCIS E. Previous therapeutic mantel radiation
E
ACS recommends annual MRI screening for women with a lifetime risk of breast ca of at least 20-25%, known BRCA 1 or 2 mutation carriers, untested 1st degree relative of known BRCA 1 or 2 mutation carriers.
Based on expert consensus opinion, annual MRI screening was recommended for patients with radiation to the chest between 10 and 30 yrs, Li Fraumeni syndrome or Cowden syndrome.
Diagnostic MRI in patient with axillary mets thought to be of breast origin and mammogram and ultrasound do not identify the primary tumor. Useful to reconcile differences between clinical findings on exam and mammo or US. Response to neoadj chemo can be evaluated on MRI.
MRi can be considered in women with dense breast tissue, invasive lobular cancer or women presenting with mammographically occult disease.
Which of the following is the most effective risk reduction strategy for woman at high risk for breast cancer?
A. BSO B. Bilateral mastectoy C. Tamoxifen D. Annual MRI E. Anastrozole
B
Women identified to be at elevated risk of breast ca should be counseled regarding their risk reduction options. Chemoprevention with meds such as tamoxifen, raloxifene or ezemestane can decr risk of breast cancer development by 50%. BSO results in risk reduction of breast cancer by 50% and decreases risk of ovarian ca by 90%.
In women with BRCA 1 or 2, bilat ppx mastectomy significantly reduces the risk of breast ca and BSO is assoc with a significant decr in ovarian cancer, risk of breast cancer, all cause mortality, breast cancer specific mortality, and ovarian cancer specific mortality. Cancer reducing benefits observed are the greatest when the procedures are performed before age 50
44y.o. premenopasal woman has R upper outer quadrant breast calcifications noted on her yearly screening mammogram. No Famhx of breast cancer. Menarche at 14 yrs and delivered her first child at age 26. No prior breast bxs. Stereotactice core needle bx reveals LCIS. Which of the following is the next most appropriate step?
A. Needle localized excisional bx B. Ppx bilat mastectomy C. Raloxifene tx for 5 yrs D. Mirror image bx of L breast E. Annual mammo
A
LCIS is typically dx in women in their 40s, full decade earlier than most women with DCIS. LCIS is considered a risk indicator for the development of invasive breast cancer, rather than a precursor lesion. When LCIS is dx on sterotactic core bx, operative excisional bx is indicated to exclude an adjacent invasive breast cancer. Surgical excisional bx has a 20-30% incidence of a malignant lesion near LCIS. If invasive breast cancer is not identified on final pathology, patient is followed with annual mammo and physical exam.
If needle localized excisional breast bx has LCIS at the margin, repeat excision is not indicated.
PPX bilat mastectomy considered if strong fam hx or BRCA 1 or 2 mutation. Premenopausal considered at high risk of developing invasive breast cancer may be candidates for chemoprevention with tamoxifen, which can decrease risk by almost 50%. Must weigh risks and benefits of 5 yrs of tamoxifen therapy including thromboembolic disease and uterine malignancy. LCIS increases risk 3x for ipsilateral breast compared to contralateral.
22F presents with 4 day hx of breast pain. On exam, she has R breast tenderness, localized erythema and a fluctuant 3 cm mass in upper inner quadrant. US shows breast abscess. In addition to abx, which of the following initial tx options is most appropriate?
A. I &D B. US guided aspiration C. US guided drain placement D. Incisional bx E. Excision of mass
B
Recent studies sow that needle aspiration and ABX can be used as 1st line tx for simple breast abscesses. I&D reserved for pts who do not resolve with repeated aspirations. Placement of drain catheter is considered for abscesses > 5 cm.
Fewer than 5% of pts with a breast abscess have an assoc malignancy. Pts should undergo repeat imaging ~6 wks after clinical resolution of the infection. If there is residual noninflammatory mass, a core bx should be performed to evaluate for malignancy. If cellulitis and assoc breast mass do not improve with abx, malignancy should be suspected and bx should be performed. Inflammatory cancer typically occurs at an older age and presents with erythema and edema of 1/3 of the breast or more. Although the dx of inflammatory breast cancer remains clinical, a punch bx of the skin showing invasion of dermal lymphatics can assist in confirming the dx
60 y.o. postmenopausal woman has L breast microcalcifications noted on her yearly screening mammo. She has no fam hx of breast or ovarian cancer. Stereotactic core needle bx reveals atypical ductal hyperplasia. Which of the following is the next most appropriate step?
A. Simple mastectomy B. Lumpectomy with SLNB C. Tamoxifen tx for 5 yrs D. Needle localized excisional bx E. Annual mammography
D
Sterotactic bx is used for dx of nonpalpable masses or microcalcifications noted on mammo. If ADH is identified on pathologic review, excisional bx is indicated to exclude invasive carcinoma; an image guided core bx underestimates the presence of DCIS or invasive cancer in 10-20%. If, after excisional bx, there is no additional DCIS or invasive cancer that requires tx, patient may consider further chemoprevention with tamoxifen and will require annual mammography for screening.
32F referred for 3 days of tender cord in L breast. On physical exam, inspection demonstrates a thin cord on lower aspect of breast. Which of the following is the next step in management
A. Reassurance and observation B. Mammography C. U/S D. Punch skin bx E. Excisional bx
A
Mondor disease is a superficial thrombophlebitis of the subcutaneous veins. Most common site is anterolateral thoracoabdominal wall, including the breast, other sites include the antecubital fossa, penis and posterior cervical region. Painless, cord like induration. Cause unknown although most commonly assoc with trauma. Easily dx by physical exam. Occasionally, mammo and U/S may be used if there is suspicion of underlying breast pathology, but neither is routinely required. Bx, either punch or excisional is rarely indicated. Tx is reassurance b/c this is self limiting. More aggressive tx is required only for symptoms, which are generally focused on symptom relief
65F underwent lumpectomy with SLNB for a T1b ER and PR + ductal carcinoma. Pathologic evaluation of LN wiht hematoxylin and eosin demonstrated a 0.1 mm focus of mets. Which of the following is true?
A. A completion ALND is required B. Adjuvant hormonal therapy is required C. Patient's OS is less than half compared with a node neg patient D. Axillary radiotherapy is required E. Local recurrence risk is increased
B
Micromets as a cluster of cancer cells 0.2-2.0 mm in size. Isolated tumor cells <0.2 mm.
Completion ALND is not required. Axillary radiotherapy is not required b/c risk of axillary recurrence is low. Local recurrence risk is not increased. No difference in survival.
55F presents to the office with erythema of the R breast. After a 2 wk course of oral Abx, the erythema is still present. and R breast mammography reveals diffusely increased density. U/S shows no fluid component. Which of the following would be the most appropriate next step?
A. Continue Abx for 2 more wks B. Change to a broader spectrum abx C. Obtain a MRI D. Perform a R breast core biopsy E. Perform an I&D of the R breast
D
Even though the most common cause of a red breast is infection, it can represent an inflammatory breast cancer. Inflammatory breast cancer is not common, ~1-5% of all breast cancers. Dx with mammogram and U/S early, whether or not there is a palpable breast mass or axillary adenopathy. If there is a palpable mass on exam or a radiographic abN, an incisional bx of the mass with attached/involved skin should be performed. If U/S suggests an abscess, drainage should occur and the pt should be placed on abx. If no abcess is seen but skin erythema is subtle, a percutaneous core bx of the underlying lesion should be performed. 5 yr survival ~40%, 20-35% will have distant mets at the time of presentation, 60-85% will have nodal mets.
Woman presents with a core needle bx dx of DCIS. In which of the following situations is SLNB indicated?
A. In conjunction with needle localized excision B. Mammographic breast lesion C. Presence of comedonecrosis D. Age < 50 yrs E. Grade 0 DCIS
C
Differentiating DCIS from invasive ductal cancer with core needle bx affects the extent of surgery. Routine SLNB in all patients with DCIS on core needle bx is not recommended due to low risk of axillary LN mets. However, CNB may underestimate the presence of invasion and a final pathologic dx of invasive cancer then necessitates reoperation to stage the axilla. ~20% pts dx with DCIS on CNB are upstages after surgical excision. Selective use of CNB in pts with DCIS on CNB is recommended based on predictors of an invasive component. Strong predictors include presence of a palpable mass, microinvasion, and lesion of at least 2 cm on imaging. Additional confirmed predictors include high nuclear grade and presence of comedonecrosis, which are assoc with invasive cancer in 28% and 44% of patients.
47F with 2 yr hx of a 8 cm mass occupying the majority of the R breast. Although there is no skin breakdown, the breast is blue at the apex of the mass, and the nipple is enlarged and excoriated. No palpable adenopathy is present in the axilla. Core bx reveals mixed epithelial stromal proliferation. Which of the following is the next step in her managment?
A. Chemoradiation B. Punch bx of the overlying skin C. Modified radical mastectomy D. Simple mastectomy E. Lumpectomy with SLNB
D
Phyllodes tumors are fibroepithelial tumors of the breast. Represent 1% of all breast tumors and most commonly affect woman 35-55 yrs of age. Dx is challenging b/x the clinical presentation is similar to fibroadenoma. Overlying skin may be discolored with dilated veins but skin invasion and nipple retraction are rare. Axillary dissection is not indicated b/c of low incidence of LN involvement. Std tx of phyllodes is surgical excision with a clear margin, regardless of tumor grade. Mastectomy may be necessary for larger tumors if they occupt the majority of the breast, b/c breast conservation with an acceptable cosmetic result is unlikely to be achieved.
Which of the following regarding lymphedema after surgery for breast cancer is TRUE?
A. It is not exacerbated by physical activity
B. It occurs rarely and seldom affects quality of life
C. It does not occur with SLNB
D. Laser therapy provides no tx benefit
E. Wt loss does not reduce upper extremity lymphedema
A
Common and debilitating side effect of breast cancer tx. May occur after SLNB if SLN is located at the level of the axillary vein or combined with axillary radiotherapy. ~7% with SLNB. If tx conservatively in the earliest stages, complications may be diminished or reversed. Intensive therapy such as PT, manual drainage, or pneumatic pump, yield greater volume reduction than compression garments or limb elevation. Low level laser therapy increases rate of lymph vessel pumping and promotes lymph vessel regeneration, reduced pain and softens fibrous tissue and surgical scarring. Exercise has been shown to significantly reduce severity of symptoms. Wt loss is shown to reduce upper extremity edema. Some studies have initiated therapy as early as 48 hrs after sx.
75F with unilateral, blood nipple d/c, which of the following is the msot common etiology?
A. Phyllodes B. Paget disease C. LCIS D. Invasive ductal carcinoma E. Intraductal papilloma
E
Determine if nipple d/c is unilateral or bilateral and whether fluid is grossly bloody or contains blood. Galactorrhea is used to describe milky d/c from both breasts
Any discharge not considered to be galactorrhea is caused by a ductal condition such as ductal ectasia, fibrocystic breast changes, intraductal papilloma, intraductal carcinoma and invasive ductal carcinoma (usually papillary type).
Duct ectasia is characterized by diln of the major ducts along with inflammation and fibrosis around the ducts. D/C is usually green or black and guiac neg. Sx therpay is not necessary in women with classive multiduct non blood black green d/c. If only 1 duct is involved, therapy is usually excision to help dx the etiology.
Fibrocystic disease typically results in serous or light green multiduct d/c. Complain of cyclic breast pain and premenstrual breast lumpiness, and exam reveals a diffuse find nodularity. Mammo and US are confirmatory and show dense breast tissue with nodularity and microcyst formation
Most common cause of bloody uniduct d/c is intraductal papilloma. D/C is usually spontaneous but can be elicited by palpation and comes from a single duct orifice. Mammo is typically neg and US may show a dilated duct with an intraluminal lesion. Most intraductal papillomas occur near the areolar edge and galactography is recommended to identify more peripheral lesions. Galactography involves cannulation of the duct orifice and injection of radiopaque dye; papillmas are seen as intraluminal filling defects. Tx is duct excision
Intraductal and invasive ductal carcinoma are only rarely assoc with nipple d/c in the absence of an abN mammo or palpable mass. DCIS and papillary carcinoma cause most cancer associated nipple d/c. Occasionally, blood d/c is present in Paget’s disease–a scaly, raw, vesicular or ulcerated lesion that beings on the nipple and spreads to the areola. LCIS and phyllodes are not assoc with nipple d/c
Which of the following is NOT an indication for testing patients for hereditary breast and ovarian cancer syndrome?
A. Breast ca dx at or before 45 y.o B. Breast ca dx in male pt at any age C. Breast ca at any age in a woman of Ashkenazi Jewish origin D. Fallopian tube cancer at any age E. Synchronous breast cancers at any age
E
Incr risk of breast and ovarian cancer in ppl who have inherited a deletrious mutation of BRCA1 or 2 genes. 10 fold relative risk of these cancers and incr risk more than any other factor. Female BRCA1 carriers have a lifetime risk of developing breast ca and ovarian/fallopian tube ca of approx 40-80% and 26-46%. For BRCA 2 mutations, risk is 30-60% and 10-20%
Same specific BRCA mutation ends to be conserved w/in a given BRCA family, it is most efficient to test an affected patient (index relative) to determine whether a mutation exists and if a mutation is found, to then test other family members for the same mutation. Guidelines include 4 categories to determine which pts should be tested
- Families with known BRCA 1/2 mutations
- Personal hx of cancer and at least 1 of several other risk factors incl dx before age 45, ashkenazi jewish, and 2 breast cancers when the first was dx before age 50, people with certain cancers (male breast cancer, ovarian/fallopian tube cancers), and certain high risk family hx
All of the following are considered accepted indications for breast MRI except
A. Evaluation of unknown primary breast cancer pts with clinical axillary mets
B. Screening in place of mammo in high risk (BRCA1 and BRCA 2) patients
C. Evaluation of contralateral occult disease in lobular cancer
D. Evaluation of indeterminate lesions seen on mammo and US inlieu of short term follow up
E. Evaluation of response to neoadjuvant therapy to determine candidacy for BCS
B
Breast MRI is appropriate in pts with nodal disease and occult primary cancers, those with multifocal or multicentric tumors and in assoc with conventional imaging for difficult primaries, such as invasive lobular carcinoma. Indicated to assess response to neoadj chemo in selected pts. Use is appropriate in addition to mammo for patients at very high risk for breast cancer, especially those with gene mutations such as BRCA 1 and 2 but it is not indicated to take the place of screening mammo
In patients with clinically involved axillary nodes without any identifiable primary occurs in ~1% of all dx breast cancers. After mammo and U/S, MRI may identify primary tumors in up to 50% of these patients.
The following are true about breast abscesses except
A. Staph aureus is the most common pathogen
B. daily needle aspiration is successful in lactating patients with abscesses < 5 cm in diameter
C. U/S guided aspiration can facilitate complete abscess drainage
D. Abscesses with thick rinds and septa may require surgical incision
E. Breast abscesses are uncommon in nonlactating women
E
Most common abscesses grow staph aureus and nearly 25% of abscesses occur in non lactating women. Breast abscesses <5cm can be tx successfully with U/S localization with repeated aspirations as required, leading to improved cosmesis at the drain site. US can distinguish abcesses from mastitis and US can facilitate complete drainage of abscesses including loculations. A thick abscess rind assoc with septa may indicate the need for surgical drainage in pts with larger abscesses
For each statement, fill in the letter that is associated with the statement
A. Periductal mastitis
B. Idiopathic granulomatous mastitis
C. Both
D. Neither
- Often responds to steroids
- Related to smoking
- Abscesses
B
A
C
Idiopathic granulomatous mastitis is a rare condition characterized histologically by noncaseating granulomas centered on lobules. Both can be associated with fistulas and abscesses. However, unlike periductal mastitis, idiopathic granulomatous mastitis is not assoc with smoking, occurs in younger women, and is often assoc with recent pregnancy. Patients with IGM are less likely to have used OCPs previously, to be white or respond to classic tx such as surgical drainage and abx. IGM is effectively tx with a variery of therapies including steroids, which may treat an underlying autoimmune casue. Aggressive surgical mgmt is rarely reqd. Regardless of tx, IGM often burns out spontaneously in 6-12 months. For both periductal mastitis and IGM, bx and close observation are critical to rule out inflammatory breast cancer
For each statement, fill in the letter that is associated with the statement
A. LCIS
B. DCIS
C. Both
D. Neither
- Requires excision with clear margins
- Radiation reduces the risk of local recurrence by one half
- Selective estrogen receptor modulators (e.g. raloxifene, tamoxifen) reduce the risk of invasive breast cancer
B
B
C
DCIS is most commonly detected as suspicious microcalcs on routine screening mammo in an asymp woman. B/c most women with newly dx DCIS are eligible for BCT, a major decision is whether to add radiation after surgical excision. Addition of radiation after lumpectomy, reduces the risk of local recurrence by approx 50% for both overall local recurrence and subset of invasive local recurrence.
Decision to excise isolated LCIS when found on core needle bx should depend on multiple factorsm including the imaging indication and findings, extent of LCIS on CNB and presence of concurrent atypical ductal hyperplasia on CNB. Proposed tx algorithm allows for close follow up for patients with calcs on routine mammo and isolated LCIS on CNB that meet the following criteria: 1) Normal risk pt undergoing routine screening mammo found to have calcs, 2) fewer than 4 foci of LCIS on CNB, 3) no other high risk lesion present. Staining for e cadherin can be used to differentiate ductal vs lobular carcinoma in situ b/c membranous staining is seen in most DCIS and neg in lobular neoplasia.
Pts with hx of LCIS have a 4-10 x increased risk of subsequent invasive disease. Tamoxifen and raloxifene are equally effective in reducing this risk.
For each statement, fill in the letter that is associated with the statement
A. Isosulfan blue
B. Technetium-labeled sulfur colloid
C. Both
D. Neither
- IV dosing best
- Pregnancy category C agent (possible teratogen)
- Can be injected up to 24 hrs before surgical procedure
D
A
B
SLNB is the preferred method of axillary staging for pts with breast cancer. 2 most common agents are isosulfan blue and technetium labeled sulfur colloid. Neither offers clear advantage. For most patients, they may be successfully used independently or in combo. Isosulfan blue is typically injected into breast 5-10 min before making the axillary incision. Technetium labeled sulfur colloid can be injected on the day before sx or as little as 20 min before sx.
SLNB is the method of choice for axillary staging in pts dx with breast cancer in pregnancy. Isosulfan blue is classified as category C and should not be used for SLNB in pregnant pts. Several studies have collected dosimetry measurements on nonpregnant wmoen during SLNB using technetium labelled sulfur colloid. Potential radiation dose to fetus appears to be minimal and should be safe. No complications in mother or fetus were reported.
For each statement, fill in the letter that is associated with the statement
A. Classic LCIS
B. Pleomorphic LCIS
C. Both
D. Neither
- Surgical excision recommended
- Marker for increased risk of breast cancer in either breast
- Assoc with invasive lobular carcinoma
C
C
C
LCIS is considered to be a marker for incr risk of breast cancer. 8x incr risk of lifetime development of breast cancer and this risk is for both breasts, regardless of which side the LCIS was identified. Pathologically, LCIS can be differentiated into classic LCIS and pleomorphic LCIS. Pleomorphic shows a more aggressive nature based on a higher proliferative index and immunohistochemical markers. LCIS is often an incidental finding. Excisional bx to r/o a coexistent malignancy is recommended. Neg margins are not required at time of surgical excision of classic LCIS; however, they are required for pleomorphic LCIS, which behaves more like DCIS. SLNB for nodal staging is not required at the time of surgical excision of LCIS and is reserved for cases with invasive disease. Adjuvant radiation is not required for classic or pleomorphic LCIS. Both classic and pleomorphic are assoc with invasive lobular carcinoma although invasive ductal carcinomas also occur in these patients.
For each statement, fill in the letter that is associated with the statement
A. Repeat mammo in 1 yr B. Repeat mammo in 6 months C. Additional breast imaging needed D. Biopsy needed E. Excision of mass required even if needle bx is neg for malignancy
- Birads 0
- Birads 1
- Birads 2
- Birads 3
- Birads 4
C A A B D
Birads 0 Incomplete
Birads 1 Neg
Birads 2 Benign findings
Birads 3 Probably benign–initial short interval follow up suggested
Birads 4 Suspicious abN–bx should be considered
Birads 5 Highly suggestive of malignancy–appropriate action should be taken
Birads 6 Known biopsy–proven malignancy–appropriate action should be taken
Biopsy is not required for lesions classified as birads 3. Bx is required for birads 4 lesions, and appropriate action should be take as determined by bx results. Birads 5 lesions found to be benign on core bx are considered “benign discordant” and should be excised
For each statement, fill in the letter that is associated with the statement
A. Atypical Ductal Hyperplasia B Intraductal Papilloma C Flat epithelial atypia D Papilloma with atypia E Sclerosing adenosis
1 Associated with 4x increased lifetime risk for breast cancer
2 Recommend use of chemoprevention
3 Highest risk of malignancy at surgical excision
A
A
D
ADH and ALH are both assoc with a 4x incr lifetime risk of breast cancer. Discussion regarding the use of chemoprevention is recommended to decr the risk of breast ca. Risk of breast ca is assoc with the number of foci of atypia ; breast ca risk incr with multi focal atypia
FEA and papillomas with atypia are recommended for surgical excisional bx to r/o malignancy. If ADH or ALH is identified, chemoprevention should be considered; however FEA alone is not an indication for chemoprevention
Papillomas with atypia have the highest risk of concurrent malignancy identified on excision, with reported rates >20%. Sclerosing adenosis is a benign finding that does not require surgical excision unless the radiologic and pathologic findings are discordant. Intraductal papilloma is the most common cause of pathologic nipple d/c and should be surgically excised to r/o concomitant malignancy.
For each statement, fill in the letter that is associated with the statement
A Cyclical mastalgia B Noncyclical mastalgia C Costochondritis D Cervical radiculopathy E Breast cancer
- Point tenderness with palpation
- Responsiveness to caffeine abstinence
- Shoulder pain
- Most commonly related to menopause
C
A
D
B
Cyclical mastalgia is intermittent breast pain, sometimes quite severe, occurring in 1 or both breasts, generally in the perimenstrual period. Generally involved in a wide area of breast but it is isolated in the breast tissue, not the skin, muscle or chest wall.
Noncyclical mastalgia is constant breast pain, not assoc with the menstrual cycle. Pain is less common than cyclical mastalgia and usually involves point or localized tenderness in 1 breast. Effects of estrogen on Noncyclical mastalgia are not as clear as with cyclical mastalgia. More commonly believed that anatomic rather than hormonal factors are the cause. However, menopause is a common inciting event.
Evaluation of mastalgia begins with H&P. In patients of appropriate age, mammo may be necessary. If palp lesions, diagnostic mammo or U/S can assess for solid or cystic masses. Tx includes couseling to proper fitting bra, relaxation training, caffeine and tobacco abstinence, use of vit E, evening primrose oil and soy. For severe cases, adjusting of OCPs, postmenopausal estrogen replacement, Danazol, gestrinone and even tamoxifen
Costochondritis is inflam of the cartilage at the costochondral or costosternal joints from ribs 2-5. Tietze syndrome is Costochondritis with assoc swelling of the joints. Physical exam will demonstrate point tenderness along the costochondral or costosternal joints. Tx with reassurance and NSAIDs.
Cervial radiculopathy with compression of nerve roots primarily at C6 and C7 can be a cause of pain in the region of the breast. Not true breast pain but rather a referred pain and is commonly assoc with shoulder pain. Tx should be directed at nerve root compression.
Which of the following has the best combo of pt satisfaction and proper prep regarding colonoscopy?
A. PM scope and same day 4 L PEG single dose prep
B. PM scope and same day, split dose 2 L prep
C. AM scope with prior day, 4L single dose prep
D. AM scope with 2 day, split dose 2 L prep
E. PM scope with 2 day, split dose 2 L prep
B
Recent study in American Journal of Gastroenterology compared morning only PEG solution with a split approach (1L 7 hrs before and 1L 4 hrs before scope) for afternoon colonoscopy. Both preps had equal clinical efficacy with respect to cleansing and polyp detection., Morning only prep was assoc with lower incidence of abdo pain, superior sleep quality and less interference with workday before scope.
Greatest pt satisfaction is observed with a morning prep and an afternoon scope on the same day. Most effective cleansing is a split dose, low dose prep.
54F with a hysterectomy 3 yrs ago for benign menorrhagia presents with abdo dissension, vomiting and obstipation for 24 hrs. Her WBC and lactate are N and there is no peritonitis. CT shows a PSBO with a TP. Which of the following is true?
A. She has <10% chance that this condition will recur
B. She will likely not require an OR on this admission
C. Decompression with a long tube is indicated
D. Most likely cause is intussusception
E. IV abx are indicated
B
Adhesive SBO is the most common cause of intestinal obstruction in pts who have undergone prev abdo ORs. During 1st episode, 70-80% will resolve with medical mgmt and therefore are not likely to require OR. RCT of long intestinal vs NG tubes showed no difference in success of non op mgmt. IV abx not necessary in SBO. Intussusception is an uncommon cause of SBO in adults and if present would require an OR b/c it is assoc with a malignant pathologic lead point in 30-50% of cases. Despite successful non op mgmt, approx 30% who have had an episode of adhesive obstruction will recur and recurrent episodes can be an indication for operation.
84 y.o. Nursing home pt presents to ER with acute abdo pain, N/V. His HR = 90, bp 130/70 and he has a distended, tympanic abdo without peritonitis. His WBC is 11 and his AXR showed a coffee bean sign. Which of the following is the most appropriate step?
A. Exp lap B. Sigmoidoscopy C. Cecostomy D. Neostigmine E. Water soluble contrast enema
B
Coffee bean sign = sigmoid volvulus. The sigmoid colon is rotated to the LUQ. Ogilvies results in a diffusely dilated colon. There are no signs or symptoms of perforation or ischemia that would necessitate operative intervention. Further, the typical elderly pt population most at risk for sigmoid volvulus often has significant comorbidities that contribute to the high morbidity and mortality (~20%) of an emergency colon resection. Therefore exp lap or cecostomy are not indicated. 75% can be decompressed with sigmoidoscopy. Water soluble contrast enema is not necessary. Neostigmine is not indicated and might result in perforation and pain.
73 y.o. healthy woman c/o incr abdo dissension. She reports an episode 1 yr ago of LLQ pain for several days accompanied by constipation. This episode resolved without specific treatment. Otherwise, she has no chronic health conditions and takes no meds. Over the past 2 days, she has had decr appetite, nausea, bloating and LLQ abdo pain. She has had little to no stool passage over several days. No blood has been seen in her stool. Exam shows a grossly distended, tympanic abdo with minimal tenderness in the left abdo. Rectal exam finds no masses and no stool in the rectal vault. Plain AXR shows a distended colon from the cecum to the sigmoid colon.
Which of the following is the most common cause of her findings?
A. Colonic pseudo obstruction B. Diverticular stricture C. Colorectal carcinoma D. Inguinal hernia with incarceration E. Sigmoid volvulus
C
Symptoms of abdo distension, pain and constipation/obstipation are neither specific nor sensitive for specific etiology but they are a common constellation of findings in pts with acute colonic obstruction. Physical findings are non specific.
Colonic obstruction can result from several different processes including mechanical (diverticular stricture or incarcerated hernia), or dynamic (toxic megacolon or colonic pseudo obstruction) causes. Colon cancer is the most common cause of LBO followed by diverticular stricture and colonic volvulus.
Lack of hernia on exam typically exclude this etiology. Absence of toxic/ill appearing pt w/o fever, tachycardia or tenderness in a patient w/o abx use or IBD precludes toxic megacolon as a likely cause. These pts typically have diarrhea that is often bloody. Acute colonic pseudo obstruction (Ogilvie syndrome) is assoc with an underlying medical condition in 95% of cases or the use of certain drugs
Which of the following is optimal therapy for sigmoid volvulus without peritonitis?
A. Subtotal colectomy with end ileostomy
B. Urgent sigmoid colectomy with end colostomy
C. Endoscopic decompression, placement of rectal tube, and sigmoid colectomy with primary anastomoses
D. Intrahepatic decompression and sigmoid colopexy
E. Subtotal colectomy with ileorectostomy and diverting loop ileostomy
C
Colonic volvulus is 3rd most common cause of acute colonic obstruction and the sigmoid colon is the most commonly involved segment of colon in 66-75%. Sigmoid volvulus can present as an acute or subacute colonic obstruction and is often indistinguishable from a distal colon cancer based on symptoms. Differentiated by AXR. When dx remains in question, a gastrograffin enema is useful showing a birds beak deformity in the LLQ. In patient w/o findings concerning for ischemia or perforation, tx should begin with fluid resusc and non op decompression by rigid or flex sig and placement of rectal tube. Successful in 70-90%. Allows elective sigmoid colectomy with a primary anastomosis and complete colonoscopy to exclude a more proximal neoplasm. Definitive sigmoid resection should be done in medically fit pts during initial hospitalization b/c recurrence risk is high, occurring in at least 50%. Tube decompression allows the avoidance of resection in unprepared bowel, sparing the pt the need for a colostomy or ileostomy. Urgent colon resection is usually not indicated unless decompression fails or pt perforates.
82F with a hx of mild dementia and chronic back pain complains of nausea, lack of appetite and abdo discomfort. Takes donepezil and uses a fentanyl patch. Nursing home staff notice that she has a distended abdo and has not had a BM in past 4 days. On exam, her abdo is grossly distended, tympanic and has no significant tenderness. Her rectal exam is normal with minimal stool in rectal vault. Plain abdo X-rays show colonic distension with air from cecum to the rectum. She is after and has a normal WBC. K is 2.8, BUN is 28, Cr is 160. Which of the following would be the next most appropriate diagnostic test?
A. Decubitus abdo films B. Barium enema C. Gastrograffin enema D. Rigid proctosigmoidoscopy E. Stool guaiac test
C
Symp of abdo distension, pain and constipation/obstipation are neither specific nor sensitive for a specific etiology but are a common constellation in patients with acute colonic obstruction. Physical findings are non specific. May result form difference processes.
Acute colonic pseudoobstruction (Ogilvie’s) is characterized by massive colonic dilation in the absence of mechanical obstruction. Assoc with an underlying predisposing medical condition in 95% of cases or the use of certain pharmacological agents (narcotics, anticholinergics (donepezil), CCB.
B/c initial conservative mgmt of colonic obstruction, whether mechanical or Ogilvies is similar, pt should be NPO, NG, IV fluids, electrolyte correction, fluid status monitoring and potentially contributing meds d/c’ed. Ambulatory and changing the pt’s position in bed are important. Conservative mgmt successful in majority of pts, neostigmine at the outset is not indicated. In addition, without clearly knowing that the pt does not have a mechanical obstruction, neostigmine is not indicated. Water soluble contrast enema is the best choice for ensuring pt does not have a mechanical cause for obstruction. Barium is a poor choice b/c of risk of ischemia/perforation and risk of barium peritoneal contamination if a perf is present. Rigid proctoscopy is unlikely to view high enough in rectosigmoid colon to r/o mechanical obstruction.
51F presents with hematemesis. Upper scope reveals a mass arising from the proximal posterior gastric wall. CT confirms that this is an isolated lesion. Emergency laparotomy, gastrotomy, and wedge resection of tumor are performed. Tissue analysis reveals a 4.5 cm GIST with 1-2 mitosis per 50 hpfs. Proximal tumor margin is <1 mm. Which of the following is the most appropriate next step?
A. Re-exploration and celiac LN dissection
B. Proximal gastrectomy without LN dissection
C. Adjuvant external beam radiation
D. 1 yr of imatinib therapy
E. Observation with serial CT scanning only
E
GISTs are mesenchy mail tumors presenting most commonly in middle age and older patients. Surgical excision is the primary method of tx and microscopically involved margins do not affect survival. Lymphadenectomy is not required b/c GISTs rarely met to LNs. Avoid rupture of pseudocapsule during removal of GISTs
Tumors at high risk of recurrence are >5 cm in diameter, have more than 5 mitosis per 50 hpfs, have ruptures, or have multiple peritoneal mets. 5 yr recurrence rates for GISTs approach 50%. GISTs are refractory to traditional chemo and radiation.
GISTs are believed to arise from interstitial cells of canal, the pacemaker cells of the gut. Assoc with a high rate of mutation in kit protein, a tyrosine kinase receptor. Imatinib and sunitinib are effective as adjuvant therapy. Imatinib is approved for high risk kit positive tumors at a dose of 400 mg/day for 1 yr. In the presence of an exon 9 mutation, the dose of 800 mg/day is recommended. Sunitinib is offered as second line.
This scenario describes a low risk GIST for which adjuvant therapy is not indicated. Further resection or irradiation would be of no expected benefit. Observation and serial CT is the most appropriate course of management for this pt.
Which of the following statements regarding solitary rectal ulcer syndrome is correct?
A. They are always solitary
B. They are always ulcerated
C. Diminished anal sphincter tone is common
D. Fecal impaction is frequent
E. They may present as a polyploid lesion
E
SRUS is a manifestation of dysfunctional evacuation and usually presents with rectal bleeding. Excessive straining causes prolapse of the rectal mucosa that results in anterior or occasionally circumferential ischemic ulceration, which may be multiple. Sphincter tone is usually N or increase. As many as 40-70% of the pts will present with a polypoid lesion which may protrude through the anal canal. The lesions are typically located 4-12 cm from the anal verge. Histologic appearance of these lesions is highly characteristic, demonstrating fibromuscular obliteration of the lamina propria with hypertrophy of the muscularis mucosal and glandular crypt abN. Previously, recurrent rectal digitalization or foreign body trauma was thought to be the cause of SRUS; this is only true in a minority.
Biofeedback therapy can lead to improved anal sphincter relaxation, less straining, and more efficient defecation in patients with SRUS. SRUS should not be confused with stercoral ulceration of the rectum, which is assoc with prolonged fecal impaction.
Low grade, well differentiated liposarcoma found after resection of cord lipoma for a inguinal hernia repair in a 55yr old male A. observe B. radiate C. excision of cord structures D. retroperitoneal lymph node dissection
C. Although Rivard says A.
Adult spermatic cord sarcomas: Management and results. Annals of Surgical Oncology Volume 10, Issue 6, 2003, Pages 669-675
Liposarcomas are frequently low-grade lesions; however, their risk of local recurrence is comparable to that of high-grade lesions, and they should be aggressively treated. Patients with inadequately resected disease should undergo a reoperative procedure for wide inguinal re-resection, including completion orchiectomy or removal of the cord remnant to the internal inguinal ring with surrounding soft tissue and scar excision ade-quate to obtain a negative margin. The presence of high-grade sarcoma in these re-resected cases represents a greater risk of recurrent and metastatic disease. The definitive value of adjunctive radiation treatment or che-motherapy cannot be assessed in this small data set.
Posterior thigh liposarcoma, well differentiated, resected, positive microscopic margins at sciatic nerve A. re excise B. radiate C. chemotherapy D. observe
B
Because local control is still worse with positive as compared with negative margins, reresection to negative margins is preferred if additional conservative surgery can be performed. However, re-excision must take into account the size of the re-excised field, the potential for damage or exposure of critical neurovascular structures necessitating reconstructive surgical procedures, and the fact that only a minority of margin-positive (R1) resections ever result in a local recurrence (approximately 30 percent in most series), whereas not all complete (R0) resections avoid recurrence (5 to 10 percent in most series). In major sarcoma centers, positive margins in patients with extremity lesions usually reflect tumor on the neurovascular bundle and further surgery to achieve negative margins would usually entail an amputation; most patients in this setting are treated with higher doses of adjuvant RT (in the range of 66 to 68 Gy) with amputation reserved for salvage.
Long standing hx of celiac, push enteroscopy done, bx shows T cell lymphoma
A. resection and chemo
B. resection alone
C. h.pylori eradication
D. chemo
E. autologous stem cell transplantation after chemo
E
Enteropathy-associated T cell intestinal lymphoma (EATL), which is most often a sequela of celiac disease, is almost always of high-grade histology; the prognosis is poor and is worse than that of other intestinal lymphomas. Patients with EATL are commonly malnourished at the time of diagnosis and have a poor performance status. Treatment consists of combination chemotherapy used for other aggressive T cell lymphomas [80]. Patients who are candidates for autologous hematopoietic cell transplantation (HCT) may benefit from HCT in first remission… For patients with EATL who have a good performance status and chemotherapy sensitive disease, we suggest treatment with intensive chemotherapy followed by autologous HCT rather than chemotherapy alone.
Mets from renal ca to pancreas. 2 focal lesion appr 1cm. Located Head/Body best MGMT? A. Whipple B. Chemo Tx C. Enucleation D. Palliate
A
Multiple case reports/series where Whipple is used to treat isolated metastatic RCC to pancreas.
Patient with SCC, trial of aldera with redness, and pain, cause?
A. side effect of drug
B. normal reaction
C. fungal infection
B
KRAS mutation, which chemotherapy does not work on this gene? A. avastin B. oxiplatin C. irinotecan D. cetuximab
D
Merkel cell carcinoma 3cm on the thigh A. WLE with SLNB B. WLE with SLNB and radiation C. WLE D. Radiation
B
DFSP with fibrosarcomatous “changes” on scalp. Excised with 3 cm lateral and 5 mm deep margin. What next
a. Radiation
b. Re-excise
c. Observe
d. Chemo
C
WLE with 2-4cm margins to investing fascia or pericranium; reexcise until negative or clinically not feasible, in which case irradiate. MMS is an option in DFSP and FS variant. MMS goal is to achieve NEGATIVE margin, not set margins. Observe if all margins are negative. WLE is because DFSP has tentacle-like projections beyond what is clinically apparent.
Old guy. Melanoma right hand 5 cm. Two punch biopsies show lentigo maligna melanoma. A. Mohs B. Excise with Skin Graft C. Observe D. Imiquimod
B
50 F with 4cm mass in posterior thigh for 4 years! Now has dyspnea and CT done which shows 4 lesions, 1-2cm in the right lung which are consistent with mets. Management?
A. Resect thigh mass and lungs
B. Staged resection, first the thigh mass, then the lungs
C. Resect thigh mass, and radiate lungs
D. Chemotherapy
B
The use of neoadjuvant chemotherapy prior to resection of pulmonary metastases is not a widespread practice, and the decision to pursue this strategy must be made on a case by case basis or in the context of a clinical trial. For most adult-type soft tissue sarcomas, if there are concerns as to the natural history of newly diagnosed untreated metastatic disease, an alternative option is radiographic reassessment of disease status after an interval of six to eight weeks to allow the disease biology to declare itself, and spare patients with aggressive disease unnecessary surgery.
Terminal ileal mass, non-obstructing, biopsy showing large B-cell lymphoma A. Chemo B. Radiation C. Resection D. Chemo and radiation
A
For most patients with GI DLBCL, we recommend the use of combination chemotherapy plus rituximab (ie, R-CHOP) with or without involved-field RT such as that used for other patients with limited stage DLBCL rather than treatment with surgery or H pylori eradication therapy…surgery is reserved for patients with complications such as perforation or obstruction or intractable bleeding. This applies to both early stage and advanced disease.
Middle age male with previous hx of melena or anemia, workup (UGI and LGI scope negative). Now comes for elective bilateral inguinal hernia repair. Intraop, found to have a mass in SB consistent with GIST. What to do?
A. Resect SB mass and proceed with hernia repair
B. Resect SB mass and delayed laparoscopic hernia repair
C. Resect SB mass and delayed open hernia repair
D. Do hernia repair and delay SB resection
D
Not consented for small bowel resection. Not emergent. Other option is aborting procedure and having discussion re:bowel resection. Defer hernia until post mgmt of small bowel mass
Elderly female with 4.4mm melanoma on back, no lymphadenopathy, no distant mets. Management? A. wide resection 1cm B. wide resection 2cm C. wide resection 1cm w/ SLNB D. wide resection with 2cm w/ SLNB
D in answer key. Some controversy
As thick primary melanoma (>4 mm) places patients at an increased risk for distant metastatic disease, prior dogma held that SLN biopsy or lymph node dissection was not benecial for these patients. However, a number of studies have shown that thick melanoma patients with tumor-negative SLN have a better prognosis than those with tumor-positive SLN. One review of 240 patients with melanomas more than 4-mm thick found that 58% of patients had a negative SLN biopsy, and compared with patients with a positive node, a negative biopsy was associated with both improved distant disease-free survival and overall survival. Because there is a continuum of risk that does not abruptly end at 4 mm Breslow thickness, SLN biopsy for thick melanomas may provide improved regional disease control and possibly cure for these patients. Similarly, although the benefit remains unproven, there are reports detailing SLN biopsy for some patients with locally recurrent or in-transit melanoma.
Suspected BCC on face/ cheek. Best management? A. Biopsy B. Moh’s surgery C. Excision with clear margins D. Observe
A on one answer key and C on the other
Doesn’t mention size of lesion
Elderly man, excision of BCC on face. BCC is nodular/ sclerosing type, with perineural involvement, clear margin on path. Mx A. Re-excision B. Topical 5FU C. Chemo 5FU/cisplatinin D. Radiation
D
Melanoma 0.75mm depth, ulcerated, clark IV. Management A. Wide resection 1cm B. Wide resection 2cm C. Wide resection 1cm w/ SLNB D. Wide resection with 2cm w/ SLNB
C
Higher risk for LN mets due to ulceration and Clark IV
Young guy with skin lesion on flank, biopsied, comes back as spitz nevus on pathology. A. Excision with clear margins B. Excision with 1cm margin C. Excision with SLNB D. Observe
A
Spitz naevi are benign skin tumours. However, they may resemble malignant melanomas clinically and microscopically, so they are often excised as a precaution.
Skin lesions with clinical features of Spitz tumors should be removed by simple excision if there is concern for an atypical melanocytic lesion or melanoma. We suggest margins of approximately 3 to 5 mm.
Atypical Spitz tumors with positive margins should undergo reexcision to achieve clear margins given the difficulty in definitively distinguishing atypical Spitz tumors from melanoma.
Male with lower lip SCC. What LN basin will it spread to? A. Levels I and II B. Levels I and V C. Levels II and III D. Levels II and V
A
Male SBO from intussusception, Intraop found 2cm SB mass in distal ileum palpated, hard LN in mesentery, 3 mets to liver surface. A. Reduce intussusception B. Resect segment C. Resect segment, mesentery nodules D. Resect segment plus liver resection
C
18M hockey player. Persistent painful mass anterior thigh (6-8cm?), not fixed, no skin changes, no inguinal LN. What’s the best initial step? A. U/S B. MRI C. Core biopsy D. Excisional biopsy
B
Carcinoid, multiple bilobar liver mets. Now flushing, bronchospasm. What to do
A. Liver resection
B. Octreotide
C. Chemo
D. Imatinib
B
Multiple bilobar is classical contraindication to resection. Cytoreduction may be effective if can achieve 90% resection with FLR >20%
Post appy found to have 1.2 cm globet cell carcinoid. What to do now? A. Observe B. R hemi C. Ileocecal resection D. Octreotide
B
Young guy, 5 cm mass mid thigh, anterior compartment. Bx inconclusive. Imaging looks consistent with a sarcoma. Mgmt
A. Radiation
B. Excisional biopsy
C. Wide local excision
D. Excision of entire anterior compartment
C
Wouldn’t compromise limb function without definitive diagnosis, but in this location should be resectable without neurovascular compromise.
1.8cm neuroendocrine tumor with liver mets A. Whipple with liver resection B. Enucleation with liver resection C. Octreotide D. Chemo
A
Gastric cancer. Intra op, popped into abscess, and 40 mL of pus. Gastric Ca found to be invading into transverse colon and abdo wall
A. Palliative gastrojej
B. Gastrectomy, segmental colectomy, abdo wall resection
C. Gastrectomy, segmental colectomy
D. Gastrectomy
B
Which provides the most important information about prognosis of GIST? (asks about most important prognostic, not best or worst prognosis) A. Size >5cm B. 5 mitoses per 50HPF C. Midjejunal location D. Moderate grade
C
Jejunoileal site has lowest 5-yr survival.
Prognosis of gastrointestinal smooth-muscle (stromal) tumors: dependence on anatomic site Am J Surg Pathol, 23 (1999), pp. 82–87
Gastric GISTS, the largest group, comprising nearly two-thirds of all GISTs, generally have a better survival than small intestinal GISTs of similar size and mitotic activity.
Can’t find anything to stratify prognostic features of size & mitotic index, but SB GIST clearly has worse prognosis.
Melanoma 1.3 mm. Non ulcerated, with palpable, clinically positive lymp node. Management A. WLE with 1 cm margin and SLN B. WLE with 1 cm margin and ALND C. WLE with 2 cm margin and SLN D. WLE with 2 cm margin and ALND
D
Do 2 cm margin unless cosmetically unaccepatble then would do 1 cm
Patient with melanoma, no palpable nodes. During SLN in groin, encounter necrotic black node. Management of nodes
A. Complete SLN
B. Superficial groin lymph node dissection
C. Superficial and deep groin dissection
A
If it was a clinically palpable LN pre-op, do superficial groin LN dissection
Otherwise do SLN, await definitive path and if positive have discussion with patient re groin dissection in light of MSLT 2
Male with history of melanoma. Had CT for diverticulitis, incidental finding of mass in adrenal. Plasma metanephrine normal. What do you do next A. Urine cortisol B. FNA C. Adrenalectomy D. Observe
B
Fine-Needle Aspiration Biopsy:
- Cannot differentiate between adrenal adenoma and adrenocortical carcinoma
- Can differentiate between adrenal tumour and metastatic disease
- Therefore clinically indicated if known malignancy or suspected malignancy outside of adrenal12
- MUST rule out pheochromocytoma prior to biopsy and is ONLY indicated if high suspicion of metastatic disease that cannot be documented by biopsy of other tissue
If this was an melanoma met to adrenal
Resection of metastases to endocrine organs may provide palliation and prolong survival for symptomatic patients [21,22]. In one report, 18 of 27 patients (67 percent) with adrenal metastases from melanoma were rendered disease free surgically; the median survival was 26 months following complete resection compared with nine months after palliative resection. Laparoscopic adrenalectomy for metastatic disease is safe, effective, and associated with less morbidity than the open approach.
40 something male, skin lesion excised upper arm. Path is 1.5mm melanoma. Wound well healed with no sign of persistent tumor. A. 1-2 cm WLE B. 1-2 cm WLE and SLNB C. 2-4 cm WLE D. 2-4 cm WLE and SLNB
B
BCC on the chest A. Excise with microscopic clear margins B. Excise with 1cm C. Mohs D. Radiate E. Punch biopsy
A
If high risk features then 1 cm margin
Which tumor is the s-100 marker assoicated with: A. Renal cell B. Sarcoma C. Melanoma D. Ovarian
C
Melanoma is most likely to arise from: A. Lentigo B. Intradermal nevus C. Spitz nevus D. Blue nevi E. Congenital nevus
E
Low risk if small, 2-5% if large nevi, higher in giant CMN.
Which suggests melanoma in a pigmented lesion: A. Hair on a lesion B. Clear area around the lesion C. Regular borders D. Size greater then 2cms E. Ulceration
E
Differentiating mets melanoma from primary lesion:
A. Junctional activity
B. Dermal invasion
C. Ulceration on skin
A
Histologically, cutaneous metastases appear as nodules in the dermis or subcutaneous fat without an epidermal connection.
In the treatment of melanoma, important prognostic factors are, except:
A. Depth
B. Nodal involvement
C. Sex
D. Location – truncal, acral, mucosal have worse prognosis
E. Number of mitosis
C
34 yo female loses her great toenail. Pathology reveals a 3mm melanoma. What is the treatment:
A. Forefoot amputation
B. Transmetartasal amputation
C. Amputation at the metatarsal-phalangeal joint
D. Amputation through proximal phalange
E. Amputation to preserve proximal phalange
E
Whenever possible, subungual melanomas of the fingers should be resected at the distal interphalangeal joint to preserve function [53]. Melanomas located more proximally on the fingers can often be managed with wide local excision of soft tissue, skin grafts, or local flaps for soft tissue coverage.
Subungual melanomas involving the toes can be managed easily with digital amputation at the metatarsal-phalangeal joint. If the first toe is involved, complete amputation should be avoided when oncologically feasible because of the importance of the toe in balance.
Persistent pigmented lesion of the great toe. The best treatment: A. Amputation B. Excisional bx C. Incisional bx D. Observe
C
There is general consensus that incisional biopsy may be occasionally acceptable for very large lesions or for certain sites, including the face, palm or sole, ear, distal digit, or subungual lesions
Merkel cell ca: A. Old people B. Slow growing C. Increased mets tendency D. Small size
A and C
Which is true regarding skin cancer post renal transplant:
A. More often multiple compared to single
B. More often on the back
C. More often in females
D. More often melanoma
E. Usually in first three years
A
RF: greater age at transplant, UV exposure, pre-transplant skin CA, HPV, type of transplant (liver is least). More likely to develop multiple and more aggressive tumours. SCC most common
With regards to melanoma, which is true:
A. Management for a 3mm lesion may be a 2cm resection margin
B. The treatment for back lesions are more aggressive than those on the legs
C. Prognosis for women are worse than men
A
Merkel cell carcinoma 3cm on the thigh A. WLE with SLNB B. WLE with SLNB and radiation C. WLE D. Radiation
B
Post mastectomy hematoma, 25 cm, drain not draining A. Evacuation in OR B. Drain at beside with 2cm incision C. Irrigate drain D. Observe
A
35 year old, has BRCA2, mammo with birads 2 and 75% density, what next A. Mammogram next year B. MRI now C. US now D. Bilateral mastectomy
B
40F 2cm spiculated mass seen on mammongram, core biopsy came back as radial scar . What is the best MX? A. Excisional biosy B. Do nothing C. Mastectomy D. Recore biospy
A
UTD
There is ongoing controversy about the need for surgical excision when radial scars are found on core biopsy. We suggest that these be excised since most series show that 8 to 17 percent of surgical specimens at subsequent excision are positive for malignancy.
Breast cancer 5cm with 4 positive nodes on dissection. Triple negative. How to treat adjuvantly? A. Chemo and rads B. Rads only C. Chemo only D. Hormonal therapy
A
Post-mastectomy radiation improves OS and DFS. Indicated in large primary, 4 or more nodes +.
40yo lady with persistent serous drainage from breast, single duct at 9 o’clock . What to do FIRST? A. Bilateral mammo B. Galactogram C. U/S D. MRI
A
73F with 7.5cm ER+ her2- breast ca. 1.2cm node with normal hilum on imaging. Bone scan consistent with multiple bone mets.
A. MRM and tamoxifen
B. AI followed by tamoxifen if no tumor response
C. AI followed by chemo if no tumor response
D. AI followed by resection if tumor response
B
Adjuvant endocrine is preferred. Can trial 2nd line endocrine if no response to AI. Also needs bisphosphonate.
40F with recurrent breast infections, 2 previous I and Ds, fistula opening found lateral to areola A. Antibiotics B. I and D C. NAC resection D. Resection subareolar ducts
D
Metastatic breast cancer, tachypneic, tachycardia, sbp 95, distended neck veins, b/l effusions and cardiomegaly on cxr A. Bilat chest tubes B. Perc pericardiocentesis C. Echo D. Ct angio
C
Presumably cardiac dysfunction related to chemo
PE shouldn’t cause cardiomegaly
35 year old female with axillary node positive for cancer on FNA, no primary lesion found on full workup A. Modified radial mastectomy B. Axillary dissection C. Axillary dissection with radiation D. Radiation
A
UTD:
Either MRM or ALND with WBI is acceptable with equivalent outcomes. MRM is standard therapy, and there are no prospective trials validating WBI as an alternative. Systemic therapy for stage II disease.
Rapidly growing 4cm angiosarcoma of breast in middle aged ish woman A. Mastectomy B. Partial mastectomy C. Mastectomy and SNB D. Radiation
A
Pregnant 12weeks, small invasive breast cancer, palpable nodes (biopsy of node not done). Plan?
a. Lumpectomy and Cx and Rx
b. Lumpectomy and Cx
c. Lumpectomy and Rx
d. Terminate pregnancy
B in the answer key
But lumpectomy requires radiation to have equivalent outcome. Could do modified radical mastectomy with SLNB.
But technically you could bx LN and if positive, treat with neoadjuvant chemo then lumpectomy and give radiation post partum
Pt with Lupus and has cribiform 0.9mm DCIS focal in upper outer quadrant. How to treat A. Total mastectomy and SLNBx B. Lump and Rx C. Lump and chemo D. WLE/lump
A
Lupus is a relative contraindication to radiation
Diffuse DCIS , mastectomy and deep margin 2mm – what to do? A. Nothing B. Rx C. Re-excision of scar and deep margin D. Chemo
A
Margin is negative; goal is minimum 2mm margins. If postive, radiate bed. NO data on post-mastectomy RT for DCIS! However, do need to treat the contralateral breast with surveillance and tamoxifen or AI depending on menopausal status.
45ish with 4.5 cm mass at level 3 area in neck, was under skin, cystic and solid with microcalcification, hypervascular A Panendsocopy B. US C. Excisional bx D. Ct/pet
D
Presuming they already did an U/S? Otherwise how do they know the characteristics of the lesion???
Primary hyperparathyroidism diagnosed after having kidney stone. Did spect CT and perhaps right inferior shows up, plan for OR
A. Focused exploration with intraop PTH
B. Bilateral exploration
C. Radio guided intraop unilateral exploration
A
When combined with use of intraoperative PTH monitoring, minimally invasive parathyroidectomy techniques result in excellent outcomes that are comparable to a traditional bilateral cervical exploration [23-27]. Localization results inform the surgeon where to start looking for the adenoma, and intraoperative PTH results suggest to her/him when to stop looking.
Best surveillance for patient who had RAI and total thyroidectomy, most sensitive A. Stimulated thyroglobulin B. Anti-thyroglobulin antibody C. PET D. US
A
55M, non-smoker present with enlarging mass on R side of neck. Biopsy shows “bland squamous cells with lymphocytes”. Most likely diagnosis? A. Acinar cell carcinoma B. Metastatic SCC C. Branchial cleft cyst D. Thyroglossal duct cyst
B
In middle-age and older, SCC»_space;> BCC, so have to assume that’s what its. Well-differentiated SCC can look bland. Lymphocytes confirm this is within a lymph node, which is less likely with BCC. Remains controversial due to bland cytology, but we decided to go with an age-based probability.
Lady about to be put asleep for a 2cm PTC of thyroid and tells anaesthesiologist that she is 6 weeks pregnant. What to do?
A. Continue with surgery
B. Hold surgery and put on TSH suppression and do surgery after birth
C. Hold off until second trimester
D. Hold off until after birth
B
UTD:
Women with differentiated thyroid cancer require surgery. However, given the typically indolent nature of thyroid cancer, thyroidectomy is usually delayed until the postpartum period to minimize maternal and fetal complications [55,61]. This approach does not appear to have a negative impact on prognosis
When surgery for biopsy-proven thyroid cancer is deferred, the patient should be monitored during pregnancy with thyroid ultrasound performed during each trimester. If, by 24 weeks, there is a significant increase in thyroid cancer size (50 percent in volume or 20 percent in diameter in two dimensions), surgery should be performed during the second trimester [9,46]. However, if the size remains stable or if it is diagnosed in the second half of pregnancy, surgery may be performed after delivery. In such cases where thyroid surgery is deferred, we suggest thyroid hormone suppressive therapy with a goal of maintaining the TSH in the range of 0.3 to 2.0 mU/L.
Surgery during pregnancy is sometimes indicated for rare patients with larger, more aggressive or rapidly growing cancers or in the presence of extensive nodal or distant metastasis. The safest time for any type of surgery during pregnancy is the second trimester
Patient with 2.5 cm thyroid nodule. Amyloid on FNA. Mx?
A. Total thyroidectomy, central node dissection, ipsilateral neck dissection
B. Observe
C. Hemithyroidectomy
D. Radioactive iodine
A
Medullary thyroid carcinoma–> amyloid on FNA.
ATA unclear on w
hether to do a prophylactic lateral LND if clinically negative, but size >2cm is reasonable to do it. Overall, the approach is to do neck ultrasound and calcitonin levels. Best answer is probably to do total and central LND (ATA 26), no lateral LND unless clinically/ radiographically positive OR CEA >200.
Primary hyperparathyroidism, left inf adenoma localized pre-op. Identified and removed at OR, couldn’t find the Left superior, Path shows normal gland. Persistent hypercalcemia post-op, goes back for 4 gland exploration. Found to have Left adenoma intra-thymic. Most likely etiology of intra-thymic mass. A. Supernumerary left para B. Ectopic superior left parathyroid C. Double adenoma D. Hyperplasia
A
13%, most commonly in thymus.
Middle aged lady with slowly growing lump at angle of the mandible and FNA is indeterminant A. Core biopsy B. Observe C. Nodule excision D. Superficial parotidectomy
D
Lateral neck mass middle aged male. Fixed in Sup/Inf plane, mobile in A/P plane. located at level of hyoid near carotid artery. Best next investigation A. FNA biopsy B. CTA C. Doppler U/S D. 24 hour urine metanephrines
B
Carotid duplex scan can localize the tumor to the carotid bifurcation, but CT or MRI is usually required to further delineate the relationship of the tumor to the adjacent structures.
Lady with RET proto-oncogene mut (MEN 2B). Has a kid. When should the baby get thyroidectomy? A. Within 1st year of life B. Before school C. Before age 10 D. During adolescence
A
B=bad; thyroidectomy before age 2.
Huerthle cell lesion on FNA. 2.8cm in left lobe. no lymph nodes on ultrasound. A. Total thyroid B. Repeat FNA in 6 months C. Thyroid lobe with isthmus D. Radioactive iodine ablation
C
ATA
If the reading is ‘‘suspicious for papillary carcinoma’’ or ‘‘Hurthle cell neoplasm,’’ a radionuclide scan is not needed, and either lobectomy or total thyroidectomy is recommended, depending on the lesion’s size and other risk factors. Recommendation rating: A
Female with hemithyroidectomy. Final path = 4cm follicular adenoma and 6mm papillary ca. A. Completion total B. TSH suppression C. RAI ablation D. Repeat u/s
B
ATA
RECOMMENDATION 26: For patients with thyroid cancer >1 cm, the initial surgical procedure should be a near-total or total thyroidectomy unless there are contraindications to this surgery. Thyroid lobectomy alone may be sufficient treatment for small (<1 cm), low-risk, unifocal, intrathyroidal papillary carcinomas in the absence of prior head and neck irradiation or radiologically or clinically involved cervical nodal metastases.
RECOMMENDATION 29: Completion thyroidectomy should be offered to those patients for whom a near-total or total thyroidectomy would have been recommended had the diagnosis been available before the initial surgery. This includes all patients with thyroid cancer except those with small (<1 cm), unifocal, intrathyroidal, node-negative, low-risk tumors.
RECOMMENDATION 40: Initial TSH suppression to below 0.1mU=L is recommended for high-risk and intermediate-risk thyroid cancer patients, while maintenance of the TSH at or slightly below the lower limit of normal (0.1–0.5mU=L) is appropriate for low-risk patients. Similar recommendations apply to low-risk patients who have not undergone remnant ablation, i.e., serum TSH 0.1–0.5mU=L.
Guy with lesion in thyroid, biopsy shows MTC. multiple enlarged nodes on US in central compartment. Management?
A. Total thyroidectomy
B. Total thyroidectomy with ipsilateral Level VI dissection
C. Total thyroidectomy with bilateral Level VI dissection
D. Total thyroidectomy with bilateral Level VI dissection and ipsilateral modified radical neck dissection
C
LLND is reasonable but not mandated by guidelines; in presence of bulky CC nodal disease or >2cm primary OR calcitonin <200, LLND is very reasonable but no consensus was reached
1. Hypercalcemia, low urinary calcium and low phosphate. Most likely cause: A. Primary HyperPTH B. Secondary HyperPTH C. Tertiary HyperPTH D. FHH + Vit D deficiency
D
Primary HyperPTH – normal or high urinary Ca, unless concomitant Vit D deficiency
Secondary HyperPTH – no, would have high phosphate
Tertiary HyperPTH – low phosphate
PHPT with Vit D deficiency can also cause low urinary Ca and hypercalcemia.
Management of MTC. No palpable lymph nodes, no U/S nodes
A. Total thyroidectomy with central compartment
B. Total thyroidectomy
C. Total thyroidectomy with laternal neck
D. Total thyroidectomy with central compartment and lateral neck
A
RECOMMENDATION 61: Patients with known or highly suspected MTC with no evidence of advanced local invasion by the primary tumor, no evidence cervical lymph node metastases on physical examination and cervical ultrasound, and no evidence of distant metastases should undergo total thyroidectomy and prophylactic central compartment (level VI) neck dissection (Grade B recommendation).
Mucoepidermoid Ca of parotid. Definition of functional neck dissection
A. Level 1-5, preservation of SAN, SCM, IJV
B. Level 1-5, preservation of SAN
C. Level 2-4, preservation of SAN
D. Cherry picking
A
Papillary thyroid Ca, U/S negative for LN, no FHx/RT
A. Total thyroidectomy with central compartment (only if T3/4)
B. Total thyroidectomy
C. Total thyroidectomy with laternal neck
D. Total thyroidectomy with central compartment and lateral neck
B